Pharma

You might also like

Download as pdf or txt
Download as pdf or txt
You are on page 1of 114

SELF-ASSESSMENT STUDY GUIDES Page 1 of 9

1. How should you examine through a head mirror in an Answer: [b] binocular
ENT examination? Rationale:
a. Monocular • View should be binocular
b. Binocular
c. Both Reference: [ENT]_001_Basic ENT Examinations, page 1

2. The ventral saccular and dorsal utricular. Ventral Answer: [d] both are correct
saccular is for cochlear function. Dorsal utricular is for Rationale:
vestibular. • The otic vesicle divides into a dorsal utricular
a. First statement is correct portion and ventral saccular portion, with the
b. Second statement is correct dorsal utricular portion giving rise to the
c. Both are wrong vestibular system and the ventral saccular
d. Both are correct portion giving rise to inner ear structures like the
organ of corti that are involved in hearing. The
ventral saccular portion develops into the
cochlear duct (which houses the organ of corti)
and saccule. The dorsal utricular portion forms
into the utricle, semicircular canals, and
endolymphatic tube.

Reference: Anatomy, Head and Neck, Ear Organ of Corti


https://www.ncbi.nlm.nih.gov/books/NBK538335/
3. Diameter of the inner hole of head lamp Answer: [a] 1/2 inch
a. 1/2 inch Rationale:
b. 1/4 in • 3 1⁄2 inch mirror with a 1⁄2 inch hole at the
c. 1 inch center
d. 2 in
Reference: [ENT ](001) BASIC ENT EXAMINATION
4. What is the common infection acquired congenitally? Answer: [a] CMV
a. CMV Rationale:
b. HZV • When infection during pregnancy is clinically
c. N.gonorrhoeae suspected, laboratory tests to detect congenital
infection might include those for
cytomegalovirus, herpes simplex virus, rubella,
HIV, toxoplasmosis, syphilis, and Zika virus

Reference:
https://www.cdc.gov/ncbddd/birthdefects/surveillancemanual/chapters/chapt
er-
5/chapter5.html#:~:text=When%20infection%20during%20pregnancy%20is,%2
C%20syphilis%2C%20and%20Zika%20virus.

5. After 3 weeks of treating OM w/ perfusion, there is Answer: [b] Examination of the nasopharynx for possible
persistence of symptoms. What is the next best step? new growth
a. Examination of the oropharynx for possible Rationale:
new growth • If persistent for more than 3 weeks – Must do
b. Examination of the nasopharynx for possible endoscopic examination of the nose and
new growth nasopharynx to rule out tumor growth
c. Initiation of antibiotic therapy
Reference: [ENT]_005_diseases of the middle ear, page 6
d. Do tympanocentesis
6. From what inorganic materials do otoliths are made Answer: [c] CaCo
from? Rationale:
a. Hydroxyapatite crystals • Otoliths are inorganic crystalline deposits made
b. Collagen of CaCO.
ENT
SELF-ASSESSMENT STUDY GUIDES Page 2 of 9

c. CaCo
d. HCl Reference: [ENT]_006_anatomy & physiology of the inner ear, page 7

7. The potassium gradient is essential to enable Answer: [b] displacement of tip links
depolarisation of the hair cell due to the influx of Rationale:
potassium ions. This is due to event/s: • the mechanical displacement of the basilar
a. Deflection of stereocilia membrane from the traveling wave. the cellular
b. Displacement of tip links depolarization and subsequent synaptic activity
c. Opening of potassium gated channel in the hair cells
d. All of the above
Reference: [ENT]]_006_anatomy & physiology of the inner ear, page 3

8. The tensor tympani muscle is inserted into the: Answer: [b] Handle of malleus
a. Lateral process of malleus Rationale:
b. Handle of malleus • TENSOR TYMPANI
c. Handle of stapes o ORIGIN Cartilage of the auditory tube and
d. Neck of stapes the bony walls of its own canal
o INSERTION Into the handle of malleus

Reference: [ENT](004) ANATOMY AND PHYSIOLOGY OF THE MIDDLE EAR

9. Which of the following stages is characterized by Answer: [b] exudation


outpouring of fluid from permeable capillaries? Rationale:
a. Hyperemia Stage of Exudation
b. Exudation • More outpouring of fluid from the dilated
c. Suppuration permeable capillaries
d. Coalescence • All symptoms are aggravated especially more
pain and more fever
• More red and thickened bulging eardrum with
loss of the light reflex is seen on otoscopy
• More fluid gathering in tympanic membrane until
it ruptures (the next stage)

Reference: [ENT]_006_anatomy & physiology of the middle ear

10. These are responsible for linear acceleration. Answer: [d] otoliths
a. Posterior SCC Rationale:
b. Utricle and Saccule • 2 otoliths/maculae – linear accelerations
c. Ampulla (example: riding an elevator or car); Utricle in the
d. Otolith horizontal plane; Saccule in the vertical plane

Reference: [ENT]_006_anatomy & physiology of the inner ear

11. Point of attachment basilar membrane. Answer: [Spiral lamina]


a. Stria vascularis Rationale:
b. Helicotrema • Spiral lamina
c. Stria kanina • Coils around the center
d. Modiolus • Partial division of Scala Tympani and
Scala Vestibuli
• Point of attachment for BM

Reference: [ENT]_006_anatomy & physiology of the inner ear


12. The ff are not recommended as management in Answer: [a] Flu vax
Acute otitis media in children, except: Rationale:
a. Flu vax • Administration of influenza vaccine
ENT
SELF-ASSESSMENT STUDY GUIDES Page 3 of 9

b. Probiotics demonstrated efficacy in the prevention of AOM


c. Decongestants by 30% to 55%.
d. Antihistamines
Reference: [ENT]_005_Diseases of the middle ear, Page 11
13. Where does the external ear arise from? Answer: [a] First branchial pharyngeal cleft ectoderm
a. First branchial pharyngeal cleft ectoderm Rationale:
b. Second branchial pharyngeal cleft ectoderm • External ear will arise from the first branchial
c. First branchial mesoderm pharyngeal cleft ectoderm
d. Second branchial mesoderm
Reference: [ENT]_002_Anotomy and physiology of the external ear, page 3

14. What articulates with the head of the malleus to Answer: [b] Body limb of incus
form an ossicular mass that occupies the majority of Rationale:
the epitympanum? • Body of the incus articulates with the head of the
a. Short limb of incus malleus to form an ossicular mass that occupies
b. Body of incus the majority of the epitympanum
c. Long limb of incus
d. Head of stapes Reference: [ENT]_005_Diseases of the middle ear, Page 2

15. Pain resolves in 48-72 hrs: damaged epithelium peels Answer: [d] Crista
off, leaving no residual scars. Rationale:
• Crista is a saddle-shaped, raised across the floor
or the ampulla at the right angles to its long axis.

Reference: [ENT]_006_Anatomy and Physiology of the Inner Ear, page 6


16. These are responsible for linear acceleration. Answer: [d] otoliths
a. Kinocilium Rationale:
b. Stereocilia • 2 otoliths/maculae – linear accelerations
c. Capula (example: riding an elevator or car); Utricle in the
d. Crista horizontal plane; Saccule in the vertical plane

Reference: [ENT]_006_anatomy & physiology of the inner ear


17. Which of the following stages of acute otitis media is Answer: [c] Suppuration
characterized by perforation of the tympanic Rationale:
membrane? • Eardrum ruptures and there is a lot of discharge
a. Posterior SCC seen from thr middle ear
b. Utricle and saccule
c. Ampulla Reference: [ENT]_006_anatomy & physiology of the middle ear
d. Otoliths

18. Measurement of head mirror. Answer: [a] 3 1/2 inch


a. 3 1/2 inch Rationale:
b. 2 1/2 inch • The head mirror is 3 1/2 inch with a ½ inch hole at
c. 1 1/2 inch the center with a focal length of 14 inches.
d. 4 1/2 inch
Reference: [ENT]_001_Basic ENT Exam, page 1

19. Needed to establish a diagnosis of acute otitis media Answer: [c] A thorough history and pneumatic otoscopy
in children. General physical exam Rationale:
a. CBC • Otitis media is diagnosed clinically via objective
b. A thorough history and pneumatic otoscopy findings on physical exam (otoscopy) combined with
c. Radiograph the patient's history and presenting signs and
symptoms. S/S of acute otitis media are opaque,
bulging or congested TM., limited or no mobility of
TM on pneumatic otoscopy,otorrhea, otalgia and

ENT
SELF-ASSESSMENT STUDY GUIDES Page 4 of 9

fever.
Reference: [ENT]_005_Diseases of middle ear, page 3

20. Vestigial lymph channels that drain into the Answer: [a] Fissures of Santorini
superficial parotid gland. Fissures of Santorini Rationale:
a. Foramen of Huschke • Fissures of Santorini—> vestigial lymph channels that
b. Foramen of Luschka drain into superficial parotid gland and is visible
c. Fissures of Morgagni though inspection

Reference: [ENT]_002_Anotomy and physiology of the external ear, page 4


: [Pedia]_012_shock, page 1
21. These are sensorineural nerves of the external ear, Answer: [a] CN 2
except Rationale:
a. CN 2 • The external auditory canal is innervated by the
b. V nerves C3, V, VII, and X.
c. VII
d. X

Reference: [ENT]_002_Anatomy and Physiology of the External Ear, page 3

22. All of the following are NOT recommended for Acute Answer: [c,d] Antihistamine, Decongestant
Otitis Media, EXCEPT:. Rationale:
a. Flu vaccine • Supportive treatment for the management of
b. Probiotics AOM include Analgesics (NSAIDs or APAP),
c. antihistamine decongestants, antihistamines, antipyretics, and
d. Decongestant local heat.

Reference: [ENT]_005_Diseases of the Middle Ear, page 3


23. What is the posterior surface of temporal bone Answer: [d] Anterior limit of the posterior fossa.
musculocutaneous Rationale:
a. side of the head • The temporal bone has a pyramidal shape, the
b. middle cranial fossa sides of which form the middle fossa floor
c. mastoid bone (superior face), the anterior limit of the posterior
d. anterior limit of the posterior fossa fossa (posterior face), muscle attachments of the
neck and infratemporal fossa (anterior-inferior
face), and the muscular-cutaneous cover side of
the head (lateral, which form the base of the
pyramid.

Reference: Cummings Otolaryngology Head and Neck Surgery 7th ed.


Ch126 p1928.
24. A part of the ossicular chain that is a single nutrient Answer: [b] Lateral Process of the Incus
vessel, absence if collateral circulation renders this Rationale:
segment to be susceptible to aseptic necrosis.
a. Handle of the Malleus Reference: Cummings Otolaryngology Head and Neck Surgery 7th
ed.Ch126 p1934 /
b. Lateral Process of the Incus [ENT]_004_Anatomy and Physiology of middle ear, page 4
c. Long Process of the Malleus
d. Neck of the Malleus

ENT
SELF-ASSESSMENT STUDY GUIDES Page 5 of 9

25. Insertion of the tensor tympani Answer: [b] Handle of the malleus
a. Lateral process of the malleus Rationale:
b. Handle of the malleus
c. Neck of the. Stapes
d. Head of the malleus

Reference:[ENT]_004_Anatomy and Physiology of middle ear, page 3


26. Not recommended management in AOM except Answer: [a] flu vaccine
a. Flu vaccine Rationale:
b. Decongestant • Clinicians may recommend annual influenza
c. Antihistamine vaccine to all children
d. Probiotics • The use of antihistamine and/or decongestant
therapy is not recommended for the treatment of
acute otitis media.
• Probiotics are not recommended for the
prevention of Acute Otitis Media in children.

Reference: [ENT]_005_Diseases of the middle ear, Pp3-4


27. Point of entry of peripheral auditory information to Answer: [b] cochlear nucleus
central auditory system Rationale:
a. Cochlear nerve • Cochlear nucleus
b. Cochlear nucleus o Point of entry of peripheral auditory
c. Superior olivary complex information to central auditory system
d. Medial geniculate body o Critical first relay station for all ascending
auditory information.
o Pontomedullary junction of the dorsolateral
brainstem.

Reference: [ENT]_006_Anatomy and Physiology of the inner ear, p4

28. This drug toxicity will alter the mechanism of cochlea. Answer: [furosemide]
Rationale:
• Loop diuretics may alter the potassium gradient
between the chambers of the cochlear, affecting
its function.

https://www.medsafe.govt.nz/profs/PUArticles/June2016/MedicineInducedHea
ringLoss.htm
29. What aog does the external ear achieve its adult Answer: [a] 20 wks
shape Rationale:
a. 20 wks • 20th week – adult shape, adult size by 9y/o
b. 21 wks
c. 22 wks Reference: [ENT]_002_Anatomy and Physiology of the External Ear, page 2

d. 9 yrs old
30. Tympanomastoid suture line is a surgical landmark of Answer: [c] exit of main trunk of facial nerve

ENT
SELF-ASSESSMENT STUDY GUIDES Page 6 of 9

which anatomic structure? Rationale:


a. anterior fossa floor • This is equivalent to posterior fossa. This line is
b. midline fossa floor also the most reliable landmark for parotid
c. exit of main trunk of facial nerve surgery.
d. styloid foramen
Reference: [ENT]_002_Anatomy and Physiology of the External Ear, page 2
31. Temporal bone articulates with the following, Answer: [d] parietal
except: Rationale:
a. Zygomatic • The temporal bone articulates with sphenoid,
b. Ethmoid parietal, OCCIPITAL and zygomatic bones.
c. Sphenoid
d. Parietal Reference: [ENT]_002_Anatomy and Physiology of the External Ear, page 1

32. What is the optimal distance of the patient’s head Answer: [d] 14 inches
from the head mirror? Rationale:
a. 10 inches • The optimal focal distance of the head mirror
b. 13 inches from the patient's head is 14 inches
c. 12inches
d. 14 inches Reference: [ENT]_001_Basic ENT examinations

33. Hearing loss w/ retinitis pigmentosa and vestibular Answer: [b] Usher syndrome
dysfunction Rationale:
a. stickler syndrome • Usher syndrome presents with SNHL + retinitis
b. Usher syndrome pigmentosa vestibular dysfunction
c. waardenburg syndrome
d. pendred syndrome Reference: [ENT]_007_Diseases of the inner ear

34. Which of the following is a good substitute for a head Answer: [a] A head lamp that can be focused
mirror? Head Mirror
a. A head lamp that can be focused • Substitute-focusable light on a head band
b. A handheld flashlight
Reference: (001) BASIC ENT EXAMINATIONS pg 1
c. A goose neck lamp directed onto the
examined area
d. A handheld light source held by an assistant
35. What is the turbinate readily visible when you do Answer: [b] Inferior turbinate
anterior rhinoscopy?
a. Supreme Turbinate Reference: (001) BASIC ENT EXAMINATIONS pg 3
b. Inferior turbinate
c. Middle turbinate
d. Superior turbinate
36. True about noise-induced hearing lost except Answer: [c] Greatest at 2 KHz
a. Almost always symmetrical and bilateral Rationale:
b. Greatest at 2 KHz • NIHL for each ear be quantified as the average NIHL
c. Rapidly progresses from 10 to15 years of across the frequencies 1, 2, and 4 kHz.
exposure Reference:
d. Sudden loss of hearing due to exposure to https://www.ncbi.nlm.nih.gov/pmc/articles/PMC9052822/
sound that is too intense and too long
37. At what level does acute hearing loss occur? Answer: [c] >130
a. >100 Rationale:
b. >120 • Sounds louder than 130 dB can cause acute
c. >130 hearing loss
d. >140
Reference: https://www.ncbi.nlm.nih.gov/books/NBK390300/
38. The head mirror has a hole at the middle which has a Answer: [b]½ in

ENT
SELF-ASSESSMENT STUDY GUIDES Page 7 of 9

diameter of? Rationale:


a. 1 inch • The head mirror is 3 1/2 inch with a ½ inch hole at
b. ½ in the center with a focal length of 14 inches.
c. ¾ in
Reference: [ENT]_001_Basic ENT Exam, page 1
d. ¼ in
39. Acute otitis media Answer: [b]less than 3 wks
a. less than 4 wks Rationale:
b. less than 3 wks • Acute (3weeks), Subacute (3mos),
c. more than 2 wks • Chronic (6mos)
d. less than 5 wks
Reference: [ENT]_005_Diseases of the middle ear
40. Superior border of Scala Media Answer: [a] Reissner’s Membrane
a. Reissner’s Membrane Rationale:
b. Basilar Membrane
c. Tectorial Membrane
d. Stria Vascularis

Reference: [ENT]_006_Anatomy & Physiology of the Inner ear, page 2


41. All of the following are true about the fluid systems Answer: [d] All of the above
a. Composed of Perilymphatic systems and Rationale:
Endolypmphatic system • Fluid Systems: Perilymphatic systems and the
b. Mechanical displacement of BM traveling Endolypmphatic syste. Mechanical displacement
wave of BM traveling wave. Cellular depolarization –
c. Cellular depolarization – synaptic activity synaptic activity

Reference: [ENT]_006_Anatomy & Physiology of the Inner ear, page 2

42. the following are the sensory innervation of external Answer: [a] C2
ear, except:
a. C2
b. VIII
c. V
d. X

Rationale:
Reference: [ENT]_002_Anatomy and Physiology of the external ear
43. Which is of the following cannot be used as Answer: [d] Sterile saline solution
cerumenolytics Rationale:
a. Sodium • The hardened cerumen can be softened using ear
b. Hydrogen peroxide drops (e.g., docusate sodium,
c. Oil paradichlorobenzene), oil, or hydrogen peroxide.
d. Sterile saline solution
Reference: [ENT]_003_Diseases of the External Ear, page 1

44. Located at the anterior neck of malleus serve as first Answer: [b] Prussak
site of cholesteatoma Rationale:

ENT
SELF-ASSESSMENT STUDY GUIDES Page 8 of 9

a. tegmen • The the Prussak space, just medial to pars


b. prussak flaccida and lateral to the head and neck of the
c. anterior process of mallus malleus
d. aditus • Prussak's space localization represents the early
stage of the cholesteatoma arising from the pars
flaccida of the tympanic membrane.
Reference: [ENT]_006_Diseases of the Middle Ear;
https://pubmed.ncbi.nlm.nih.gov/1579668/#:~:text=Prussak's%20space%20loc
alization%20represents%20the,and%20to%20mastoid%20air%20cells.
45. Treatment of hematoma in external ear Answer: [b] Drainage
a. Observation Rationale:
b. Drainage • The Evacuation of hematoma- Recommended
c. Antibiotic treatment is just draining it
d. Pressure dressing
Reference: [ENT]_003_Diseases of the External Ear, page 2

46. What structure is immediately seen when conducting Answer: [c]Inferior turbinate
anterior rhinoscopy? Rationale:
a. Superior turbinate • The first structure that you would see when doing
b. Middle turbinate anterior rhinoscopy is the inferior turbinate
c. Inferior turbinate
Reference: [ENT]_001_Basic ENT Examination
d. Nasal septum
47. Which of the following is true about using a head Answer: [a] headlight of 100 watts and more than mounted
mirror in doing ENT basic examination? to a goose neck lamp
a. headlight of 100 watts and more than Rationale:
mounted to a goose neck lamp • Focal length should be 14 inches
b. The focal length is 16 inches • Examiner directs the area to be examined into
c. The examiner has the freedom to move the field of view and avoids repositioning himself
d. Option d Reference: [ENT]_001_Basic ENT Examination

48. A 60 year-old female presented with otalgia and Answer: [a] Ramsay-hunt syndrome
aural discharge for 3 days. On physical examination Rationale:
there was noted vesicles in the auricle, EAC, and face • Painful vesicles in EAC and auricle with seventh
of the patient. There is also noted decreased nerve involvement. It presents with cutaneous
movement of the facial muscles in the ipsilateral side. herpes in a dermatomal manner. It is caused by
What is the most probable diagnosis? chickenpox virus.
a. Ramsay-hunt syndrome
Reference: [ENT]_003_Diseases of the Middle Ear
b. Otomycosis
c. Diffuse otitis externa
d. Perichondritis

49. Inferior boundary of the middle ear Answer: [c] Internal jugular vein
a. Tegmen Rationale:
b. Aditus • Lateral: tympanic membrane
c. Internal jugular vein • Superior: Tegmen tympani
d. Eustachian tube • Posterior: Mastoid
• Anterior: Carotid
• Medial: Labyrinth
• Floor: Internal jugular vein
Reference: [ENT]_004_Anatomy and Physiology of the Middle Ear
50. Treatment of Migraine Associated Vertigo Answer: [b] Regular sleep
a. Anaerobic Exercise Rationale:
ENT
SELF-ASSESSMENT STUDY GUIDES Page 9 of 9

b. Regular sleep • Treatment of Migraine-Associated Vertigo


c. Beta Agonist o Control diet
d. Benzodiazepines o Regular sleep
o Aerobic Exercise

Reference: [ENT]_007_ADiseases of the Inner Ear


51. What is the focal distance of the head mirror used in Answer: [a] 14 inches
ENT examination? Rationale:
a. 14 inches • Focal length should be 14 inches
b. 13 inches
Reference: [ENT]_001_Basic ENT Examination
c. 12inches
d. 10 inches
52. How should the examiner be seated compared to the Answer: [c] The head of the examiner should be lower than
patient? that of the patient
a. The heads of both the patient and the Rationale:
examiner should be at the same height • Examiner may sit or stand but must not stoop
b. The head of the examiner should be higher down and must be comfortable while the patient
than that of the patient's must be seated with the head slightly higher than
c. The head of the examiner should be lower that of the examiner’s head
than that of the patient
Reference: [ENT]_001_Basic ENT Examination
d. The patient can lie down with the examiner
stooping down to examine the patient

53. What connects the cochlea and the vestibule? Answer: [d] Ductus reuniens
a. Cochlear aqueduct Rationale:
b. Aditus ad antrum • Ductus reuniens - connects vestibule and the
c. Vestibular cochlea cochlea
d. Ductus reuniens • Cochlear aqueduct - bony channel for the
communication between perilymphatic fluid and
CSF
• Aditus ad antrum - opening to the antrum
• Vestibular cochlea - anterior to vestibule and is
connected to the vestibule by the narrow ductus
reuniens
Reference: [ENT]_006_Anatomy and Physiology of the Inner Ear

ENT
RECALLS: EARS, NOSE, AND THROAT
Compiled Recalls ● 2nd Semester (Midterms)

R.01.1 (Maxillofacial Trauma Pre-test)


QUESTION ANSWER/RATIONALE
1. The most frequently fractured facial bone? ANSWER:
A. Nasal bone A. Nasal bone
B. Mandible The nasal bones form the bony nasal projection and support the upper
C. Maxilla lateral cartilages, which form the internal nasal valves. Because of their
D. Zygomatic arch prominent position in the middle of the face, the nasal bones are the most
frequently fractured bones in the human body.

Flint, Paul W., et al. (2021). Cummings Otolaryngology-Head and Neck Surgery 7th
Ed. Philadephia: Elsvier Inc Chapter 23 (Maxillofacial Trauma) P. 328
2. Identify the part of the mandible labeled as "P" ANSWER:
A. angle D. Condyle
B. body
C. coronoid
D. condyle

Asuncion, A. (2022) Maxillofacial Trauma. Powerpoint Presentation and Discussion.


March 22, 2022. UNP-CMed
3. Which of the following xray view is NOT part of a ANSWER:
Mandibular series xray? C. Submentovertex view
A. high towne's view Mandibular series X-ray
B. AP  AP
C. submentovertex view  Oblique
D. oblique  High Towne’s

Asuncion, A. (2022) Maxillofacial Trauma. Powerpoint Presentation and Discussion.


March 22, 2022. UNP-CMed
4. All of the following are vertical midface buttress EXCEPT ANSWER:
A. nasomaxillary C. Zygomaticosphenoid
B. zygomaticomaxillary Vertical Midface Buttresses
C. zygomaticosphenoid  Nasomaxillary
D. zygomaticofrontal  Zygomaticomaxillary
 Zygomaticofrontal
 Pterygomaxillary

Asuncion, A. (2022) Maxillofacial Trauma. Powerpoint Presentation and Discussion.


March 22, 2022. UNP-CMed
5. The most ideal xray view to identify anterior and posterior ANSWER:
table frontal sinus fractures B. Lateral view, skull
A. waters view
B. lateral view, skull
C. submentovertex Asuncion, A. (2022) Maxillofacial Trauma. Powerpoint Presentation and Discussion.
D. high townes March 22, 2022. UNP-CMed

1 | 8 EARS, NOSE, AND THROAT Compiled Recalls (Midterms) Editors | ENT TRANS TEAM
R.01.2 ( Maxillofacial Trauma Post-test )
QUESTION ANSWER/RATIONALE
1. All of the following are characteristics of primary bone ANSWER:
healing. EXCEPT D. Differentiation cascade takes place
A. No motion across the fracture Primary bone healing
B. increased amount of intracortical remodeling inside and in  No motion across the fracture
between fragment ends  Increased amount of intracortical remodeling inside and in between
C. doesn't proceed through the entire differentiation cascade fragment ends
D. differentiation cascade takes place  Doesn’t proceed through the entire differentiation cascade
 Achieved through rigid fixation

Asuncion, A. (2022) Maxillofacial Trauma. Powerpoint Presentation and Discussion.


March 22, 2022. UNP-CMed
2. Markowitz classification described as a single central bone ANSWER:
fragment bearing the medial canthal ligament A. Markowitz I
A. Markowitz I
B. Markowitz II
C. Markowitz III
D. None of the above

Nasoorbital ethmoid fractures have been classified as type I, type II, and
type III by Markowitz and colleagues.
 Type I fractures (A) include a solid central segment to which the
medial canthus is attached.
 Type II injuries (B) are more comminuted than type I but still leave a
central segment to which the medial canthus is attached.
 In type III injuries (C), the bone is shattered, and no solid bone is
attached to the medial canthal tendon.

Asuncion, A. (2022) Maxillofacial Trauma. Powerpoint Presentation and Discussion.


March 22, 2022. UNP-CMed
Flint, Paul W., et al. (2021). Cummings Otolaryngology-Head and Neck Surgery 7th
Ed. Philadephia: Elsvier Inc Chapter 23 (Maxillofacial Trauma) P. 335
3. What completes a Lefort fracture of any class? ANSWER:
A. nasal septum D. a and b
B. pterygoids Nasal septum and Pterygoid plates complete a Lefort fracture of any
C. maxillary dentition class (Type I, II, and III)
D. a and b
E. a and c Asuncion, A. (2022) Maxillofacial Trauma. Powerpoint Presentation and Discussion.
March 22, 2022. UNP-CMed
F. all of the above
4. Favorable or Unfavorable mandibular fracture? ANSWER:
A. Favorable A. Favorable
B. Unfavorable

Favorable (L) and Unfavorable (R) Mandibular fracture

Favorable Unfavorable
 Passes from the alveolar  Passes from the alveolar ridge
margin downward antero- infero-posteriorly downward to
inferiorly towards the border the inferior border
 Upward displacement of the  Upward displacement of the
posterior segment is prevented posterior segment is opposed
by the physical obstruction by the body
caused by the body of the  Passes from the outer cortex
mandible obliquely forward lingually
 Passes from the outer buccal  Medial displacement of the
cortex obliquely backward and posterior fragment can be
lingually opposed by the body
 Medial displacement of the
posterior segment is prevented
by the obstruction of the body
of the mandible

Asuncion, A. (2022) Maxillofacial Trauma. Powerpoint Presentation and Discussion.


March 22, 2022. UNP-CMed

2 | 8 EARS, NOSE, AND THROAT Compiled Recalls (Midterms) Editors | ENT TRANS TEAM
5. Recommended management for mandibular fractures that ANSWER:
are favorable, nondisplaced, nonmobile, unchanged pre- A. Observation and soft diet
traumatic occlusion.
A. observation and soft diet
B. closed reduction with immobilization using MMF for 4-6
weeks
C. ORIF using titanium plates and screws Asuncion, A. (2022) Maxillofacial Trauma. Powerpoint Presentation and Discussion.
D. interdental wiring March 22, 2022. UNP-CMed
6. Most common entrap muscle characterized as limitation of ANSWER:
upward gaze B. Inferior Rectus
A. Superior rectus With orbital floor fractures, the inferior rectus muscle most commonly is
B. Inferior Rectus entrapped leading to limitation in upward gaze.
C. Lateral Rectus
D. Superior Oblique Woernley, Timothy & Wright, Thomas & Lam, Duc & Jundt, Jonathon. (2017).
Oculocardiac Reflex in an Orbital Fracture Without Entrapment. Journal of Oral and
Maxillofacial Surgery. 75. 10.1016/j.joms.2017.03.014.
7. Describes the normal intercanthal distance. EXCEPT ANSWER:
A. it is half of normal interpupillary distance B. 45mm
B. 45mm
C. approximately equal to each palpebral width
D. may indicate NOE fracture

Asuncion, A. (2022) Maxillofacial Trauma. Powerpoint Presentation and Discussion.


March 22, 2022. UNP-CMed
8. The globe is protected by the following facial crumple ANSWER:
structures. EXCEPT C. Frontal sinus
A. medial orbital wall “Survival Protection” Anatomic Structures
B. orbital floor Facial Crumple Zone Area Protected
C. frontal sinus Medial orbial wall Optic nerve, Globe
D. maxillary sinus Orbital floor Globe
Maxillary sinus Globe, Middle Cranial fossa
Ethmoid sinus Globe, Optic nerve, Anterior
Cranial Fossa, Middle cranial
fossa
Frontal sinus Anterior cranial fossa
Sphenoid sinus Carotid arteries, Cavernous
sinuses
Face as a whole Cranial cavity
Condylar necks of mandible Middle cranial fossa

Asuncion, A. (2022) Maxillofacial Trauma. Powerpoint Presentation and Discussion.


March 22, 2022. UNP-CMed
9. Physical examination done to evaluate entrapment of ANSWER:
extraocular muscle B. Force duction test
A. drawer's test Forced Duction Test: used to evaluate entrapment of the EOM
B. force duction test
C. bimanual test
Asuncion, A. (2022) Maxillofacial Trauma. Powerpoint Presentation and Discussion.
D. battle's test March 22, 2022. UNP-CMed
10. Timing of closed reduction in cases of nasal bone fracture. ANSWER:
EXCEPT C. Delay may be possible up to a month
A. ASAP or 1 to 2 hrs after injury Closed Reduction (Timing)
B. before the 10th day of injury  ASAP or 1-2h after injury
C. delay may be possible up to a month  After 2 to 3 days for swelling to subside
D. after 2 to 3 days  Before 10th day, prior to fibrous-connective tissue development (10-14
days)
 Delay may be possible up to 21 days

Asuncion, A. (2022) Maxillofacial Trauma. Powerpoint Presentation and Discussion.


March 22, 2022. UNP-CMed

3 | 8 EARS, NOSE, AND THROAT Compiled Recalls (Midterms) Editors | ENT TRANS TEAM
R.02 (Anatomy and Physiology of the External Ear, Anatomy and Physiology, and Diseases of the Inner Ear)
QUESTION ANSWER/RATIONALE
1. What bone forms the roof of the glenoid fossa? ANSWER:
A. Petrous C. Zygomatic process
B. Mastoid Zygomatic process forms the bony roof of the glenoid fossa.
C. Zygomatic process
D. Squamosa [ENT] T.02 - Anatomy and Physiology of the External Ear p.1
2. The 6th Hillock of His will develop into this structure ANSWER:
A. Helix D. Lobule
B. Anti-helix  First branchial arch: 1st – 3rd hillock
C. Tragus o 1st hillock – forms the tragus
D. Lobule o 2nd hillock – forms the crus of helix
o 3rd hillock – forms the helix
 Second branchial arch: 4th – 6th hillock
o 4th hillock – forms the anti-helix
o 5th hillock – forms the crus of anti-helix
o 6th hillock – forms the anti-tragus (in some books, 6th is the
lobule)

[ENT] T.02 - Anatomy and Physiology of the External Ear p.3


3. At what AOG does the external ear achieve its adult shape? ANSWER:
A. 12th wk D. 20th wk
B. 14th wk  7th week – cartilage formation
C. 16th wk  12th week – auricle formed
D. 20th wk  20th week: adult shape
 adult size by 9-year-old
[ENT] T.02 - Anatomy and Physiology of the External Ear p.3
4. What are the muscle/s attached to the zygomatic process? ANSWER:
A. Masseter D. A and C
B. Lateral Pterygoids Two muscles of mastication are attached to the zygomatic process:
C. Temporalis Temporalis muscle and the Masseter muscle
D. A and C
[ENT] T.02 - Anatomy and Physiology of the External Ear p.1
5. Major blood supply to the EAC ANSWER:
A. External carotid artery A. External carotid artery
B. Superficial temporal artery
Major blood supply to auricle and EAC: External Carotid Artery
C. Internal carotid artery
D. Anterior auricular artery [ENT] T.02 - Anatomy and Physiology of the External Ear p.4
6. A possible route of spread for tumors in the EAC into the ANSWER:
parotid B. Fissure of Santorini
A. Foramen of Huschke
Two patterns of Cancer spread in the EAC
B. Fissure of Santorini
 Fissures of Santorini: vestigial lymph channels that drain into
C. Stylomastoid foramen superficial PG (parotid gland); Provide avenues of spread to the
D. Tympanomastoid suture line superficial lobe of the gland.
 Foramen of Huschke: Serve as a means for extension of malignant
tumors from the EAC to the deep lobe of the parotid gland.

[ENT] T.02 - Anatomy and Physiology of the External Ear p.4


7. What is the length of the EAC? ANSWER:
A. 1 cm C. 2.5 cm
B. 1.5 cm
EAC: ~2.5 cm long
C. 2.5 cm
D. 3 cm [ENT] T.02 - Anatomy and Physiology of the External Ear p.4
8. Sensory innervation of the floor of EAC ANSWER:
A. CN X A. CN X
B. CN VII
External Auditory Canal Innervation
C. C3
 Anterior – C3
D. CN V
 Posterior – CN V
 Superior – CN VII
 Floor – CN X

[ENT] T.02 - Anatomy and Physiology of the External Ear p.3


9. Features of Eagle Syndrome except ANSWER:
A. Elongated styloid process B. Enlarged stylomastoid foramen
B. Enlarged stylomastoid foramen
Eagle Syndrome: Elongation or angulation of the styloid process can
C. Odynophagia
produce the triad of odynophagia, dysphagia, and foreign body sensation
D. Calcified stylohyoid ligament in the throat

[ENT] T.02 - Anatomy and Physiology of the External Ear p.2

4 | 8 EARS, NOSE, AND THROAT Compiled Recalls (Midterms) Editors | ENT TRANS TEAM
10. Muscle attached to the mastoid ANSWER:
A. Trapezius C. Posterior belly of digastric
B. Anterior belly of digastric
Muscles attached to the Mastoid: SCM and Posterior belly digastric
C. Posterior belly of digastric
D. Platysma [ENT] T.02 - Anatomy and Physiology of the External Ear p.2
11. First relay station for all ascending auditory information ANSWER:
A. Cochlear nerve 11-12. B. Cochlear nucleus
B. Cochlear nucleus
Cochlear Nucleus:
C. Superior Olivary Complex
 Critical first relay station for all ascending auditory information
D. Lateral lemniscus
 Point of entry of peripheral auditory information to central auditory
12. Point of entry of peripheral auditory information to central system
auditory system  Pontomedullary junction of the dorsolateral brainstem
A. Cochlear nerve
B. Cochlear nucleus
C. Superior olivary complex
D. Lateral lemniscus [ENT] T.05 - Anatomy and Physiology of the Inner Ear p. 4
13. Portal for all ascending auditory innervation to ANSWER:
telencephalon B. Medial geniculate body
A. Lateral lemniscus Medial Geniculate Body:
B. Medial geniculate body  Portal for all ascending auditory innervation to telencephalon
C. Inferior colliculus  Ventral, dorsal, and medial division
D. Auditory cortex
[ENT] T.05 - Anatomy and Physiology of the Inner Ear p. 4
14. These are actin filaments deflecting with mechanical ANSWER:
disturbance B. Stereocilia
A. Cuticular plate Stereocilia: actin filaments deflecting with mechanical disturbance;
B. Stereocilia rooted in the cuticular plate of each hair cell and projecting through the
C. Synaptic bars reticular lamina (into endolymph)
D. Tip links [ENT] T.05 - Anatomy and Physiology of the Inner Ear p. 3
15. It is a compliant gelatinous structure over the inner and ANSWER:
outer hair cells A. Tectorial membrane
A. Tectorial membrane Tectorial membrane: Compliant gelatinous (collagen II – over the inner
B. Spiral limbus and outer cells).
C. Tunnel of Corti
[ENT] T.05 - Anatomy and Physiology of the Inner Ear p. 2
D. Reticular lamina
16. Passageway for nerve fibers ANSWER:
A. Tunnel of Corti D. Modiolus
B. Rosenthal Canal Modiolus: central core of the cochlea which is a highly porous bone -
C. Cochlear Aqueduct passageway for nerve fibers from the internal auditory meatus to the hair
D. Modiolus cell synapses

[ENT] T.05 - Anatomy and Physiology of the Inner Ear p. 1


17. Which statement is correct? ANSWER:
A. Stereocilia are deflected in the direction of the tallest row A. Stereocilia are deflected in the direction of the tallest row
B. This causes the tip links to relax
C. Relaxation of tip links causes channels to open and the
creation of large infux of cationic current
D. All statements are correct

Actin Filaments and Stereocilia


Movements are dependent on whether tip links are relaxed or stretched.
 Figure on left shows stereocilia are deflected away from the tallest
row that causes relaxation of tip links which also then closes the ion
channel opening creating cellular hyperpolarization: negative
displacement = relaxation → hyperpolarization
 Figure on right shows tip links are stretched that opens ion channel
gradient causes influx of cat ions resulting to hair cell depolarization:
positive displacement = stretching → depolarization

[ENT] T.05 - Anatomy and Physiology of the Inner Ear p. 3

5 | 8 EARS, NOSE, AND THROAT Compiled Recalls (Midterms) Editors | ENT TRANS TEAM
18. Which statement is correct? ANSWER:
A. The fluid systems in the inner ear is responsible for cellular 18. A. The fluid systems in the inner ear is responsible for cellular
depolarization creating a synaptic activation depolarization creating a synaptic activation
19. D. All statements are correct
B. The perilymph is located within the membranous labyrinths
C. High K+, low Na+ is characteristic of perilymph FLUID SYSTEMS IN THE INNER EAR
D. The endolymph is located between osseous and  Perilymphatic system and the Endolymphatic system
membranous labyrinths  Crucial because it will create an environment that will result to
Mechanical displacement of BM traveling wave
 Cellular depolarization resulting to a synaptic activity
19. Which statement is true?
A. Endolymph comprises the scala media PERILYMPH ENDOLYMPH
B. It produces a large electrochemical gradient relative to the Between osseus and
perilymph Within membranous labyrinths;
membranous labyrinths (with
C. This is accomplished by the stria vascularis maintained by cells of stria vascularis
scala tympani and vestibuli)
D. All statements are correct High K+, low Na+ = same with
Intracellular envt (Potassium
High Na+, low K+ = similar
maintains the large positive electric
to the Blood, CSF
gradient within the endolymph
compared to perilymph)
Scala vestibuli, Scala
tympani and internal spaces Scala media
of the organ of Corti
Endolymphatic system produces
Communicates with Endocochlear potential – large
cerebrospinal fluid via cochlear electrochemical gradient → +60 to
aqueduct +100 mV relative to the perilymph.

Accomplished by the stria


vascularis → contains multiple
Likely conduit for bacterial active ion channels and maintains the
meningitis into the inner ear chemical composition of the
endolymph and its positive electrical
potential
Disorders include endolymphatic
hydrops (Ménière disease) or wide
vestibular aqueduct

[ENT] T.05 - Anatomy and Physiology of the Inner Ear p. 2


20. Protects the inner ear from acoustic trauma, discriminate ANSWER:
transient sounds from background noise A. Olivocochlear reflex
A. Olivocochlear reflex
EFFERENT AUDITORY SYSTEM
B. Vestibulospinal reflex
 Middle ear muscle reflex – protective by the contraction of stapedius
C. Vestibulocular reflex and tensor tympani muscles
D. Vestibulocollic reflex  Olivocochlear reflex – protect from acoustic trauma, discriminate
transient sounds from background noise

[ENT] T.05 - Anatomy and Physiology of the Inner Ear p. 4


21. J.S. 19/M was rushed to the ER with bloody otorrhea, AD ANSWER:
and multiple facial injuries secondary to motor accident. A. Battle sign
Patient is conscious, coherent but was dizzy and
Battle's sign: Postauricular ecchymosis is a sign of skull base fracture.
nauseated. Vital signs normal. Otoscopy showed active
bleeding at EAC, TM cannot be visualized and on
inspection there is a violaceous discoloration at the right
postauricular area. The discoloration is also known as
_____ which is suggestive of temporal bone fracture
A. Battle sign
B. Bird's beak
C. Herman sign
D. None of the above
[SRG] T.06.1 Traumatic Brain Injury p. 4
22. What will you request to confirm your diagnosis? ANSWER:
A. Audiometry B. Temporal bone CT Scan
B. Temporal bone CT Scan
A Plain CT-Scan is the initial “Imaging of choice” for acute traumatic brain
C. Skull Xray
injury.
D. Mastoid series
[SRG] T.06.1 Traumatic Brain Injury p. 6
23. Treatment options for Meniere's Disease ANSWER:
A. Diuretics A. Diuretics
B. SSRI
Treatment for Meniere’s Disease:
C. TCA
 Dietary modification (less salt)
D. B-blockers
 Diuretics
 Vasodilators
[ENT] T.06 – Diseases of the Inner Ear p. 6

6 | 8 EARS, NOSE, AND THROAT Compiled Recalls (Midterms) Editors | ENT TRANS TEAM
24. Nystagmus observed with a patient was <1min and was ANSWER:
upbeat and torsional. What is your impression? C. BPPV
A. Labyrinthitis
Benign Paroxysmal Positional Vertigo (BPPV) PE findings
B. Neuritis
 Combined upbeating and torsional
C. BPPV
 Latency of onset of nystagmus: seconds
D. Vestibular migraine  Duration of Nystagmus: <1 min
 Vertiginous symptoms such as nausea and vomiting
 Nystagmus is fatigable if the nystagmus disappears with repeated
testing
 Symptoms recur with the nystagmus in the opposite direction upon
return of the head to the upright position
 LC-BPPV: horizontal nystagmus
 SC-BPPV: down torsional

[ENT] T.06 – Diseases of the Inner Ear p. 6


25. Diagnostic criteria for Certain Meniere Disease ANSWER:
A. Audiometry D. Autopsy
B. Tympanometry
DIAGNOSIS OF MENIERE DISEASE
C. Positive Epley
 Possible Meniere Disease
D. Autopsy o Episodic vertigo without hearing loss or
o Sensorineural hearing loss, fluctuating or fixed, with disequilibrium
but with definite episodes
o Other causes excluded
 Probable Meniere Disease
o One definitive episode of vertigo
o Hearing loss documented by audiogram at least once
o Tinnitus or aural fullness in the suspected ear
o Other causes excluded
 Definite Meniere Disease
o Two or more definitive spontaneous episodes of vertigo lasting at
least 20 minutes
o Audiometrically documented hearing loss on at least one occasion
o Tinnitus or aural fullness in the suspected ear
o Other causes excluded
 Certain Meniere Disease
o Definite Meniere disease plus histopathologic confirmation

[ENT] T.06 – Diseases of the Inner Ear p. 7


26. Features of PC-BPPV except ANSWER:
A. Horizontal and upbeating nystagmus A. Horizontal and upbeating nystagmus
B. Fatigable and reversible
Features of PC-BPPV:
C. Canalithiasis
 Canalithiasis mechanism. This explains the latency of nystagmus as a
D. Nystagmus <1 min result of the time needed for motion of the material within the posterior
canal to be initiated by gravity.
 Nystagmus duration of less than one minute
 Vertical and torsional nystagmus
 Reversal of nystagmus when patient returns to the sitting upright
position is due to retrograde movement of material in the lumen of the
posterior canal back toward the ampulla with resulting ampullopetal
deflection of the cupula.
 Fatigability of nystagmus evoked by repeated Dix-Hallpike positional
testing is explained by dispersion of material within the canal.

[ENT] T.06 – Diseases of the Inner Ear p. 6


27. Most common cause of nongenetic hearing loss ANSWER:
A. Mumps D. CMV
B. Connexin 26
CMV: Most common cause of nongenetic hearing loss
C. ACTG1
D. CMV [ENT] T.06 – Diseases of the Inner Ear p. 7
28. What is the allowable duration of unprotected exposure to ANSWER:
a jack hammer noise according to OSHA? C. 15-30 mins
A. 6-8 hrs
Occupational Safety and Health Administration Standard for Hearing
B. 2-5 hrs
Duration Hearing Level
C. 15-30 mins
8 hours 90 dB
D. 30-45 mins 4 hours 95 dB
2 hours 100 dB
1 hour 105 dB
30 mins 110 dB
15 mins 115 dB
*Jack hammer approx. 100-130 dB

[ENT] T.06 – Diseases of the Inner Ear p. 4


*various sources

7 | 8 EARS, NOSE, AND THROAT Compiled Recalls (Midterms) Editors | ENT TRANS TEAM
29. Most common neoplastic cause of SNHL ANSWER:
A. Meningioma D. Vestibular Schwannoma
B. Glioblastoma
Vestibular Schwannoma:
C. Astrocytoma
 Most common neoplastic cause of SNHL
D. Vestibular Schwannoma
 6% of all intracranial neoplasms
 Originates from vestibular nerves within CPA or IAC

[ENT] T.06 – Diseases of the Inner Ear p. 5


30. All these drugs are cochleotoxic except: ANSWER:
A. Dihydrostreptomycin B. Streptomycin
B. Streptomycin Ototoxicity:
C. Kanamycin  Cochleotoxic
o Kanamycin, Tobramycin, Amikacin, Neomycin,
D. Tobramycin Dihydrostreptomycin
o Neomycin toxicity – rapid and profound
o Neomycin Ear drops are used in some ear infection but in very
minimal dose to avoid hair cells damage
 Vestibulotoxic
o Streptomycin and gentamicin

[ENT] T.06 – Diseases of the Inner Ear p. 3

8 | 8 EARS, NOSE, AND THROAT Compiled Recalls (Midterms) Editors | ENT TRANS TEAM
RECALLS: EARS, NOSE, AND THROAT
Compiled Recalls ● 2nd Semester (Finals)

R.03 (Cleft Lip and Palate)


Pre-test
QUESTION ANSWER/RATIONALE
1. What is your diagnosis? ANSWER/s:

 By Veau Classification: Cleft lip and Palate Class 3


 By Thallwitz Classification: Cleft Lip and Palate, Unilateral,
Complete, Left, L0A0H0S3H3A3L3

[ENT] T.11 - Cleft Lip and Palate p.4

Post-test
QUESTION ANSWER/RATIONALE
1. Cleft lip and palate deformities are the most common ANSWER:
congenital defect of the head. The most common etiology is: D. Multifactorial
A. Genetic Most common etiology – multifactorial (prevalence varies among ethnic
B. Teratogens groups and within families)
C. Prenatal vitamin deficiency
D. Multifactorial [ENT] T.11 - Cleft Lip and Palate p.1
2. Palatogenesis begins at the end of: ANSWER:
A. 4th Week 2. B. 5th Week
B. 5th Week 3. C. 12th week
C. 6th Week
Palatogenesis
D. 7th Week  Palatogenesis begins at end of 5th week
3. The age of development when there is complete fusion of  Complete fusion at 12 weeks.
the palate.
A. 4th week
B. 8th week
C. 12th week
D. 16th week [ENT] T.11 - Cleft Lip and Palate p.1
4. A male newborn baby is referred to you due to lip ANSWER:
deformity. What muscle is affected in complete cleft lip? D. Orbicularis oris
A. Orbicularis oculi Orbicularis oris
B. Levator labii superioris  Main muscle affected in cleft lip
 Forms a complete sphincter around the oral cavity and provides the
C. Risorius
substrate for proper form and function of the lips and mouth.
D. Orbicularis oris
[ENT] T.11 - Cleft Lip and Palate p. 2
5. What gender is cleft lip more common? ANSWER:
A. Male 5. A. Male
B. Female 6. D. Fusion of palatine shelves is 1 week later in girls than boys
C. Both are equally affected CLEFT LIP CLEFT PALATE
6. Why is cleft palate more common in females?
 1 in 1000 live births in the US  1 in 2000 live births in the US
A. Fusion of palatine shelves is 1 month later in boys than girls  High incidence among  GIRLS > BOYS
B. Fusion of palatine shelves is 1 week later in boys than girls  Native Americans (3.6 in 1000)  Fusion of palatine shelves
C. Fusion of palatine shelves is 1 month later in girls than boys  Asians (2.1 in 1000) 1 week later in girls vs
D. Fusion of palatine shelves is 1 week later in girls than boys  Whites (1 in 1000) boys – higher incidence of
 Blacks (0.41 in 1000) cleft palate in girls
 BOYS > GIRLS
[ENT] T.11 - Cleft Lip and Palate p. 3
7. What prenatal diagnostic consideration in cleft lip and ANSWER:
palate is important especially in the 3rd trimester? C. 3D Ultrasonography
A. Amniotic fluid exam Diagnostic considerations in cleft lip and palate
B. Genetic analysis  Ultrasonography and 3D ultrasonography enables utero diagnosis of
C. 3D Ultrasonography clefts especially in 3rd trimester.
D. All of the above  Part of the prenatal check up to look for deformities.
[ENT] T.11 - Cleft Lip and Palate p. 3
8. The incisive foramen divides the palate into primary and ANSWER:
secondary palate. The following structures are found D. None of the above
posterior to the incisive foramen. PRIMARY PALATE SECONDARY PALATE
A. Premaxilla  Anterior to the incisive foramen  Develops after completion of
B. Lip  Consists of: the primary palate
C. Columella  Premaxilla  Nasal tip  Extends from incisive
D. None of the above  Lip  Columella foramen anteriorly to uvula
posteriorly
 Posterior to incisive foramen
[ENT] T.11 - Cleft Lip and Palate p. 3

1 | 22 EARS, NOSE, AND THROAT Compiled Recalls (Finals) Editors | ENT TRANS TEAM
9. The aim of this first surgery for the patient with cleft lip is ANSWER:
to restore the entire cleft defect to as near a normal anatomy A. Cheiloplasty
as possible.  Cheiloplasty
A. Cheiloplasty o Done as early as three months
o PRIMARY REPAIR
B. Alveoloplasty o Rule of Ten: must be considered to avoid complications
C. Primary Rhinoplasty  The child weighs 10 pounds
D. Palatoplasty  The child has a hemoglobin of at least 10 GRAMS
 The child has a WBC count of no higher than 10,000
 The child is at least 10 WEEKS OF AGE
 Alveoloplasty: can be done with primary cheiloplasty or until the ideal
age for bone grafting is reached
 Primary rhinoplasty: can be done with primary cheiloplasty or until as
early as 14 years old for females and 16 years old for males which is the
ideal age for definitive rhinoplasty.
 Palatoplasty: can be done at 12 to 18 months, or before the child is
able to speak so his/her that speech will be normal
[ENT] T.11 - Cleft Lip and Palate p. 4
10. This is formed when the paired maxillary prominences ANSWER:
grow medially toward the paired medial nasal prominences. B. Upper lip
A. Nasal tip
Paired maxillary prominences grow medially toward the paired medial
B. Upper lip
nasal prominences — UPPER LIP
C. Columella
D. Nasal ala [ENT] T.11 - Cleft Lip and Palate p. 1

R.04 (Anatomy, Physiology, and Diseases of the Neck)


QUESTION ANSWER/RATIONALE
1. `This fascial plane is the main plane of resistance to deep ANSWER:
neck spread of cellulitis, allows mobility of skin over deep A. Superficial Cervical Fascia
neck structures and easily separated when raising neck flaps Significance of Superficial Cervical Fascia
from deep cervical fascia in the subplatysmal potential space  Main plane of resistance to deep neck spread of cellulitis
(adipose, sensory nerves, blood vessels)  Allows mobility of skin over deep neck structures
A. Superficial cervical fascia  Easily separated when raising neck flaps from deep cervical fascia in
B. Investing fascia the subplatysmal potential space (adipose, sensory nerves, blood
C. Visceral fascia vessels)
D. Prevertebral fascia [ENT] T.12 - Anatomy, Physiology, and Diseases of the Neck p.2

2. A 63 y/o male, smoker and alcohol beverage drinker came ANSWER:


into your clinic with an incision biopsy result of his middle B. Do panendoscopic physical examination
jugulodigastric cervical lymph node, right. The result read: Occult Regional Metastatic Squamous Cell Carcinoma
Squamous Cell Carcinoma. As an ENT, what would be your  Diagnosed histologically by FNA biopsy/excisional biopsy of a
next step? cervical lymph node
A. Schedule for neck dissection  Most common sites that harbor occult primaries commonly missed by
B. Do panendoscopic physical examination non-ENTs (ie. Nasopharynx, base of tongue, tonsil, and piriform sinus)
C. Refer for radiotherapy must remain in the practitioner’s mind during physical examination.
D. Request abdominal ultrasound and chest X-ray [ENT] T.12 - Anatomy, Physiology, and Diseases of the Neck p. 9
3. What divides the submandibular space into a sublingual ANSWER:
(infections anterior to second molar) and submaxillary A. Mylohyoid line
(infections of second and third molars) compartments? Involvement of the submandibular and sublingual spaces are usually
A. Mylohyoid line secondary to infection in dentition. If the infection exceeds the mylohyoid
B. Geniohyoid line line which divides the sublingual and submandibular spaces, a
C. Digastric line complication called Ludwig’s angina can occur which is definitely an
emergency.
D. Level I line [ENT] T.12 - Anatomy, Physiology, and Diseases of the Neck p. 3
4. A 10 y/o male with an anterior neck mass located inferior to ANSWER:
the hyoid bone. His parents noticed the mass 5 years ago but C. Thyroglossal Duct Cyst
no consult was done until the child has URTI and the mass
Thyroglossal Duct Cyst
has been noticeable to be enlarging and tender. Antibiotics  Most common congenital neck lesion in children
were given at a local RHU. The tenderness subsided however  Represents 70% of congenital pediatric neck masses
the mass still persisted hence sought consult at your clinic.  Located inferior to the hyoid bone (most common) but can also be
Upon examination, the mass is soft and cystic and elevates found just superior to the hyoid bone
upon tongue protrusion. Your working diagnosis would be?  On examination, TGDCs are usually soft and cystic to palpation and
A. Branchial cleft cyst will elevate in the neck with tongue protrusion
B. Ranula  Usually patients come for consultation, when there is already infection.
C. Thyroglossal duct cyst
[ENT] T.12 - Anatomy, Physiology, and Diseases of the Neck p. 7
D. Lymphangioma
5. Which of the following is TRUE regarding Cervical Lymph ANSWER:
Node groups? Except D. Spinal accessory nerve divides Level Va and Vb
A. Cricoid cartilate divides Level III and IV
Spinal accessory nerve divides Levels IIa and IIb
B. Spinal accessory nerve divides IIa and IIb
C. Cricoid cartilage divides Level Va and Vb
D. Spinal accessory nerve divides Level Va and Vb [ENT] T.12 - Anatomy, Physiology, and Diseases of the Neck p. 4

2 | 22 EARS, NOSE, AND THROAT Compiled Recalls (Finals) Editors | ENT TRANS TEAM
6. Complication of deep neck infection that is rare ANSWER:
thrombophlebitis of the internal jugular vein caused by B. Lemierre’s syndrome
anaerobic, gram-negative bacillus Fusobacterium Lemierre syndrome
necrophorum.  Rare thrombophlebitis of the internal jugular vein - anaerobic, gram-
negative bacillus Fusobacterium necrophorum.
A. Ludwig’s angina
 Follows a period of pharyngitis before progressing to fever, lethargy,
B. Lemierre’s syndrome lateral neck tenderness and edema, occasional trismus, and septic
C. Necrotizing fasciitis emboli
D. Cavernous sinus thrombosis [ENT] T.12 - Anatomy, Physiology, and Diseases of the Neck p. 6
7. Which of the following does NOT define the Carotid ANSWER:
triangle? A. Superior: Hyoid bone
A. Superior: Hyoid bone Carotid (Superior Carotid/Vascular) Triangle
B. Anterior: Superior belly of omohyoid  Superior: Posterior belly of the digastric
C. Posterior: Sternocleidomastoid  Anterior: Superior belly of omohyoid
D. Superior: Posterior belly of the digastric o The omohyoid runs from the hyoid bone to the clavicle
E. NOTA  Posterior: Sternocleidomastoid
 Floor: Parts of the thyrohyoid, hyoglossus, medial and inferior
pharyngeal constrictor muscles
[ENT] T.12 - Anatomy, Physiology, and Diseases of the Neck p. 1
8. A 26 y/o male came in with an erythematous, tender, ANSWER:
submandibular neck mass with associated febrile episodes. G. AOTA
Upon further assessment, you noted several dental caries and Deep Neck Infection
trismus. You diagnosed the case as: Deep Neep Infection.  Dental infections – most common cause of DNI in adults
 Medical management:
Your management will include?
o Airway management:
A.Tooth extraction of decayed teeth after resolution of infection  Urgent airway intervention or tracheotomy: stridor and dyspnea,
B. Prescribe Clindamycin or Ampicillin-Sulbactam airway obstruction of more than 50%
C. Fluid resuscitation using 1-2L of isotonic solution  Fiber optic laryngoscopy done immediately
D. Contact Anesthesiologist for a possible awake fiberoptic  needing anesthesiologist because it is “awake intubation”
intubation of disease progresses procedure
E. A,B,C o Fluid resuscitation
F. B,C,D  Most patients benefit from timely infusion of 1 to 2 L of isotonic IV
fluids
G. AOTA
o Antibiotic therapy with Broad-spectrum coverage as the main
treatment

[ENT] T.12 - Anatomy, Physiology, and Diseases of the Neck p. 5


9. TRUE of carotid paraganglioma, Except ANSWER:
A. Most common class of benign vascular neoplasms of the C. Fontaine sign characterized as a bowing and displacing of the
neck and arise from extraadrenal paraganglionic cells internal and external carotid arteries.
derived from the neural crest.
Carotid Paraganglioma
B. Usually presents as a lateral pulsatile soft and elastic to firm  Most common class of benign vascular neoplasms of the neck and arise
non-tender cervical mass, which is mobile laterally but less from extraadrenal paraganglionic cells derived from the neural crest
mobile in the craniocaudal direction because of its  Pathognomonic sign:
adherence to the carotid arteries. o Usually presents as a lateral pulsatile soft and elastic to firm non-
C. Fontaine sign characterized as a bowing and displacing of tender cervical mass, which is mobile laterally but less mobile in the
the internal and external carotid arteries. craniocaudal direction because of its adherence to the carotid
D. Expectant treatment in older, debilitated patients in absence arteries. (Fontaine sign)
of symptoms due to risk of bleeding during operations; may  Pathognomonic in imaging (CT angiogram):
include radiation o Lyre sign characterized as a bowing and displacing of the
internal and external carotid arteries
 Expectant treatment in older, debilitated patients in absence of
symptoms due to risk of bleeding during operations; may include
radiation.

[ENT] T.12 - Anatomy, Physiology, and Diseases of the Neck p. 8


10. Bounderies of the Parapharyngeal Space, Except ANSWER:
A. Superior: Base of skull (Middle cranial fossa) C. Posterior: Pterygomandibular raphe
B. Inferior: Hyoid bone
Parapharyngeal Space
C. Posterior: Pterygomandibular raphe
 Superior: Base of the skull (Middle cranial fossa)
D. Medial: Pharyngobasilar fascia (superiorly), superior
 Inferior: Hyoid bone
constrictor  Anterior: Pterygomandibular raphe
E. Lateral: Deep lobe of parotid gland, mandible, and medial  Posterior: Prevertebral fascia
pterygoid  Medial: Pharyngobasilar fascia (superiorly), superior constrictor
 Lateral: Deep lobe of parotid gland, mandible, and medial pterygoid

[ENT] T.12 - Anatomy, Physiology, and Diseases of the Neck p. 3

3 | 22 EARS, NOSE, AND THROAT Compiled Recalls (Finals) Editors | ENT TRANS TEAM
R.05 (Anatomy, Physiology, and Diseases of the Larynx and Trachea)
Pre-test
QUESTION ANSWER/RATIONALE
1. Give 3 functions of the Larynx ANSWER/s:

 Respiration / Breathing
 Protection of the respiratory tract against foreign bodies
 Speech (Phonation)

[ENT] T.13 - Anatomy and Physiology of the Larynx and Trachea p.4
Post-test
QUESTION ANSWER/RATIONALE
1. This muscle in the ONLY ABDUCTOR of the vocal folds ANSWER:
A. Interarytenoid muscle C. Posterior cricoarytenoid muscle
B. Lateral cricoarytenoid muscle  Posterior Cricoarytenoid (PCA): Only vocal fold abductor
C. Posterior cricoarytenoid muscle  All intrinsic muscles of the larynx adducts the vocal chords except for
D. Cricothyroid muscle the Posterior Cricoarytenoid
[ENT] T.13 - Anatomy and Physiology of the Larynx and Trachea p.2
2. All intrinsic muscles of the larynx are innervated by this ANSWER:
nerve except for the cricothyroid muscle C. Recurrent laryngeal nerve
A. External branch of Superior laryngeal nerve
All Intrinsic muscles of the larynx are innervated by the Recurrent
B. Internal branch of Superior laryngeal nerve
Laryngeal Nerve except for the Cricothyroid
C. Recurrent laryngeal nerve
D. None of the above [ENT] T.13 - Anatomy and Physiology of the Larynx and Trachea p.2
3. A 3 y/o boy presented with stridor for 1 day, barking ANSWER:
cough, fever and drooling. What is your diagnosis? B. Acute laryngotracheitis
A. Acute epiglottitis
Acute laryngotracheitis
B. Acute laryngotracheitis
 Viral infection, affects children <3 y/o
C. Gastroesophageal reflux disease
 Febrile URTI, followed by classic barky or croupy cough (nonproductive
D. Acute pharyngitis and more common at night)
 Neck X-ray: Classic steeple sign (inverted V)

[ENT] T.14 – Diseases of the Larynx and Trachea p. 1, 5


4. A 33 y/o female teacher came for consult due to ANSWER:
hoarseness, mild cough and sometimes disappearance of A. Vocal cord nodule
voice. No history of URTI noted. What is your diagnosis?
Vocal cord nodule
A. Vocal cord nodule
 Almost always vocal “over-doers” patients
B. Vocal cord polyp
 Voice is chronically raspy and there may be frequent bouts of laryngitis
C. Vocal cord cyst
D. All of the above [ENT] T.14 – Diseases of the Larynx and Trachea p. 2, 5
5. A 55 y/o male presented with hoarseness, right-sided ANSWER:
otalgia, and mild dysphagia. He is a smoker and alcohol D. Malignant neoplasm of the Larynx
drinker. What is your initial impression?
Squamous Cell Carcinoma
A. Reflux esophagitis
 Most common laryngeal Ca (>90%)
B. Vocal cord polyp
 Male:female = 6:1
C. Laryngeal papillomatosis  Etiology: Tobacco (most common), Alcohol, XRT, asbestos, wood
D. Malignant neoplasm of the larynx dust, mustard gas, GERD, HPV
 Clinical Manifestation: hoarseness, throat pain, cough, hemoptysis,
referred otalgia, dysphagia

[ENT] T.14 – Diseases of the Larynx and Trachea p. 4, 5


6. Which among these laryngeal skeletons are paired? ANSWER:
A. Arytenoid cartilage A. Arytenoid cartilage
B. Cricoid cartilage Laryngeal Skeleton
C. Thyroid cartilage  Unpaired or single cartilages:  Paired cartilages:
o Epiglottis o Arytenoids
D. Epiglottis o Thyroid Cartilage o Cuneiform Cartilage
o Cricoid Cartilage o Corniculate Cartilage
o Hyoid
[ENT] T.13 - Anatomy and Physiology of the Larynx and Trachea p.1
7. These synovial joints serve as the most functionally ANSWER:
important joints in the larynx. A. Cricoarytenoid joints
A. Cricoarytenoid joints
Cricoarytenoid Joints
B. Cricothyroid joint
 The most functionally important joint
C. Epiglottoarytenoid joint
 Formed by the articulation of base of the arytenoid cartilage to the
D. Arytenocorniculate joints superior border of the cricoid cartilage
 Synovial rocking motion (anteromedially for vocal fold adduction and
posterolaterally for vocal fold abduction)

[ENT] T.13 - Anatomy and Physiology of the Larynx and Trachea p.1

4 | 22 EARS, NOSE, AND THROAT Compiled Recalls (Finals) Editors | ENT TRANS TEAM
8. This procedure is used to view the larynx in speech and ANSWER:
singing C. Flexible endoscopy
A. Mirror laryngoscopy Flexible Laryngoscopy
B. Rigid endoscopy  Advantage: View larynx in speech and singing because this is usually
C. Flexible endoscopy inserted in the nose, Gottic gap more accurate
D. Direct laryngoscopy  Assess nasal cavity and velopharynx; Better for movement problem
than structure or mucosal health
 Disadvantage: Inferior light transport and magnification; distortion of
periphery of image
[ENT] T.13 - Anatomy and Physiology of the Larynx and Trachea p.5
9. True of the trachea ANSWER:
A. From the cricoid down to the sternal angle or below the 9. D. All of the above
carina where it divides into left and right main bronchus. 10. B. Trachealis muscle
Trachea
B. Mobile cartilagenous and membranous tube
 From the cricoid (C6) down to the sternal angle / below the carina where
C. Approximately 11.25cm long (adults) and 2.5cm in it divides into the left and right main bronchus
diameter  Mobile cartilagenous and membranous tube
D. All of the above  Approximately 11.25 cm long (adult) and 2.5 cm in diameter
10. Posteriorly, the trachea is attached to the esophagus via  The fibroelastic cartilage is kept patent by a U – shaped rings of hyaline
the: cartilage
A. Hyaline cartilage o 14 to 20 hyaline cartilage (tracheal rings) – incomplete posteriorly
B. Trachealis muscle  Posteriorly, it is attached to the esophagus via the Trachealis muscle
C. Circular esophageal muscle  Lined by pseudostratified columnar ciliated epithelium resting on an
D. All of the above elastic lamina propria

[ENT] T.13 - Anatomy and Physiology of the Larynx and Trachea p.6

R.06 (Basic ENT Examination)


QUESTION ANSWER/RATIONALE
1. Prior to making a basic ENT examination what should be ANSWER:
done first? B. Make a thorough general history and physical examination
A. Have all the necessary equipment ready.
B. Make a thorough general history and physical examination A general history and physical examination should be included of every
head and neck evaluation
C. Apply topical anesthesia
D. Acquire an informed consent [ENT] T.01 - Basic ENT Examination p.1.
2. How is the head mirror placed on the examiner’s head? ANSWER:
A. Over the dominant eye of the examiner A. Over the dominant eye of the examiner
B. Wherever the light source is positioned Head mirror
C. Over the forehead to have binocular sight  3 1⁄2 inch mirror with a 1⁄2 inch hole at the center
 Focal length of 14 inches
D. Positioning is not important
 Placed over dominant eye
[ENT] T.01 - Basic ENT Examination p.1.
3. What is the patient’s head position in relation to that of the ANSWER:
examiner? B. Patient’s head should be higher than that of the examiner
A. Patient’s head should be at same level as that of the
Patient
examiner
 Seated with the head slightly higher than that of the examiner’s
B. Patient’s head should be higher than that of the examiner head
C. Patient’s head should be lower than that of the examiner  Slightly leaning forward (not leaning on the chair) and back straight with
D. Head position is not important as the examiner can always their chin slightly protruding
tilt the head upwards  Legs should be uncrossed and flat on the floor.
 Patient’s feet should be close to the body as possible so that the
examiner can comfortably approach the patient.

[ENT] T.01 - Basic ENT Examination p.1.


4. What is the optimal position for the examiner? ANSWER:
A. The examiner should be seated C. Examiner can either sit or stand as long as they maintain optimal
B. Examiner must be standing head level in respect to that of the patient’s and will not slouch
C. Examiner can either sit or stand as long as they maintain
Examiner
optimal head level in respect to that of the patient’s and will  May stand or sit but must not stoop down and must be
not slouch comfortable.
D. Examiner must stoop down to have a commanding view of  Think of work ergonomics.
the field to be examined  You want to maintain a good position because doing the examination in
the long run could put a strain on the back.
[ENT] T.01 - Basic ENT Examination p.1.
5. What is the optimal position for the examiner? ANSWER:
A. Not important so as long as the examiner can see the field C. At the focal length of the head mirror being used
being examined
Focal length of 14 inches
B. At the examiner’s comfort
 This is the optimum or best length in which you have a small area with
C. At the focal length of the head mirror being used the brightest reflection you can get
D. Approximately 6 inches  This means that the examiner’s head is approximately 14 inches away
from the patient.
[ENT] T.01 - Basic ENT Examination p.1.

5 | 22 EARS, NOSE, AND THROAT Compiled Recalls (Finals) Editors | ENT TRANS TEAM
6. When doing an anterior rhinoscopy, on which side should ANSWER:
the speculum be held? B. On the contralateral hand of the nasal cavity being examined (i.e.
A. On the hand opposite to where the head mirror is placed so when examining the right nostril the speculum should be held by the
examiner at the left hand)
as not to block the angle of reflection of the light
B. On the contralateral hand of the nasal cavity being Speculum is held on the SAME SIDE of the nasal cavity examined – held
examined (i.e. when examining the right nostril the with a thumb and a middle finger keeping index finger free (RIGHT
speculum should be held by the examiner at the left hand) NOSTRIL = LEFT HAND OF EXAMINER)
C. At the examiner’s dominant hand
D. On the hand where the examiner is more comfortable with [ENT] T.01 - Basic ENT Examination p.3.
7. In doing a posterior rhinoscopy which of the following ANSWER:
materials/instrument is not used? D. Sterile gauze to hold the tongue
A. Tongue depressor Examination of the Nasopharynx
B. Small laryngeal mirror  Utilize same light source
C. Head mirror  Utilize a nasopharyngeal mirror size 0
D. Sterile gauze to hold the tongue  Tongue is retracted as with oral examination and nasopharyngeal
mirror slide over the (tongue) depressor
 Ask the patient to breathe through the nose to relax the soft palate
 Mirror is rotated around to visualize entirely the entire nasopharynx

[ENT] T.01 - Basic ENT Examination p.4.


8. In doing an indirect laryngoscopy, which of the following ANSWER:
materials/instruments is not used? C. Tongue depressor
A. Gauze
Examination of the Hypopharynx and Larynx
B. Light source  Utilize same light source
C. Tongue depressor  Proper positioning of the patient is vital
D. A laryngeal mirror  Patient is asked to stick out his/her tongue which is grasped by the
middle finger and the thumb while the index is used to push up on the
upper incisors of the patient (use of gloves or sterile gauze)
 Laryngeal mirror of adequate size is warmed or application of anti-
fogging solution is done
 Mirror is gently slid, avoiding posterior third of the tongue or posterior
pharyngeal wall and pressed upward against the soft palate
 If patient gags, ask the patient to pant/ apply topical anesthesia spray

[ENT] T.01 - Basic ENT Examination p.4.


9. In doing otoscopy of the right ear on which hand should ANSWER:
the otoscope be held? A. Right hand
A. Right hand
The otoscope is always held by the physician in the hand that
B. Left hand
correlates with the side of the ear to be examined, ie. the patient’s right
C. Dominant hand ear is examined with the otoscope in the examiner’s right hand.
D. Whichever the examiner is comfortable with
https://www.racgp.org.au/getattachment/78597fc3-e315-4d04-9fc738094160d5ba/200510chang.pdf
10. In doing an indirect laryngoscopy which area should be ANSWER:
avoided to prevent a gag reflex? C. Posterior pharyngeal wall
A. Anterior tongue
Avoid touching the tonsils, posterior pharyngeal wall, and posterior third of
B. Soft palate
the tongue to avoid gag reflex
C. Posterior pharyngeal wall
D. Tonsils [ENT] T.01 - Basic ENT Examination p. 4
11. What is the focal length of your head mirror? ANSWER:
A. 10 inches C. 14 inches
B. 12 inches Head mirror
C. 14 inches  3 1⁄2 inch mirror with a 1⁄2 inch hole at the center
 Focal length of 14 inches
D. 16 inches
 Should be placed as close as possible to the face – wider angle of view
 View should be binocular
 Examiner directs the area to be examined into the field of view and
avoids repositioning himself
 Substitute – focusable light on a headband

[ENT] T.01 - Basic ENT Examination p.1


12. How would you straighten the external auditory canal of a ANSWER:
21 year old? A. By pulling on the auricle upward, backward, and outward
A. By pulling on the auricle upward, backward, and outward
Pull backward and upward in adults
B. By pulling on the pinna downward and forward
C. By using the speculum of my otoscope
D. By pulling on the auricle downward and backward [ENT] T.01 - Basic ENT Examination p.2
13. What can be utilized in place of a head mirror and a ANSWER:
gooseneck lamp? D. A head light
A. A penlight Head Mirror
B. Your otoscopy  Substitute-focusable light on a head band
C. A drop light
D. A head light [ENT] T.01 - Basic NT Examination p.01

6 | 22 EARS, NOSE, AND THROAT Compiled Recalls (Finals) Editors | ENT TRANS TEAM
14. When you have focused the head mirror on the area you ANSWER:
wish to examine how will you move the next area to be C. By moving the head of the patient
examined? Head Mirror
A. By moving the lamp to change the angle of reflection  Examiner directs the area to be examined into the field of view and
B. Placing my head mirror on the other eye avoids repositioning himself
 The 14 inches length is a fixed angle so you cannot move your head
C. By moving the head of the patient
around. Maintain a fixed posture or angle and you move the part to be
D. By turning my head
examined into your field of view.
[ENT] T.01 - Basic NT Examination p.01
15. What is the proper way of handling your nasal speculum? ANSWER:
A. It is held on the opposite side of the nasal cavity being B. The index finger is used to grasp the nasal ala against the
examined speculum
B. The index finger is used to grasp the nasal ala against the
Nasal speculum
speculum  The index finger is used to grasp the nasal ala against the speculum
C. The speculum is closed or released prior to removal from
the nasal vestibule
D. The patient’s head is tilted slightly downward [ENT] T.01 - Basic ENT Examination p. 3

R.07 (Diseases of the Middle Ear)


QUESTION ANSWER/RATIONALE
1. Which of the following types of acute otitis media is the ANSWER:
most common C. Bacterial otitis media
A. Viral myringitis Four Types of AOM
1) Bacterial Otitis Media
B. Secretory otitis media
○ The usual and most common type of middle ear infection
C. Bacterial otitis media 2) Viral Myringitis
D. Necrotizing otitis media 3) Necrotic Otitis Media
4) Secretory Otitis Media
[ENT] T.03 - Diseases of the External Ear p.3
2. Which of the following antibiotics is used in the primary ANSWER:
treatment of acute otitis media A. Cefuroxime
A. Cefuroxime
Management of AOM
B. Clindamycin
 Antimicrobial therapy (7-10 days)
C. Ciprofloxacin o Amoxicillin/Ampicillin (first-line)
D. Metronidazole o Erythromycin and sulfisoxazole
o TMP-SMX
o Amoxicillin-clavulanate
o Cephalosporins

[ENT] T.03 - Diseases of the External Ear p.3


3. Which of the following is a condition that can be found in a ANSWER:
“safe ear”? C. Infected Eustachian Tube
A. Poorly aerated or sclerotic mastoid air cells
B. Peripherally located tympanic perforation
C. Infected Eustachian tube
D. Disease in attic or antrum

SAFE EAR

UNSAFE EAR

[ENT] T.03 - Diseases of the External Ear p.4


4. Which is considered a conservative treatment of chronic ANSWER:
otitis media? A. Removal of keratin debris
A. Removal of keratin debris
Conservative treatment of COM
B. Atticotomy
○ Removal of entrapped keratin: Direct or Saline irrigation
C. Myringotomy ○ Irrigation with 1:1 distilled white vinegar and 70% isopropyl alcohol for
D. Mastoidectomy stabilization

[ENT] T.03 - Diseases of the External Ear p.5

7 | 22 EARS, NOSE, AND THROAT Compiled Recalls (Finals) Editors | ENT TRANS TEAM
5. When is the highest incidence of acute otitis media? ANSWER:
A. 48-60 months of age C. 6-24 months of age
B. 5-36 months of age Incidence of AOM
○ Highest incidence occurs between 6 and 24 months of age
C. 6-24 months of age
○ Male predilection
D. 2-12 months of age ○ Crowding, poor hygiene, inadequate nutrition, delay in seeking medical
attention
[ENT] T.03 - Diseases of the External Ear p.3
6. Which of the following is a sign/symptom of acute otitis ANSWER:
media? A. Fever
A. Fever
Signs and Symptoms of AOM
B. A pearly white TM on otoscopy
 TM opaque, bulging, or congested
C. Vertigo
 Limited or no mobility of TM on pneumatic otoscopy
D. Hearing loss  Otorrhea
 Otalgia
 Fever
[ENT] T.03 - Diseases of the External Ear p.3
7. Which of the following bacteria is not involved in acute ANSWER:
otitis media? A. Pseudomonas aeruginosa
A. Pseudomonas aeruginosa
Microbiology
B. Streptococcus pyogenes
 Streptococcus pneumoniae
C. Staphylococcus aureus
 Haemophilus influenza
D. Haemophilus influenza  Branhamella catarrhalis
 Streptococcus pyogenes
 Staphylococcus aureus
 Staphylococcus epidermidis

[ENT] T.04 - Anatomy, Physiology, and Diseases of the Middle Ear p. 3


8. Which of the following is a normal flora of the middle ear? ANSWER:
A. S. aureus D. None of the above
B. S. pyogenes
In an otherwise healthy middle ear, it is supposed to be aseptic, meaning,
C. M. catarrhalis
there is no normal flora.
D. None of the above

[ENT] T.04 - Anatomy, Physiology, and Diseases of the Middle Ear p. 3.


9. This type of otitis media occurs mainly in children who are ANSWER:
severely ill or suffers from a systemic disease B. Necrotizing otitis media
A. Viral myringitis
Necrotic otitis media
B. Necrotizing otitis media
 Occurs chiefly in young children who are severely ill or suffer from a
C. Secretory otitis media
systemic disease
D. Bacterial otitis media

[ENT] T.04 - Anatomy, Physiology, and Diseases of the Middle Ear p. 3


10. This type of acute otitis media also occurs in children but ANSWER:
is seen less nowadays C. Secretory otitis media*
A. Viral myringitis
Secretory otitis media
B. Bacterial otitis media
 Predominantly occurs in children, most frequently seen nowadays*
C. Secretory otitis media
D. Necrotizing otitis media [ENT] T.04 - Anatomy, Physiology, and Diseases of the Middle Ear p. 3
11. This stage of acute otitis externa* is characterized by ANSWER:
erythema and decreased mobility of the tympanic C. Hyperemia
membrane
A. Exudation *Should be acute otitis media
Stage of Hyperemia/Retraction
B. Suppuration
 Onset of disease
C. Hyperemia
 Characterized by a generalized hyperemia of the mucoperiosteum
D. Coalescence  Mild earache, ear fullness, and fever
 Erythematous and markedly retracted eardrum is seen upon otoscopy

[ENT] T.04 - Anatomy, Physiology, and Diseases of the Middle Ear p. 7


12. Which of the following is not included in the management ANSWER:
of acute otitis externa? D. Tympanocentesis/Myringotomy
A. Antimicrobial
B. Supportive therapy Principles of therapy for Otitis Externa:
C. Aural toilette  Relief of pain – supportive
 Eliminate predisposing factors – supportive
D. Tympanocentesis/Myringotomy
 Thorough cleaning – aural toilette
Tympanocentesis/Myringotomy is mostly applicable for acute otitis
media

[ENT] T.03 - Diseases of the External Ear page 3.

8 | 22 EARS, NOSE, AND THROAT Compiled Recalls (Finals) Editors | ENT TRANS TEAM
13. Which of the following is a supportive therapy in the ANSWER:
management of acute otitis externa? C. Paracetamol
A. Vitamin B Complex
B. Amoxicillin Principles of therapy for Otitis Externa:
C. Paracetamol  Relief of pain – NSAIDs, non-opioid analgesics (acetamenophen)
 Eliminate predisposing factors
D. Nebulization
 Thorough cleaning

[ENT] T.03 - Diseases of the External Ear page 3


14. This procedure is defined as the aspiration of the middle ANSWER:
ear secretions for purpose of culture guided therapy 14. C. Tympanocentesis
A. Myringotomy 15. C. Myringotomy
B. Atticotomy
 Tympanocentesis – aspiration of the middle ear fluid for the
C. Tympanocentesis
purpose of culture of microorganisms
D. Aural toilette  Myringotomy – incision and drainage of TM for prompt relief of
15. This procedure is done to relieve the middle ear of pain and drainage of middle ear fluid
pressure to alleviate pain  Atticotomy – transcanal surgical approach, done by drilling the scutum
A. Tympanocentesis for cholesteatoma drainage
B. Atticotomy  Aural toilette - “flushing”
C. Myringotomy
D. Aural toilette [ENT] T.04 - Anatomy, Physiology, and Diseases of the Middle Ear page 3
16. This procedure may be done with or without the insertion ANSWER:
of a ventilation tube C. Myringotomy
A. Tympanocentesis In myringotomy, you may or may not insert a ventilation tube. The
ventilation tube is to maintain a patent opening between middle and outer
B. Atticotomy
ear for air to enter and for the exudate to exit.
C. Myringotomy
D. Aural toilette [ENT] T.04 - Anatomy, Physiology, and Diseases of the Middle Ear page 3
17. How many weeks of long-standing inflammation and ANSWER:
suppuration can cause permanent irreversible damage to C. 6
the middle ear cleft and underlying bone?
Suppurations continuing for more than 6 weeks, usually causing
A. 8
irreversible damage to the middle ear cleft mucosa and underlying bone
B. 4 which includes the ossicles.
C. 6
D. 12
[ENT] T.04 - Anatomy, Physiology, and Diseases of the Middle Ear page 4
18. Which of the following subareas are not affected by ANSWER:
chronic otitis media? A. Auditory canal
A. Auditory canal
Chronic otitis media is a long-standing inflammation of the middle ear
B. Middle ear cleft
cleft, eustachian tube, tympanum, attic (epitympanum) and the
C. Eustachian tube mastoid air cell system.
D. Mastoid air cells
[ENT] T.04 - Anatomy, Physiology, and Diseases of the Middle Ear page 4
19. Which of the following is a sign/symptom of chronic otitis ANSWER:
media? A. Persistent tympanic membrane perforation
A. Persistent tympanic membrane perforation
B. Tympanic membrane erythema and limited mobility Signs and symptoms:
C. Fever  Foul-smelling otorrhea
 Hearing loss
D. Bulging tympanic membrane
 Earache
 Dizziness
 Headache
 Persistent tympanic membrane perforation
 Ossicular bone erosion
 Granulation tissue/cholesteatoma

[ENT] T.04 - Anatomy, Physiology, and Diseases of the Middle Ear page 4
20. Which of the following is not a characteristic otoscopic ANSWER:
finding of cholesteatoma? A. Auditory canal inflammation
A. Auditory canal inflammation Otoscopic findings:
B. Retraction at the posterior superior portion of the tympanic  Retraction pockets along the posterosuperior portion of the TM
membrane  Whitish to yellowish flakes
C. White to yellowish flakes  Erosion of the ossicular bones
D. Yellowish to brownish discharge  Yellowish to brownish, foul-smelling discharge
 Presence of granulation tissue, and/or aural polyp

[ENT] T.04 - Anatomy, Physiology, and Diseases of the Middle Ear page 4

9 | 22 EARS, NOSE, AND THROAT Compiled Recalls (Finals) Editors | ENT TRANS TEAM
R.08 (Anatomy, Physiology, and Diseases of the External Ear)
QUESTION ANSWER/RATIONALE
1. In clearing the ears of impacted cerumen, when is aural ANSWER:
toilette or aural irrigation not advisable? B. When there is a suspected or known tympanic membrane
A. When there is diffuse swelling in the ear canal perforation
B. When there is a suspected or known tympanic membrane
Impacted Cerumen
perforation  Water irrigation can be done
C. In pediatric patients as the experience can prove to be o Not done if TM is perforated
traumatic o Do not do aural toilette (“flushing”) if there is a suspected TM
D. In dry, hard cerumen since it cannot be readily removed perforation so always get a thorough patient’s history. Ask for
discharge, ear pain, and diagnosis of middle ear problems among
others.

[ENT] T.03 - Diseases of the External Ear p.1


2. Which of the following cannot be used as cerumenolytics? ANSWER:
A. Commercially prepared ear drops D. Sterile saline solution
B. Hydrogen peroxide
Hardened cerumen can be softened using ear drops (e.g., docusate
C. Oil
sodium, paradichlorobenzene), oil, or hydrogen peroxide.
D. Sterile saline solution
[ENT] T.03 - Diseases of the External Ear p.1
3. In lacerations of the external ear, which of the following is ANSWER:
included in the management? C. Antibiotic therapy
A. Layered repair of the laceration (to include all layers)
Management of lacerated/incised wounds of the auricle:
B. Secondary intention healing
 Cleaning
C. Antibiotic therapy
 Suturing
D. Analgesic therapy  Antibiotic therapy
 Daily wound care

[ENT] T.03 - Diseases of the External Ear p.2


4. Hematoma in the external ear, untreated can result in this ANSWER:
unwanted outcome A. Cauliflower-ear deformity
A. Cauliflower-ear deformity
Hematomas of the external ear are commonly seen in wrestlers and
B. Resorption of the underlying cartilage
boxers. If untreated it may lead to “cauliflower ears” (auricular
C. Suppurative chondritis chondropathy).
D. Necrosis of the overlying skin

[ENT] T.03 - Diseases of the External Ear p.2


5. Furunculosis in the external auditory canal can be treated ANSWER:
by which of the following ways? B. Systemic antibiotics and analgesics
A. Topical antibiotics and anti-inflammatory ear drops
B. Systemic antibiotics and analgesics  Incision and drainage are the best option
 Systemic antibiotics (antistaphylococcal) and analgesics
C. Avoid incising or draining the abscess as this may spread
infection to the confluent areas
D. Antibiotics are targeted at anaerobic bacteria [ENT] T.03 – Diseases of the External Ear p.3
6. Which should be done first in the treatment of hematomas ANSWER:
in the external ear? 6. B. Drainage
A. Pressure dressing 7. C. Staphylococcal
B. Drainage
Management of Hematomas
C. Observation  Clean the area
D. Antibiotic therapy  Incision and Drainage
7. Antibiotic treatment in the hematoma formation in the  Pressure bolsters are sutured in place to close the dead space and
external ear is directed at? prevent a recollection of blood
A. Atypical bacteria  Give anti-staphylococcus antibiotics
B. Gram negative
C. Staphylococcal
D. Anaerobic bacteria [ENT] T.03 – Diseases of the External Ear p.2
8. Which of the following is the mechanism by which damage ANSWER:
is caused by frostbite in the external ear? 8. A. Microvascular insult leading to local ischemia
A. Microvascular insult leading to local ischemia 9. B. Rapid warming and antibiotics initially done
B. Generation of free radicals
C. Loss of innervation leading to atrophy Frostbites
D. Loss of nutritional supply  Injury secondary to direct cellular damage and microvascular
9. Which of the following is true about the management of insult leading to local ischemia
frostbite in the external ears?  Initial management includes rapid warming and antibiotics
A. Immediate surgical and antibiotics is done  The delay of debridement is done to assess fully tissue status: if it is
B. Rapid warming and antibiotics initially done salvageable or necrosed
C. Expectant observation
D. Gradual heating of the exposed part
[ENT] T.03 – Diseases of the External Ear p.2

10 | 22 EARS, NOSE, AND THROAT Compiled Recalls (Finals) Editors | ENT TRANS TEAM
10. Which of the following is true about perichondritis or ANSWER:
chondritis? D. Can be secondary to trauma
A. Always with a precedent cause
Perichondritis / Chondritis
B. Characterized by pain localized in the external auricle
 Infection of perichondrium or cartilage
C. Can be autoimmune in etiology
 Result of trauma to auricle
D. Can be secondary to trauma  May be spontaneous (overt diabetes)
 Symptoms: Pain over auricle and extend deep in canal. Pruritus,
Induration, Edema
[ENT] T.03 – Diseases of the External Ear p.2
11. Which of the following is true in the management of ANSWER:
foreign bodies in the external auditory canal? C. Inspection after removal should be done to assess for any
A. Removal can be done blindly additional injuries
B. After extraction of the foreign body there is no longer any
Foreign Bodies
needed precaution  Visualization is mandatory for removal
C. Inspection after removal should be done to assess for any  Sedation or general anesthesia may be required if patient is
additional injuries uncooperative
D. Even if the patient is uncooperative there is no indication for  After removal, additional injuries must be assessed.
general anesthesia or sedation
[ENT] T.03 – Diseases of the External Ear p.3
12. Swimmer’s ear is a condition which is commonly caused ANSWER:
by which of the following bacteria? 12. C. Pseudomonas sp.
A. Staphylococcus 13. B. Topical corticosteroids
B. Anaerobic bacteria
Diffuse Otitis Externa
C. Pseudomonas species  Swimmer’s Ear
D. Aspergillus niger species  Pseudomonas sp. and viruses
13. Swimmer’s ear can be managed by which of the following?  Occurs during hot humid weather
A. Systemic antibiotics  Topical corticosteroids and antibiotics
B. Topical corticosteroids
C. Antifungal preparations
D. Systemic antiviral medication [ENT] T.03 – Diseases of the External Ear p.4
14. Treatment of herpes zoster oticus includes which of the ANSWER:
following? C. Systemic steroids for facial palsy
A. Systemic antibiotics for infection
Herpes Zoster Oticus (Ramsay-Hunt Syndrome)
B. Surgical debridement
 Chickenpox virus
C. Systemic steroids for facial palsy
 Symptomatic treatment:
D. Topical keratolytics o Systemic steroids for facial palsy
o Local ear care
o Topical antibiotics (drops) for suspected secondary bacterial
infection
o Analgesics
o Antiherpetic agent
[ENT] T.03 – Diseases of the External Ear p.4
15. This condition can be precipitated by the neomycin ANSWER:
componenct in common otic preparations B. Contact dermatitis
A. Diffuse otitis externa
Contact dermatitis
B. Contact dermatitis
 Erythematous, edematous, pruritic lesions, with vesicles or exudates.
C. Perichondritis
 Common among hearing aid and ear plug users or patients who wear
D. Otomycosis earrings
 Hypersensitivity to neomycin component of otic preparations

[ENT] T.03 – Diseases of the External Ear p.4


16. A 50 year old male complains of lesions on the external ANSWER:
ear which is described as raised, erythematous with a A. Seborrheic dermatitis
grayish scale and thick lichenification. He is a known
Seborrheic dermatitis
frequent swimmer and uses a behind the ear hearing aid.
 Malasezzia furfur has a role
What is the most probable diagnosis?
 Erythematous
A. Seborrheic dermatitis  Raised with a greasy scale
B. Psoriasis  Pink or orange and thick lichenification if chronic o Topical steroids
C. Contact dermatitis and keratolytic
D. Otomycosis
[ENT] T.03 – Diseases of the External Ear p.4
17. A 18 year old female came in for lesions on her right ear. ANSWER:
On inspection there were erythematous papules forming A. Psoriasis
large patches with scaling. What is your probable Psoriasis:
diagnosis?  Etiology is unknown
 Multifactorial causes
A. Psoriasis
 Erythematous papules from large patches with thick scale
B. Seborrheic dermatitis
 Epithelial hyperplasia with hyperkeratosis
C. Contact dermatitis
D. Otomycosis [ENT] T.03 – Diseases of the External Ear p.5

11 | 22 EARS, NOSE, AND THROAT Compiled Recalls (Finals) Editors | ENT TRANS TEAM
18. The following condition is caused by Group A beta ANSWER:
hemolytic streptococci B. Erysipelas
A. Perichondritis
Erysipelas
B. Erysipelas
 Acute superficial cellulitis
C. Relapsing polychondritis
 It is a dermal infection that may involve the skin of the head and face,
D. Necrotizing otitis externa including the ear.
 Group A, beta hemolytic streptococci
 Skin: Bright red; well-demarcated, advancing margin

[ENT] T.03 – Diseases of the External Ear p.5


19. This condition in the external ear can be fatal ANSWER:
A. Psoriasis 19. C. Necrotizing otitis externa
B. Relapsing polychondritis 20. C. Pseudomonas species
C. Necrotizing otitis externa
Necrotizing otitis externa
D. Ramsay-Hunt syndrome  An aggressive and potentially fatal infection originating in the external
20. What is the most common etiologic agent of necrotizing canal, with progressive spread along the soft tissues and bone of the
otitis externa? skull base, ultimately involving intracranial structures. It is also (more
A. Beta hemolytic bacteria accurately) known as necrotizing otitis externa.
B. Streptococcus species  Potentially lethal infection of EAC and surrounding structures
C. Pseudomonas species  Typically seen in diabetics and immunocompromised patients
D. Malassezia furfur  Pseudomonas aeruginosa is the usual culprit followed by S. aureus.
 Aspergillus sp are the most common fungal pathogens to cause this
disorder, but infections by more exotic molds have also been reported.

[ENT] T.03 – Diseases of the External Ear p.5

R.09 (Audiology)
QUESTION ANSWER/RATIONALE
1. How is audiometric zero defined? ANSWER:
A. The total absence of sound D. The median average hearing threshold of young adults with no
B. The median average hearing threshold of the general history of hearing problem, ear infection or recent colds
population
Audiometric Zero (ANSI)
C. The median average hearing threshold of young children  The median average hearing threshold of young adults with no
with no history of ear infection or recent colds history of hearing problem, ear infection or recent colds.
D. The median average hearing threshold of young adults with  Each frequency has its separate zero with zero calibrated values built
no history of hearing problem, ear infection or recent colds into the audiometer.

[ENT] T.07 - Audiology p.5


2. What is the criteria of conductive hearing loss on an ANSWER:
audiometry? 2. C. When bone conduction thresholds are better than air
A. When bone conduction thresholds are the same as air conduction by 10 db or more & are normal
3. A. When bone conduction thresholds are the same as air
conduction & neither is normal
conduction & neither is normal
B. When bone conduction thresholds are reduced but are still 4. B. When bone conduction thresholds are reduced but are still
better than air conduction by 10 db or more better than air conduction by 10 db or more
C. When bone conduction thresholds are better than air
conduction by 10 db or more & are normal
D. None of the above Air conduction-Bone conduction Relationship
3. What is the criteria of sensorineural hearing loss on an A. When BC thresholds are better than air conduction by 10 db or
audiometry? more and are normal, the loss is conductive
A. When bone conduction thresholds are the same as air B. When BC thresholds are the same as air conduction and neither is
normal, the loss is sensorineural
conduction & neither is normal
B. When bone conduction thresholds are reduced but are still C. When BC thresholds are reduced but are still better than air
conduction by 10 db or more, the loss is mixed or combined
better than air conduction by 10 db or more
C. When bone conduction thresholds are better than air
conduction by 10 db or more & are normal
D. None of the above
4. What is the criteria of mixed type hearing loss on an
audiometry?
A. When bone conduction thresholds are the same as air
conduction & neither is normal
B. When bone conduction thresholds are reduced but are still
better than air conduction by 10 db or more
C. When bone conduction thresholds are better than air
conduction by 10 db or more & are normal
D. None of the above

[ENT] T.07 - Audiology p.5

12 | 22 EARS, NOSE, AND THROAT Compiled Recalls (Finals) Editors | ENT TRANS TEAM
5. What is the maximum decibels HTL for a normal hearing ANSWER:
threshold? D. 25 dB
A. 10 dB
B. 15 dB
C. 20 dB
D. 25 dB

 It is acceptable if the patient would hear deviation by 25 dB hearing


threshold.
 25-40 dB is mild hearing loss
 40-60 dB is moderate hearing loss
 60-80 dB is severe hearing loss
 >80 dB is profound hearing loss
[ENT] T.07 - Audiology p.5
6. What is the difference of Speech reception threshold (SRT) ANSWER:
and Speech discrimination threshold (SDT)? D. SRT uses spondee words
A. SRT has an expected score of 94 of 100%
 SDT has an expected score of 94 of 100%
B. SDT agrees closely with the average of pure tone at 500 Hz,
 SRT agrees closely with the average of pure tone at 500 Hz, 1000 Hz,
1000 Hz, and 2000 Hz and 2000 Hz
C. SRT is presented at 20 to 40 dB above SDT or the patient’s  SDT is presented at 20 to 40 dB above SDT or the patient’s most
most comfortable level comfortable level
D. SRT uses spondee words  SRT uses spondee words
[ENT] T.07 - Audiology p.7
7. Which of the following tests is considered to be the most ANSWER:
accurate measurement of auditory function and is B. Audiotory Braintem Evoked Response
completely objective?
Brainstem Evoked Response Audiometry (BERA)
A. Pure tone audiometry
 It measures the potential arising in the auditory nerve and brainstem
B. Auditory Brainstem evoked response structures with a latency of approximately up to 10msec.
C. Play audiometry  It is an accurate measure of the auditory function & is completely
D. Cardiovascular audiometry objective
o the golden standard for audiometry
o Results cannot be cheated because the electric signal movements
will be measured

[ENT] T.07 - Audiology p.8


8. This test is used to measure the compliance of the ANSWER:
tympanic membrane B. Tympanometry
A. Otoacoustic emission testing
Tympanometry
B. Tympanometry
 Indirect measure of the compliance (mobility) of the TM and the
C. Pure tone audiometry
ossicular chain system under conditions of positive, normal, and
D. Speech discrimination test
negative pressure

[ENT] T.07 - Audiology p.6


9. How many hours is the maximum amount of time allowed ANSWER:
for a continuous exposure to sounds with 90 dB without D. 8
incurring damage to the auditory system?
8 hours
A. 14
 The maximum exposure duration with 90 dB sound without incurring
B. 12 damage to the auditory system.
C. 10
D. 8
[ENT] T.07 - Audiology p.11
10. This part of pure tone audiometry is taken as a measure of ANSWER:
the integrity of acoustic nerve C. Both
A. Air conduction
Both Air conduction and Bone conduction
B. Bone conduction
 The pure tone audiometry converts electrical energy to either acoustic
C. Both or vibration type to assess hearing integrity. An oscillator is placed on
D. None of the above the mastoid area for bone conduction and earphones are placed on the
ears for acoustic output.

[ENT] T.07 - Audiology p.7

13 | 22 EARS, NOSE, AND THROAT Compiled Recalls (Finals) Editors | ENT TRANS TEAM
11. In Schwabach’s test, a diminished result has a hearing ANSWER:
loss that is described as B. Sensorineural
A. Conductive
B. Sensorineural Diminished Result in Schwabach’s Test
C. Mixed  Sensorineural
D. Normal  Locus is Cochlear/Retrocochlear

[ENT] T.07 - Audiology p.4


12-13. In Bing’s Test, a positive result can be interpreted as ANSWER:
A. Conductive hearing loss B. Normal hearing
B. Normal hearing
C. Mixed hearing Positive Result in Bing’s Test
D. None of the above  Increase in loudness/Decrease in loudness
 Normal/ SN
 Locus is None/ Cochlear/Retrocochlear

[ENT] T.07 - Audiology p.4


14. In Bing’s Test, a negative result can be interpreted as ANSWER:
A. Conductive hearing loss A. Conductive hearing loss
B. Normal hearing
Negative Result in Bing’s Test
C. Mixed hearing
 Conductive
D. None of the above
 Locus is EAC/ME

[ENT] T.07 - Audiology p.4


15. This tuning for test compares the patient’s air conduction ANSWER:
with the bone conduction A. Rinne’s Test
A. Rinne’s Rinne’s Test
B. Webber’s  Compare’s patient’s hearing by bone conduction against air
conduction (AC vs BC). Weber’s Test on the other hand measures
C. Schwabach’s lateralization of hearingby bone conduction against air conduction (AC
D. Bing’s vs BC).
 Weber’s Test on the other hand measures lateralization of hearing.
 Schwabach’s Test compares patients bone conduction against normal
reference.
 Bing’s Test in an application of an occlusion test.
[ENT] T.07 - Audiology p.7-8

R.10 (Anatomy, Physiology, and Diseases of the Oral Cavity, Oropharynx, and Hypopharynx)
QUESTION ANSWER/RATIONALE
1. Which is not a boundary of the oropharynx? ANSWER:
A. Tongue base C. Anterior Faucial Pillars
B. 2nd and 3rd cervical vertebra Anterior Faucial Pillar
 The anterior facial Pillar or the Palatoglossal Arch formed by the
C. Anterior faucial pillars
palatoglossus is the posterior border of the Oral Cavity.
D. Posterior faucial pillars The Boundary of Oropharynx include:
 Anterior: Tongue base and lingual tonsils
 Posterior: 2nd and 3rd cervical vertebrae with prevertebral fascia
 Lateral: Faucial Pillars with flank of the palatine tonsils.

[ENT] T.15 - Anatomy, Physiology, and Disease of the Oral Cavity p.3
[ENT] T.16 - Anatomy, Physiology, and Disease of the Oropharynx and Hypopharynx p.1
2. Majority of the oropharynx and hypopharynx is lined with ANSWER:
what type of epithelium? B. Stratified, nonkeratinized squamous epithelium
A. Ciliated pseudostratified columnar epithelium
Mucosal lining of the oropharynx and hypopharynx:
B. Nonkeratinized stratified squamous epithelium
Stratified, nonkeratinized squamous epithelium
C. Keratinized stratified squamous epithelium
D. Simple columnar epithelium [ENT] T.16 – Anatomy, Physiology, and Diseases of the Oropharynx and Hypopharynx p.1
3. What is the superior border of the hypopharynx? ANSWER:
A. Hyoid bone A. Hyoid bone
B. Superior border of the thyroid cartilage
Borders of the hypopharynx
C. Superior border of the epiglottis
 Superior border: hyoid bone, upper esophageal sphincter (UES)
D. Base of the tongue
 Inferior border: cricopharyngeus
 Anterior border: epiglottis, paired aryepiglottic folds and arytenoid
cartilage

Throat Anatomy. By Tjoson Tjoa, Department of Otolaryngology-Head and Neck Surgery, University
of California, Irvine, School of Medicine. July 10, 2013
4. Which of the following is not true about the pharyngeal ANSWER:
musculature? D. They are all overlapping
A. There is a gap between each muscle
Only the constrictor pharyngis medius and inferior muscles overlap
B. The inferior most fibers joins with the esophagus
C. Almost all fibers run an oblique direction
D. They are all overlapping [ENT] T.16 – Anatomy, Physiology, and Diseases of the Oropharynx and Hypopharynx p.2

14 | 22 EARS, NOSE, AND THROAT Compiled Recalls (Finals) Editors | ENT TRANS TEAM
4. Which portion would have fibers of the pharyngeal muscles ANSWER:
would have fibers that run a transverse direction? C. Constrictor inferioris
A. Constrictor superioris
Killian–Jamieson region: between the oblique and transverse fibers of
B. Constrictor medius
the constrictor pharyngis.
C. Constrictor inferioris
D. None of the above

[ENT] T.16 – Anatomy, Physiology, and Diseases of the Oropharynx and Hypopharynx p.2
6. This weak point of the pharyngeal musculature occurs ANSWER:
between the constrictor pharyngis inferior and the A. Killian triangle
uppermost fibers of the cricopharyngeus muscle.
Killian triangle is between the constrictor pharyngis inferior and the
A. Killian triangle
uppermost fibers of the cricopharyngeus muscle.
B. Killian-Jamieson triangle
C. Lamier’s triangle
D. Killian-Jamieson region [ENT] T.16 – Anatomy, Physiology, and Diseases of the Oropharynx and Hypopharynx p.2
7. This weak point of the pharyngeal musculature occurs ANSWER:
between the oblique and transverse fibers of the D. Killian-Jamieson region
constrictor pharynges.
Killian–Jamieson region: between the oblique and transverse fibers of
A. Killian triangle
the constrictor pharynges.
B. Killian-Jamieson triangle
C. Lamier’s triangle
D. Killian-Jamieson region [ENT] T.16 – Anatomy, Physiology, and Diseases of the Oropharynx and Hypopharynx p.2
8. This weak point of the pharyngeal musculature is bounded ANSWER:
above by the cricopharyngeus and below by the C. Lamier’s triangle
uppermost fibers of the esophageal musculature.
Laimer triangle, which is bounded above by the cricopharyngeus and
A. Killian triangle
below by the uppermost fibers of the esophageal musculature.
B. Killian-Jamieson triangle
C. Lamier’s triangle
D. Killian-Jamieson region [ENT] T.16 – Anatomy, Physiology, and Diseases of the Oropharynx and Hypopharynx p.2
9. This weak point of the pharyngeal musculature is the most ANSWER:
common area of occurrence of hypopharyngeal diverticula A. Killian triangle
A. Killian triangle
Killian triangle: between the constrictor pharyngis inferior and the
B. Killian-Jamieson triangle
uppermost fibers of the cricopharyngeus muscle - common site for the
C. Lamier’s triangle formation of hypopharyngeal diverticula.
D. Killian-Jamieson region
[ENT] T.16 – Anatomy, Physiology, and Diseases of the Oropharynx and Hypopharynx p.2
10. Which of the tonsils is covered with stratified squamous ANSWER:
epithelium? A or D
A. Palatine tonsils
Palatine and lingual tonsils are covered by stratified, nonkeratinized
B. Tubulopharyngeal tonsils
squamous epithelium
C. Pharyngeal tonsils
D. Lingual tonsils [ENT] T.16 – Anatomy, Physiology, and Diseases of the Oropharynx and Hypopharynx p.2
11. Up to what age are your palatine tonsil have pronounced ANSWER:
activity? B. 8-10 years old
A. 6-8 years old
Activity pronounced during childhood until 8-10 years of age, when
B. 8-10 years old
immunologic challenges from the environment induce hyperplasia of the
C. 10-12 years old palatine tonsils.
D. 12-14 years old
[ENT] T.16 – Anatomy, Physiology, and Diseases of the Oropharynx and Hypopharynx p.3
12. In doing a chest radiogram in a patient who ingested ANSWER:
caustic fluids, what are we actually investigating? B. Esophageal perforation
A. Aspiration pneumonia
Chest radiograph must be done to check for mediastinal widening due to
B. Esophageal perforation
esophageal perforation.
C. Esophageal stenosis
D. Extent of mucosal erosion
[ENT] T.16 – Anatomy, Physiology, and Diseases of the Oropharynx and Hypopharynx p.4

15 | 22 EARS, NOSE, AND THROAT Compiled Recalls (Finals) Editors | ENT TRANS TEAM
13. What is the initial appearance of mucosa injured by a ANSWER:
scalding or caustic fluids? C. Erythema
A. Blister formation
Diagnosis – Acute evaluation
B. Ulceration
Begins with a mirror examination of the oral cavity, oropharynx,
C. Erythema hypopharynx, and larynx - mucosa initially appears erythematous and
D. Perforation edematous and later may show epithelial defects and a whitish fibrin
coating.

[ENT] T.16 – Anatomy, Physiology, and Diseases of the Oropharynx and Hypopharynx p.9
14. Which of the following is true about rapid streptococcal ANSWER:
tests? C. Asymptomatic patients with positive test should
A. Results can be done in one hour not be treated with antibiotics
B. Has a sensitivity of 75%
Rapid Streptococcal test
C. Asymptomatic patients with positive test should not be
treated with antibiotics
• Makes use of colloid-labeled specific antibodies which are placed onto
reaction strips along with the pharyngeal smear
D. Streptococcal angina is ruled out if the test is negative in
symptomatic patients
• Sensitivity of 80%to 90% making them useful tools in deciding whether to
administer antibiotics
• Should be correlated to clinical findings
• Asymptomatic patients with a positive rapid test should not be
placed on antibiotics
• Conversely, a culture should be taken in cases where there is clinical
suspicion of streptococcal tonsillitis but the rapid test is negative

[ENT] T.16 - Anatomy, Physiology, and Diseases of the Oropharynx and Hypopharynx p. 4
15. This acute inflammatory condition of the pharynx is ANSWER:
characterized by rash formation over the entire body with D. Scarlet Fever
perioral pallor
Scarlet fever – Clinical presentation
A. Streptococcal angina
B. Plaut-Vincent Angina • Rash that begins on the trunk with sparing of the area around the mouth
is spared (“perioral pallor”)
C. Diphtheria
D. Scarlet fever • Pathognomonic feature is a bright red tongue with a glistening surface
and hyperplastic papillae (“raspberry tongue,”)
• The tonsils are greatly swollen with a deep red color
• Occasionally there is an enanthema of the soft palate with
hemorrhagic areas

[ENT] T.16 - Anatomy, Physiology, and Diseases of the Oropharynx and Hypopharynx p. 5
16. This acute inflammatory condition of the pharynx is ANSWER:
caused by fusiform rods and spirochetes B. Plaut-Vincent Angina
A. Streptococcal angina
Plaut – Vincent Angina
B. Plaut-Vincent Angina
C. Diphtheria • Inflammatory disease is caused by fusiform rods and spirochetes
D. Scarlet fever • Unilateral dysphagia and a fetid breath odor with very little malaise
• The causative organisms can be detected by the direct microscopic
examination of a gram-stained smear

[ENT] T.16 - Anatomy, Physiology, and Diseases of the Oropharynx and Hypopharynx p. 5
17. This acute inflammation of the pharynx is caused by a ANSWER:
bacteria that produces a toxin that causes cell necrosis C. Diphtheria
and ulcerations
Diphtheria
A. Streptococcal angina
B. Plaut-Vincent Angina • Caused by Corynebacterium diphtheriae, transmitted by droplet inhalation
or skin-to-skin contact with an incubation period of 1–5 days
C. Diphtheria
D. Scarlet fever • Bacterium produces a special endotoxin that causes epithelial cell
necrosis and ulcerations

[ENT] T.16 - Anatomy, Physiology, and Diseases of the Oropharynx and Hypopharynx p. 5
18. Which of these following is not a measure to treat ANSWER:
peripheral obstructive sleep apnea syndrome? D. Use of muscle stimulants to increase muscle tone during sleep
A. Continuous positive air pressure masks
Peripheral Obstructive Sleep Apnea Syndrome – Treatment
B. Uvulopharyngoplastry
C. Weight reduction • General measures: weight reduction, abstinence from
alcohol and nicotine, and avoiding big meals, and avoid the use of
D. Use of muscle stimulants to increase muscle tone during
sedatives
sleep
• Esmarch splint (a mandibular advancement device), an occlusive splint
that advances the lower jaw
• Continuous positive airway pressure (CPAP) mask
• Uvulopalatopharyngoplasty (UPPP) with tonsillectomy

[ENT] T.16 - Anatomy, Physiology, and Diseases of the Oropharynx and Hypopharynx p. 8

16 | 22 EARS, NOSE, AND THROAT Compiled Recalls (Finals) Editors | ENT TRANS TEAM
19. Diverticula in the hypopharynx is usually of what type ANSWER:
A. Traction diverticulum C. Both can occur in the hypopharynx
B. Pulsion diverticulum
Diverticula – Two distinct types:
C. Both can occur in the hypopharynx
D. None of the above • Pulsion Diverticula - Mucosa herniates through a weak point in the
muscular coat due to a rise of intraluminal pressure
• Traction diverticula - Which usually form at parabronchial sites due to
scar traction following hilar lymphadenitis and involve all layers of the
esophageal wall

[ENT] T.16 - Anatomy, Physiology, and Diseases of the Oropharynx and Hypopharynx p10
20. In accidental foreign body ingestion, where are they ANSWER:
usually embedded/found? C. Palatine tonsil
A. Pyriform sinus
Foreign bodies are most commonly located in the tonsils and at the
B. Vallecula
tongue base. Foreign bodies typically become lodged in the hypopharynx
C. Palatine tonsil or in the upper constriction of the esophagus.
D. Vestibular folds
[ENT] T.16 – Anatomy, Physiology, Diseases of the Oropharynx and Hypopharynx p.4, 10

R.11 (Anatomy and Physiology of the Nose and Paranasal sinuses)


QUESTION ANSWER/RATIONALE
1. The following paranasal sinuses drain into the middle ANSWER:
meatus, except: B. Posterior ethmoid
A. Frontal sinus Middle Meatus:
B. Posterior ethmoid  Middle ethmoidal air cells – bulla
 Anterior Ethmoidal air cells – Hiatus Semilunaris
C. Anterior ethmoid
 Maxillary Sinus – Hiatus Semilunaris
D. Maxillary sinus
 Frontal Sinus – Infundibulum
Superior Meatus: Posterior Ethmoidal air cells

[ENT] T.09 – Anatomy and Physiology of the Nose and Paranasal Sinuses p.9.
2. Which of the following drains into the inferior meatus: ANSWER:
A. Sphenoid Sinus C. Nasolacrimal duct
B. Posterior ethmoids
Inferior Meatus: Opening of Nasolacrimal duct
C. Nasolacrimal duct
D. None of the above [ENT] T.09 – Anatomy and Physiology of the Nose and Paranasal Sinuses p.9.
3. The following are components of the nasal septum, except: ANSWER:
A. Vomer D. Cribriform plate of the ethmoid
B. Maxillary crest
Bony components of the septum:
C. Perpendicular plate of the ethmoid
 Nasal crest of the palatine bone
D. Cribriform plate of the ethmoid
 Nasal crest of the maxilla and premaxilla
 Vomer
 Perpendicular plate of the ethmoid
 Nasal crest of the frontal bone
 Spine of the paired nasal bones

[ENT] T.09 – Anatomy and Physiology of the Nose and Paranasal Sinuses p.10.
4. Which of the paranasal sinuses is the only sinus routinely ANSWER:
present at birth. B. Maxillary sinus
A. Anterior ethmoid
The maxillary sinus is present at birth but remains very small until the
B. Maxillary sinus second dentition, because the presence of tooth germs in the maxilla limit
C. Frontal sinus the extent of the sinuses.
D. Posterior ethmoid
Probst R, Grevers G, Iro H. (2017) Basic Otolaryngology: A Step-by- Guide 2 nd Ed. Stuttgard:
Thieme chapter 1 (Anatomy and physiology of the nose, paranasal sinuses and face) page 4
5. All of the following are part of the lateral border of the ANSWER:
nasal cavity except: D. None of the above
A. Turbinates
Lateral wall of the nasal cavity
B. Horizontal plate of palatine bone
 Nasal bone
C. Maxillary bone
 Maxillary
D. None of the above  Inferior concha/turbinate
 Palatine
 Ethmoid
 Sphenoid

[ENT] T.09 – Anatomy and Physiology of the Nose and Paranasal Sinuses p.6.
6. The largest turbinate/conchae ANSWER:
A. Inferior A. Inferior
B. Middle Inferior Turbinate and Meatus:
C. Superior  Largest turbinate
D. Supreme  Separate bone covered by thick mucous membrane

[ENT] T.09 – Anatomy and Physiology of the Nose and Paranasal Sinuses p.7

17 | 22 EARS, NOSE, AND THROAT Compiled Recalls (Finals) Editors | ENT TRANS TEAM
7. The middle meatus contains all of the following except: ANSWER:
A. Ethmoid bulla D. Sphenoethmoidal rescess
B. Uncinate process
Rest of middle meatus posteroinferiorly contains:
C. Hiatus semilunaris
 Ethmoidal bulla
D. Sphenoethmoidal recess
 Uncinate Process
 Semilunar hiatus

[ENT] T.09 – Anatomy and Physiology of the Nose and Paranasal Sinuses p.8
8. Which of the following parts of the nasal septum unites ANSWER:
with the cribriform plate B. Perpendicular Plate of Ethmoid
A. Vomer
Perpendicular Plate of Ethmoid
B. Perpendicular plate of ethmoid
 Forms upper 1/3 or more of the nasal septum
C. Quadrangular cartilage
 Unites superiorly with cribriform plate of the ethmoid
D. Crest of maxillary bone
[ENT] T.09 – Anatomy and Physiology of the Nose and Paranasal Sinuses p.10
9. All of the following are part of kiesselbach plexus except: ANSWER:
A. Ethmoidal artery C. Lesser palatine artery
B. Superior labial artery
Arteries important in the Kieselbach area: LEGS
C. Lesser palatine artery  L- superior Labial artery
D. Sphenopalatine artery  E- anterior and posterior Ethmoidal artery
 G- Greater palatine artery
 S- Sphenopalatine artery

[ENT] T.09 – Anatomy and Physiology of the Nose and Paranasal Sinuses p.11
10. The space between the bulla ethmoidalis and uncinate ANSWER:
process is called: D. Hiatus Semilunaris
A. Agger nasi
Hiatus semilunaris inferioris
B. Olfactory cleft
 The gap between the ethmoid bulla and the free edge of the uncinate
C. Frontonasal duct
process defines the hiatus semilunaris
D. Hiatus semilunaris
[ENT] T.09 – Anatomy and Physiology of the Nose and Paranasal Sinuses p.12
11. The osteomeatal unit is composed of the following except: ANSWER:
A. Ethmoidal bulla D. Middle turbinate
B. Uncinate process
Composed of:
C. Hiatus semilunaris
 Maxillary sinus ostia
D. Middle turbinate
 Anterior ethmoid cells and their ostia
 Ethmoid infundibulum
 Hiatus semilunaris
 Middle meatus
 Uncinate process

[ENT] T.09 – Anatomy and Physiology of the nose and Paranasal Sinuses p. 14
12. The primary drainage pathway of the maxillary sinus ANSWER:
A. Hiatus semilunaris A. Hiatus semilunaris
B. Uncinate process
Nasal Cavity Openings/Drainage: Middle meatus
C. Ethmoid infundibulum
 Middle ethmoidal air cells – bulla
D. Ethmoid bulla
 Ant. Ethmoidal air cells – Hiatus Semilunaris
 Max. Sinus – Hiatus Semilunaris
 Frontal Sinus – Infundibulum

[ENT] T.09 – Anatomy and Physiology of the nose and Paranasal Sinuses p. 9
13. Which of the following does NOT supply the medial and ANSWER:
lateral walls of the nasal cavity? D. Infraorbital artery
A. Sphenopalatine artery
Important arteries in the Kiesselbach area:
B. Anterior and posterior ethmoidal arteries
L – Superior Labial artery
C. Greater palatine artery E- anterior and posterior Ethmoidal artery
D. Infraorbital artery G – Greater palatine artery
S – Sphenopalatine artery

[ENT] T.09 – Anatomy and Physiology of the nose and Paranasal Sinuses p. 11
14. Depressor muscle ANSWER:
A. Procerus B. Alar nasi
B. Alar nasalis Depressor muscles:
C. Levator labii alaequae nasi  Alar nasi
 Depressor septi nasi
D. Anomalous nasi
Elevator muscles:
 Procerus
 Levator labii alaeque nasi
 Anomalous nasi

[ENT] T.09 – Anatomy and Physiology of the nose and Paranasal Sinuses p. 3

18 | 22 EARS, NOSE, AND THROAT Compiled Recalls (Finals) Editors | ENT TRANS TEAM
15. Boundaries of external nasal valve except: ANSWER:
A. Anterior head of inferior turbinate A. Anterior head of inferior turbinate
B. Nasal floor
External nasal valve:
C. Lateral crus
 Bounded by the lateral crus or the lower lateral cartilage laterally
D. Medial crus
 Medially, by the medial crus of the lateral cartilage
 Inferiorly, by the nasal floor
Internal nasal valve:
Bounded by the caudal border of the upper lateral cartilage, septum and
the inferior head of the inferior turbinate.

[ENT] T.09 – Anatomy and Physiology of the nose and Paranasal Sinuses p. 5
16. Crescent shaped bone curved downward/backwards ANSWER:
attached to the perpendicular process of the palatine bone C. Uncinate process
and the ethmoid process of the inferior turbinate:
Uncinate process:
A. Hiatus semilunaris
 Latin ―”processus uncinatus”—hook outgrowth
B. Ethmoid bulla
 Sharp ridge of bone sagittally oriented
C. Uncinate process  Crescent shaped bone curved downward/backwards
D. Ethmoid infundibulum  Attaches to perpendicular process of the palatine bone and the
ethmoid process of the inferior turbinate

[ENT] T.09 – Anatomy and Physiology of the Nose and Paranasal Sinuses p.12
17. Two-dimensional, sagittally oriented, crescent-shaped ANSWER:
cleft that represents the shortest distance between the free A. Hiatus semilunaris
posterior margin of the uncinate and the anterior surface of
Hiatus semilunaris inferioris:
the ethmoid bulla:
 Two-dimensional, sagittally oriented, Crescent-shaped cleft
A. Hiatus semilunaris
 Represents the shortest distance between the free posterior margin of
B. Ethmoid bulla the uncinated process and the anterior surface of the ethmoid bulla
C. Uncinate process  Passageway or ―” door” for gaining access to the ethmoid
D. Ethmoid infundibulum infundibulum.

[ENT] T.09 – Anatomy and Physiology of the Nose and Paranasal Sinuses p.12
18. Three-dimensional space bordered medially by the UP ANSWER:
and laterally by the LP with the maxillary ostium in its floor: D. Ethmoidal infundibulum
A. Hiatus semilunaris
Ethmoidal infundibulum
B. Ethmoid bulla
 3D space bordered medially by the uncinated process and laterally by
C. Uncinate process the lamina papyracea (LP) with the maxillary ostium in its floor
D. Ethmoid infundibulum  Lateral to LP

[ENT] T.09 – Anatomy and Physiology of the Nose and Paranasal Sinuses p.12
19. Posteroinferior portion of the nasal septum: ANSWER:
A. Perpendicular plate C. Vomer
B. Septal cartilage
 Vomer – posterior inferior part of the nasal septum
C. Vomer
 Perpendicular plate of ethmoid – posterior superior part of septum
D. Membranous septum
 Septal cartilage/Quadrangular cartilage – anterior part of septum

[ENT] T.09 – Anatomy and Physiology of the Nose and Paranasal Sinuses p.10
20. Antrum of Highmore: ANSWER:
A. Frontal sinus B. Maxillary sinus
B. Maxillary sinus
Maxillary sinus also known as Antrum of Highmore
C. Ethmoid sinus
D. Sphenoid sinus https://medicine.en-academic.com/77657/antrum_of_Highmore

19 | 22 EARS, NOSE, AND THROAT Compiled Recalls (Finals) Editors | ENT TRANS TEAM
R.12 (Diseases of the Nose, Paranasal sinuses, and Nasopharynx)
QUESTION ANSWER/RATIONALE
1. All of the following describes an angiofibroma except: ANSWER:
A. Occurs in male adolescents 1. B. Originates at the superior margin of the greater palatine foramen
B. Originates at the superior margin of the greater palatine
2. B. Anterior bowing of the posterior maxillary sinus wall
foramen
C. Can extend intracranially in about 10-36% 3. B. Surgical
D. Presents with unilateral nasal obstruction and epistaxis
2. Classic pathognomonic radiographic finding of 4. A. I
angiofibroma
A. Anterior bowing of the anterior maxillary sinus wall
B. Anterior bowing of the posterior maxillary sinus wall Juvenile angiofibroma
C. Posterior bowing of the anterior maxillary sinus wall  Histologically benign, locally aggressive
D. Posterior bowing of the posterior maxillary sinus wall  Male adolescents
 Origin: Superior margin of sphenopalatine foramen
 Intracranial extension 10%-36%
3. Mainstay treatment of angiofibroma:  Clinical Features:
A. Medical o Occurs in males 10-25 y.o.; Unilateral nasal obstruction, Epistaxis,
B. Surgical Facial swelling, Proptosis, Diplopia
C. Chemotherapy  Radiology: Classic radiographic feature
D. Radiotherapy o Anterior bowing of posterior maxillary sinus wall
4. Chandler classification of angiofibroma where in the tumor (Pathognomonic)
is confined to the nasopharyngeal vault:  Management: Surgery is the mainstay of treatment
A. I  Chandler classification
B. II o I – Tumor confine to nasopharyngeal vault
o II – Tumor extending to nasal cavity or sphenoid sinus
C. III o III – Tumor extending into antrum, ethmoid sinus, orbit
D. IV o IV – Intracranial tumor

[ENT] T.10 – Diseases of the Nose, Paranasal Sinuses, and Nasopharynx p.14-15
5. Which of the following is/are true about the acute bacterial ANSWER:
rhinosinusitis (ABRS) may be made in adults with: D. All of the above
A. Symptoms of a viral upper respiratory infection (URI) that
Acute Bacterial Rhinosinusitis
have not improved after 10 days or worsen after 5 to 10
 Symptoms of a viral upper respiratory infection (URI) that have not
days improved after 10 days or worsen after 5 to 10 days.
B. Symptoms may include nasal drainage, nasal congestion,  Some or all of the following symptoms: nasal drainage, nasal
facial pressure/pain, postnasal drainage, hyposmia/anosmia, congestion, facial pressure/pain, postnasal drainage,
fever, cough, fatigue, maxillary dental pain, and ear hyposmia/anosmia, fever, cough, fatigue, maxillary dental pain, and ear
pressure/fullness pressure/fullness.
C. Duration of symptoms is less than 12 weeks  European Position Paper on Rhinosinusitis and Nasal Polyps (EPOS) --
D. All of the above all cases lasting for < 12 weeks with complete resolution of symptoms
 Most common bacterial species:
o Streptococcus pneumoniae
o Haemophilus influenza
o Moraxella catarrhalis
 Other streptococcal spp, anaerobic bacteria
 Staphylococcus aureus

[ENT] T.10 – Diseases of the Nose, Paranasal Sinuses, and Nasopharynx p.7
6. Primary treatment of ABRS except: ANSWER:
A. Empiric antibiotic therapy D. Antihistamine
B. Nasal saline irrigation Management of ABRS
C. Intranasal corticosteroids  Empiric antibiotic therapy
 Nasal saline irrigation – adjunctive treatment
D. Antihistamine
 INCS (Intranasal corticosteroids)

[ENT] T.10 – Diseases of the Nose, Paranasal Sinuses, and Nasopharynx p.7
7. Work up for patients with ABRS with inadequate response ANSWER:
to treatment may include the following: D. All of the above
A. CT of the Paranasal Sinuses
Failure of 2nd line antibiotic treatment warrants further work-up
B. Sinus or meatal culture
 CT of the Paranasal Sinuses
C. Immune system studies
 Sinus or meatal culture
D. All of the above  Immune system studies

[ENT] T.10 – Diseases of the Nose, Paranasal Sinuses, and Nasopharynx p.7
8. Chronic rhinosinusitis (CRS) is defined as inflammation of ANSWER:
the nasal cavity and paranasal sinuses and/or the A. 12 weeks
underlying bone that has been present for at least:
Chronic rhinosinusitis
A. 12weeks
 Inflammation of the nasal cavity and paranasal
B. 13 weeks
 Sinuses and/or the underlying bone that has been present for at least
C. 14 weeks 12 weeks
D. 15weeks
[ENT] T.10 – Diseases of the Nose, Paranasal Sinuses, and Nasopharynx p. 13

20 | 22 EARS, NOSE, AND THROAT Compiled Recalls (Finals) Editors | ENT TRANS TEAM
9. What do you call when there is sudden deterioration of the ANSWER:
patient’s condition with either worsening of baseline A. Acute Exacerbation of CRS
symptoms or development of additional symptoms in
cases of CRS? Symptoms lasting for less than 4
Acute rhinosinusitis (ARS)
weeks with complete resolution
A. Acute exacerbation of CRS
duration between 4 and 12
B. Recurrent ABRS Subacute RS
weeks
C. Noth Symptoms lasting for more than
D. CRS Chronic RS (CRS) (with or
12 weeks without complete
without nasal polyps)
resolution of symptoms
≥ 4 episodes per year, each
lasting ≥ 7-10 days with
Recurrent ARS
complete resolution in between
episodes
sudden worsening of baseline
Acute exacerbation of CRS: CRS with return to baseline
after treatment

ENT] T.10 – Diseases of the Nose, Paranasal Sinuses, and Nasopharynx p. 6


10. CRS without nasal polyp, being an inflammatory disease, ANSWER:
should be primarily treated with: B. Intranasal corticosteroid
A. Antibiotic
Management of CRS without NP
B. Intranasal corticosteroid  INCS (intranasal corticosteroid) - primary treatment
C. Nasal saline irrigation  Nasal saline irrigation
D. Leukotriene receptor antagonist  Short-term antibiotics (less than 4 weeks; amoxicillin-clavulanic acid,
cefuroxime axetil and ciprofloxacin)
 Long-term: Low dose macrolide therapy (>12 weeks), may be given
together INCS especially when there is inadequate response in INCS
alone
 Surgery, if it does not improve after 2-3 months of INCS treatment

ENT] T.10 – Diseases of the Nose, Paranasal Sinuses, and Nasopharynx p. 13


111. Allergic rhinitis is treated based on the following: ANSWER:
A. severity of symptoms D. All of the above
B. frequency of symptoms
Based on severity and frequency, hence the ARIA classification: Mild,
C. co-morbidities
moderate, or severe / intermittent or persistent
D. all of the above

Based on comorbidities – No surgical for allergic rhinitis but is indicated in


the management of comorbid conditions

[ENT] T.10 - Diseases of the Nose, Paranasal sinuses, and Nasopharynx p.12
12. Theories of choanal atresia formation except: ANSWER:
A. Persistence of retropharyngeal membrane A. Persistence of retropharyngeal membrane
B. Abnormal persistence of bucconasal membrane
Four Basic Theories
C. Abnormal mesoderm forming adhesions in nasochoanal
 Persistence of buccopharyngeal membrane
region
 Abnormal persistence of bucconasal membrane
D. Misdirection of neural crest cell migration  Abnormal mesoderm forming adhesions in nasochoanal region
 Misdirection of neural crest cell migration

[ENT] T.10 - Diseases of the Nose, Paranasal sinuses, and Nasopharynx p.1
13. Paradoxical cyanosis: ANSWER:
A. Nasal glioma C. Choanal atresia
B. Nasal dermoid cyst
Choanal atresia presents with paradoxical cyanosis
C. Choanal atresia
 Cyanosis present at rest and improves with exertion
D. Encephalocele
 Opposite pattern relative to cyanosis with a cardiac cause

[ENT] T.10 - Diseases of the Nose, Paranasal sinuses, and Nasopharynx p.1

21 | 22 EARS, NOSE, AND THROAT Compiled Recalls (Finals) Editors | ENT TRANS TEAM
14. Mc Govern nipple: ANSWER:
A. Nasal glioma C. Choanal atresia
B. Nasal dermoid cyst
McGovern nipple: used for airway stabilization of Choanal atresia
C. Choanal atresia
D. Encephalocele [ENT] T.10 - Diseases of the Nose, Paranasal sinuses, and Nasopharynx p.1
15. Epithelium-lined cavities or sinus tracts filled with keratin ANSWER:
debris, hair follicles, sweat glands, and sebaceous glands: B. Nasal Dermoid Cyst
A. Nasal glioma
Nasal dermoid cyst
B. Nasal dermoid cyst  Epithelium-lined cavities or sinus tracts filled with keratin debris, hair
C. Choanal atresia follicles, sweat glands, and sebaceous glands
D. Encephalocele  Arise from ectodermal elements of fetal trilaminar septum which fail to
degenerate

[ENT] T.10 - Diseases of the Nose, Paranasal sinuses, and Nasopharynx p.2
16. This is due to abnormal closure of the fonticulus frontalis ANSWER:
which leads to an ectopic rest of glial tissue left A. Nasal glioma
extracranially:
Nasal glioma
A. Nasal glioma
 Rare congenital lesion composed of dysplastic glial cells which have lost
B. Nasal dermoid cyst their intracranial connections and present as an extra nasal or intranasal
C. Choanal atresia mass
D. Encephalocele  Abnormal closure of the fonticulus frontalis can lead to an ectopic
rest of glial tissue if left extracranially

[ENT] T.10 - Diseases of the Nose, Paranasal sinuses, and Nasopharynx p. 2


17. Most common site of PNS mucocele: ANSWER:
A. Maxillary B. Frontal
B. Frontal
Mucocele – Pathogenesis
C. Ethmoid
 Frontal sinus – most common site of occurrence
D. Sphenoid
 Least common site – sphenoid sinus

[ENT] T.10 - Diseases of the Nose, Paranasal sinuses, and Nasopharynx p. 7


18. Furunculosis: ANSWER:
A. Group A Beta hemolytic streptococcus B. Staphylococcus aureus
B. Staphylococcus aureus
Furunculosis:
C. Group Beta streptococcus
 Infection spreads to deeper tissues and forms a central core of purulent
D. Moraxella catarrhalis liquefaction (furuncle)
 Extensive and invasive infection of sebaceous glands or hair follicles
with some involvement of subcutaneous tissues
 CA: Staphylococcus aureus

[ENT] T.10 - Diseases of the Nose, Paranasal sinuses, and Nasopharynx p. 6


19. Acute fulminant fungal rhinosinusitis: ANSWER:
A. Acute invasive fungal RS A. Acute invasive fungal RS
B. Chronic invasive Fungal RS
Acute Invasive Fungal RS is also known as Acute fulminant fungal RS
C. Chronic granulomatous fungal RS
D. Allergic fungal RS [ENT] T.10 - Diseases of the Nose, Paranasal sinuses, and Nasopharynx p. 7
20. Charcot Leyden crystals: ANSWER:
A. Acute invasive fungal RS D. Allergic Fungal RS
B. Chronic invasive Fungal RS
Allergic Fungal RS - Criteria of Bent and Kuhn
C. Chronic granulomatous fungal RS
 Eosinophilic mucin (Charcot-Leyden crystals)
D. Allergic fungal RS
 Noninvasive fungal hyphae
 Nasal polyposis

[ENT] T.10 - Diseases of the Nose, Paranasal sinuses, and Nasopharynx p. 8

22 | 22 EARS, NOSE, AND THROAT Compiled Recalls (Finals) Editors | ENT TRANS TEAM
RECALLS: EARS, NOSE, AND THROAT
Q9 ● Dr. Nicanor Lacuesta ● May 26, 2022 ● 2nd Semester

Audiology
QUESTION ANSWER/RATIONALE
1. How is audiometric zero defined? ANSWER:
A. The total absence of sound D. The median average hearing threshold of young adults with no
B. The median average hearing threshold of the general history of hearing problem, ear infection or recent colds
population
Audiometric Zero (ANSI)
C. The median average hearing threshold of young children  The median average hearing threshold of young adults with no
with no history of ear infection or recent colds history of hearing problem, ear infection or recent colds.
D. The median average hearing threshold of young adults with  Each frequency has its separate zero with zero calibrated values built
no history of hearing problem, ear infection or recent colds into the audiometer.

[ENT] T.07 - Audiology p.5


2. What is the criteria of conductive hearing loss on an ANSWER:
audiometry? 2. C. When bone conduction thresholds are better than air
A. When bone conduction thresholds are the same as air conduction by 10 db or more & are normal
3. A. When bone conduction thresholds are the same as air
conduction & neither is normal
conduction & neither is normal
B. When bone conduction thresholds are reduced but are still 4. B. When bone conduction thresholds are reduced but are still
better than air conduction by 10 db or more better than air conduction by 10 db or more
C. When bone conduction thresholds are better than air
conduction by 10 db or more & are normal
D. None of the above Air conduction-Bone conduction Relationship
3. What is the criteria of sensorineural hearing loss on an A. When BC thresholds are better than air conduction by 10 db or
audiometry? more and are normal, the loss is conductive
A. When bone conduction thresholds are the same as air B. When BC thresholds are the same as air conduction and neither is
normal, the loss is sensorineural
conduction & neither is normal
B. When bone conduction thresholds are reduced but are still C. When BC thresholds are reduced but are still better than air
conduction by 10 db or more, the loss is mixed or combined
better than air conduction by 10 db or more
C. When bone conduction thresholds are better than air
conduction by 10 db or more & are normal
D. None of the above
4. What is the criteria of mixed type hearing loss on an
audiometry?
A. When bone conduction thresholds are the same as air
conduction & neither is normal
B. When bone conduction thresholds are reduced but are still
better than air conduction by 10 db or more
C. When bone conduction thresholds are better than air
conduction by 10 db or more & are normal
D. None of the above

[ENT] T.07 - Audiology p.5


5. What is the maximum decibels HTL for a normal hearing ANSWER:
threshold? D. 25 dB
A. 10 dB
B. 15 dB
C. 20 dB
D. 25 dB

 It is acceptable if the patient would hear deviation by 25 dB hearing


threshold.
 25-40 dB is mild hearing loss
 40-60 dB is moderate hearing loss
 60-80 dB is severe hearing loss
 >80 dB is profound hearing loss
[ENT] T.07 - Audiology p.5

1 | 3 EARS, NOSE, AND THROAT Recalls No. 9 Editors | ENT TRANS TEAM
6. What is the difference of Speech reception threshold (SRT) ANSWER:
and Speech discrimination threshold (SDT)? D. SRT uses spondee words
A. SRT has an expected score of 94 of 100%
 SDT has an expected score of 94 of 100%
B. SDT agrees closely with the average of pure tone at 500 Hz,
 SRT agrees closely with the average of pure tone at 500 Hz, 1000 Hz,
1000 Hz, and 2000 Hz and 2000 Hz
C. SRT is presented at 20 to 40 dB above SDT or the patient’s  SDT is presented at 20 to 40 dB above SDT or the patient’s most
most comfortable level comfortable level
D. SRT uses spondee words  SRT uses spondee words

[ENT] T.07 - Audiology p.7


7. Which of the following tests is considered to be the most ANSWER:
accurate measurement of auditory function and is B. Audiotory Braintem Evoked Response
completely objective?
Brainstem Evoked Response Audiometry (BERA)
A. Pure tone audiometry
 It measures the potential arising in the auditory nerve and brainstem
B. Auditory Brainstem evoked response structures with a latency of approximately up to 10msec.
C. Play audiometry  It is an accurate measure of the auditory function & is completely
D. Cardiovascular audiometry objective
o the golden standard for audiometry
o Results cannot be cheated because the electric signal movements
will be measured

[ENT] T.07 - Audiology p.8


8. This test is used to measure the compliance of the ANSWER:
tympanic membrane B. Tympanometry
A. Otoacoustic emission testing
Tympanometry
B. Tympanometry
 Indirect measure of the compliance (mobility) of the TM and the
C. Pure tone audiometry
ossicular chain system under conditions of positive, normal, and
D. Speech discrimination test
negative pressure

[ENT] T.07 - Audiology p.6


9. How many hours is the maximum amount of time allowed ANSWER:
for a continuous exposure to sounds with 90 dB without D. 8
incurring damage to the auditory system?
8 hours
A. 14
 The maximum exposure duration with 90 dB sound without incurring
B. 12 damage to the auditory system.
C. 10
D. 8
[ENT] T.07 - Audiology p.11
10. This part of pure tone audiometry is taken as a measure of ANSWER:
the integrity of acoustic nerve C. Both
A. Air conduction
Both Air conduction and Bone conduction
B. Bone conduction
 The pure tone audiometry converts electrical energy to either acoustic
C. Both or vibration type to assess hearing integrity. An oscillator is placed on
D. None of the above the mastoid area for bone conduction and earphones are placed on the
ears for acoustic output.

[ENT] T.07 - Audiology p.7


11. In Schwabach’s test, a diminished result has a hearing ANSWER:
loss that is described as B. Sensorineural
A. Conductive
B. Sensorineural Diminished Result in Schwabach’s Test
C. Mixed  Sensorineural
D. Normal  Locus is Cochlear/Retrocochlear

[ENT] T.07 - Audiology p.4


12-13. In Bing’s Test, a positive result can be interpreted as ANSWER:
A. Conductive hearing loss B. Normal hearing
B. Normal hearing
C. Mixed hearing Positive Result in Bing’s Test
D. None of the above  Increase in loudness/Decrease in loudness
 Normal/ SN
 Locus is None/ Cochlear/Retrocochlear

[ENT] T.07 - Audiology p.4


14. In Bing’s Test, a negative result can be interpreted as ANSWER:
A. Conductive hearing loss A. Conductive hearing loss
B. Normal hearing
Negative Result in Bing’s Test
C. Mixed hearing
 Conductive
D. None of the above
 Locus is EAC/ME

[ENT] T.07 - Audiology p.4

2 | 3 EARS, NOSE, AND THROAT Recalls No. 9 Editors | ENT TRANS TEAM
15. This tuning for test compares the patient’s air conduction ANSWER:
with the bone conduction A. Rinne’s Test
A. Rinne’s Rinne’s Test
 Compare’s patient’s hearing by bone conduction against air
B. Webber’s
conduction (AC vs BC). Weber’s Test on the other hand measures
C. Schwabach’s lateralization of hearingby bone conduction against air conduction (AC
D. Bing’s vs BC).
 Weber’s Test on the other hand measures lateralization of hearing.
 Schwabach’s Test compares patients bone conduction against normal
reference.
 Bing’s Test in an application of an occlusion test.
[ENT] T.07 - Audiology p.7-8

3 | 3 EARS, NOSE, AND THROAT Recalls No. 9 Editors | ENT TRANS TEAM
RECALLS: EARS, NOSE, AND THROAT
Q10 ● Dr. Nicanor Lacuesta ● May 26, 2022 ● 2nd Semester

Anatomy, Physiology, and Diseases of the Oral Cavity, Oropharynx, and Hypopharynx
QUESTION ANSWER/RATIONALE
1. Which is not a boundary of the oropharynx? ANSWER:
A. Tongue base C. Anterior Faucial Pillars
B. 2nd and 3rd cervical vertebra Anterior Faucial Pillar
C. Anterior faucial pillars  The anterior facial Pillar or the Palatoglossal Arch formed by the
palatoglossus is the posterior border of the Oral Cavity.
D. Posterior faucial pillars
The Boundary of Oropharynx include:
 Anterior: Tongue base and lingual tonsils
 Posterior: 2nd and 3rd cervical vertebrae with prevertebral fascia
 Lateral: Faucial Pillars with flank of the palatine tonsils.

[ENT] T.15 - Anatomy, Physiology, and Disease of the Oral Cavity p.3
[ENT] T.16 - Anatomy, Physiology, and Disease of the Oropharynx and Hypopharynx p.1
2. Majority of the oropharynx and hypopharynx is lined with ANSWER:
what type of epithelium? B. Stratified, nonkeratinized squamous epithelium
A. Ciliated pseudostratified columnar epithelium
Mucosal lining of the oropharynx and hypopharynx:
B. Nonkeratinized stratified squamous epithelium
Stratified, nonkeratinized squamous epithelium
C. Keratinized stratified squamous epithelium
D. Simple columnar epithelium [ENT] T.16 – Anatomy, Physiology, and Diseases of the Oropharynx and Hypopharynx p.1
3. What is the superior border of the hypopharynx? ANSWER:
A. Hyoid bone A. Hyoid bone
B. Superior border of the thyroid cartilage Borders of the hypopharynx
C. Superior border of the epiglottis  Superior border: hyoid bone, upper esophageal sphincter (UES)
 Inferior border: cricopharyngeus
D. Base of the tongue
 Anterior border: epiglottis, paired aryepiglottic folds and arytenoid
cartilage

Throat Anatomy. By Tjoson Tjoa, Department of Otolaryngology-Head and Neck Surgery, University
of California, Irvine, School of Medicine. July 10, 2013
4. Which of the following is not true about the pharyngeal ANSWER:
musculature? D. They are all overlapping
A. There is a gap between each muscle
Only the constrictor pharyngis medius and inferior muscles overlap
B. The inferior most fibers joins with the esophagus
C. Almost all fibers run an oblique direction
D. They are all overlapping [ENT] T.16 – Anatomy, Physiology, and Diseases of the Oropharynx and Hypopharynx p.2
4. Which portion would have fibers of the pharyngeal muscles ANSWER:
would have fibers that run a transverse direction? C. Constrictor inferioris
A. Constrictor superioris
Killian–Jamieson region: between the oblique and transverse fibers of
B. Constrictor medius
the constrictor pharyngis.
C. Constrictor inferioris
D. None of the above

[ENT] T.16 – Anatomy, Physiology, and Diseases of the Oropharynx and Hypopharynx p.2
6. This weak point of the pharyngeal musculature occurs ANSWER:
between the constrictor pharyngis inferior and the A. Killian triangle
uppermost fibers of the cricopharyngeus muscle.
Killian triangle is between the constrictor pharyngis inferior and the
A. Killian triangle
uppermost fibers of the cricopharyngeus muscle.
B. Killian-Jamieson triangle
C. Lamier’s triangle
D. Killian-Jamieson region [ENT] T.16 – Anatomy, Physiology, and Diseases of the Oropharynx and Hypopharynx p.2
7. This weak point of the pharyngeal musculature occurs ANSWER:
between the oblique and transverse fibers of the D. Killian-Jamieson region
constrictor pharynges.
Killian–Jamieson region: between the oblique and transverse fibers of
A. Killian triangle
the constrictor pharynges.
B. Killian-Jamieson triangle
C. Lamier’s triangle [ENT] T.16 – Anatomy, Physiology, and Diseases of the Oropharynx and Hypopharynx p.2
D. Killian-Jamieson region

1 | 3 EARS, NOSE, AND THROAT Recalls No. 10 Editors | ENT TRANS TEAM
8. This weak point of the pharyngeal musculature is bounded ANSWER:
above by the cricopharyngeus and below by the C. Lamier’s triangle
uppermost fibers of the esophageal musculature.
Laimer triangle, which is bounded above by the cricopharyngeus and
A. Killian triangle
below by the uppermost fibers of the esophageal musculature.
B. Killian-Jamieson triangle
C. Lamier’s triangle
D. Killian-Jamieson region [ENT] T.16 – Anatomy, Physiology, and Diseases of the Oropharynx and Hypopharynx p.2
9. This weak point of the pharyngeal musculature is the most ANSWER:
common area of occurrence of hypopharyngeal diverticula A. Killian triangle
A. Killian triangle
Killian triangle: between the constrictor pharyngis inferior and the
B. Killian-Jamieson triangle
uppermost fibers of the cricopharyngeus muscle - common site for the
C. Lamier’s triangle formation of hypopharyngeal diverticula.
D. Killian-Jamieson region
[ENT] T.16 – Anatomy, Physiology, and Diseases of the Oropharynx and Hypopharynx p.2
10. Which of the tonsils is covered with stratified squamous ANSWER:
epithelium? A or D
A. Palatine tonsils
Palatine and lingual tonsils are covered by stratified, nonkeratinized
B. Tubulopharyngeal tonsils
squamous epithelium
C. Pharyngeal tonsils
D. Lingual tonsils [ENT] T.16 – Anatomy, Physiology, and Diseases of the Oropharynx and Hypopharynx p.2
11. Up to what age are your palatine tonsil have pronounced ANSWER:
activity? B. 8-10 years old
A. 6-8 years old
Activity pronounced during childhood until 8-10 years of age, when
B. 8-10 years old
immunologic challenges from the environment induce hyperplasia of the
C. 10-12 years old palatine tonsils.
D. 12-14 years old
[ENT] T.16 – Anatomy, Physiology, and Diseases of the Oropharynx and Hypopharynx p.3
12. In doing a chest radiogram in a patient who ingested ANSWER:
caustic fluids, what are we actually investigating? B. Esophageal perforation
A. Aspiration pneumonia
Chest radiograph must be done to check for mediastinal widening due to
B. Esophageal perforation
esophageal perforation.
C. Esophageal stenosis
D. Extent of mucosal erosion [ENT] T.16 – Anatomy, Physiology, and Diseases of the Oropharynx and Hypopharynx p.4
13. What is the initial appearance of mucosa injured by a ANSWER:
scalding or caustic fluids? C. Erythema
A. Blister formation
Diagnosis – Acute evaluation
B. Ulceration
Begins with a mirror examination of the oral cavity, oropharynx,
C. Erythema hypopharynx, and larynx - mucosa initially appears erythematous and
D. Perforation edematous and later may show epithelial defects and a whitish fibrin
coating.

[ENT] T.16 – Anatomy, Physiology, and Diseases of the Oropharynx and Hypopharynx p.9
14. Which of the following is true about rapid streptococcal ANSWER:
tests? C. Asymptomatic patients with positive test should
A. Results can be done in one hour not be treated with antibiotics
B. Has a sensitivity of 75%
Rapid Streptococcal test
C. Asymptomatic patients with positive test should not be
treated with antibiotics
• Makes use of colloid-labeled specific antibodies which are placed onto
reaction strips along with the pharyngeal smear
D. Streptococcal angina is ruled out if the test is negative in
• Sensitivity of 80%to 90% making them useful tools in deciding whether to
symptomatic patients administer antibiotics
• Should be correlated to clinical findings
• Asymptomatic patients with a positive rapid test should not be
placed on antibiotics
• Conversely, a culture should be taken in cases where there is clinical
suspicion of streptococcal tonsillitis but the rapid test is negative

[ENT] T.16 - Anatomy, Physiology, and Diseases of the Oropharynx and Hypopharynx p. 4
15. This acute inflammatory condition of the pharynx is ANSWER:
characterized by rash formation over the entire body with D. Scarlet Fever
perioral pallor
Scarlet fever – Clinical presentation
A. Streptococcal angina
B. Plaut-Vincent Angina • Rash that begins on the trunk with sparing of the area around the mouth
is spared (“perioral pallor”)
C. Diphtheria
D. Scarlet fever • Pathognomonic feature is a bright red tongue with a glistening surface
and hyperplastic papillae (“raspberry tongue,”)
• The tonsils are greatly swollen with a deep red color
• Occasionally there is an enanthema of the soft palate with
hemorrhagic areas

[ENT] T.16 - Anatomy, Physiology, and Diseases of the Oropharynx and Hypopharynx p. 5

2 | 3 EARS, NOSE, AND THROAT Recalls No. 10 Editors | ENT TRANS TEAM
16. This acute inflammatory condition of the pharynx is ANSWER:
caused by fusiform rods and spirochetes B. Plaut-Vincent Angina
A. Streptococcal angina
Plaut – Vincent Angina
B. Plaut-Vincent Angina
C. Diphtheria • Inflammatory disease is caused by fusiform rods and spirochetes
D. Scarlet fever • Unilateral dysphagia and a fetid breath odor with very little malaise
• The causative organisms can be detected by the direct microscopic
examination of a gram-stained smear

[ENT] T.16 - Anatomy, Physiology, and Diseases of the Oropharynx and Hypopharynx p. 5
17. This acute inflammation of the pharynx is caused by a ANSWER:
bacteria that produces a toxin that causes cell necrosis C. Diphtheria
and ulcerations
Diphtheria
A. Streptococcal angina
B. Plaut-Vincent Angina • Caused by Corynebacterium diphtheriae, transmitted by droplet inhalation
or skin-to-skin contact with an incubation period of 1–5 days
C. Diphtheria
D. Scarlet fever • Bacterium produces a special endotoxin that causes epithelial cell
necrosis and ulcerations

[ENT] T.16 - Anatomy, Physiology, and Diseases of the Oropharynx and Hypopharynx p. 5
18. Which of these following is not a measure to treat ANSWER:
peripheral obstructive sleep apnea syndrome? D. Use of muscle stimulants to increase muscle tone during sleep
A. Continuous positive air pressure masks
Peripheral Obstructive Sleep Apnea Syndrome – Treatment
B. Uvulopharyngoplastry
C. Weight reduction • General measures: weight reduction, abstinence from
alcohol and nicotine, and avoiding big meals, and avoid the use of
D. Use of muscle stimulants to increase muscle tone during
sedatives
sleep
• Esmarch splint (a mandibular advancement device), an occlusive splint
that advances the lower jaw
• Continuous positive airway pressure (CPAP) mask
• Uvulopalatopharyngoplasty (UPPP) with tonsillectomy

[ENT] T.16 - Anatomy, Physiology, and Diseases of the Oropharynx and Hypopharynx p. 8
19. Diverticula in the hypopharynx is usually of what type ANSWER:
A. Traction diverticulum C. Both can occur in the hypopharynx
B. Pulsion diverticulum
Diverticula – Two distinct types:
C. Both can occur in the hypopharynx
D. None of the above • Pulsion Diverticula - Mucosa herniates through a weak point in the
muscular coat due to a rise of intraluminal pressure
• Traction diverticula - Which usually form at parabronchial sites due to
scar traction following hilar lymphadenitis and involve all layers of the
esophageal wall

[ENT] T.16 - Anatomy, Physiology, and Diseases of the Oropharynx and Hypopharynx p10
20. In accidental foreign body ingestion, where are they ANSWER:
usually embedded/found? C. Palatine tonsil
A. Pyriform sinus
Foreign bodies are most commonly located in the tonsils and at the
B. Vallecula
tongue base. Foreign bodies typically become lodged in the hypopharynx
C. Palatine tonsil or in the upper constriction of the esophagus.
D. Vestibular folds
[ENT] T.16 – Anatomy, Physiology, Diseases of the Oropharynx and Hypopharynx p.4, 10

3 | 3 EARS, NOSE, AND THROAT Recalls No. 10 Editors | ENT TRANS TEAM
RECALLS: EARS, NOSE, AND THROAT
Q11 ● Dr. Jason Cabrera ● May 29, 2022 ● 2nd Semester

Anatomy and Physiology of the Nose and Paranasal sinuses


QUESTION ANSWER/RATIONALE
1. The following paranasal sinuses drain into the middle ANSWER:
meatus, except: B. Posterior ethmoid
A. Frontal sinus Middle Meatus:
B. Posterior ethmoid  Middle ethmoidal air cells – bulla
 Anterior Ethmoidal air cells – Hiatus Semilunaris
C. Anterior ethmoid
 Maxillary Sinus – Hiatus Semilunaris
D. Maxillary sinus
 Frontal Sinus – Infundibulum
Superior Meatus: Posterior Ethmoidal air cells

[ENT] T.09 – Anatomy and Physiology of the Nose and Paranasal Sinuses p.9.
2. Which of the following drains into the inferior meatus: ANSWER:
A. Sphenoid Sinus C. Nasolacrimal duct
B. Posterior ethmoids
Inferior Meatus: Opening of Nasolacrimal duct
C. Nasolacrimal duct
D. None of the above [ENT] T.09 – Anatomy and Physiology of the Nose and Paranasal Sinuses p.9.
3. The following are components of the nasal septum, except: ANSWER:
A. Vomer D. Cribriform plate of the ethmoid
B. Maxillary crest
Bony components of the septum:
C. Perpendicular plate of the ethmoid
 Nasal crest of the palatine bone
D. Cribriform plate of the ethmoid
 Nasal crest of the maxilla and premaxilla
 Vomer
 Perpendicular plate of the ethmoid
 Nasal crest of the frontal bone
 Spine of the paired nasal bones

[ENT] T.09 – Anatomy and Physiology of the Nose and Paranasal Sinuses p.10.
4. Which of the paranasal sinuses is the only sinus routinely ANSWER:
present at birth. B. Maxillary sinus
A. Anterior ethmoid
The maxillary sinus is present at birth but remains very small until the
B. Maxillary sinus second dentition, because the presence of tooth germs in the maxilla limit
C. Frontal sinus the extent of the sinuses.
D. Posterior ethmoid
Probst R, Grevers G, Iro H. (2017) Basic Otolaryngology: A Step-by- Guide 2 nd Ed. Stuttgard:
Thieme chapter 1 (Anatomy and physiology of the nose, paranasal sinuses and face) page 4
5. All of the following are part of the lateral border of the ANSWER:
nasal cavity except: D. None of the above
A. Turbinates
Lateral wall of the nasal cavity
B. Horizontal plate of palatine bone
 Nasal bone
C. Maxillary bone
 Maxillary
D. None of the above  Inferior concha/turbinate
 Palatine
 Ethmoid
 Sphenoid

[ENT] T.09 – Anatomy and Physiology of the Nose and Paranasal Sinuses p.6.
6. The largest turbinate/conchae ANSWER:
A. Inferior A. Inferior
B. Middle Inferior Turbinate and Meatus:
C. Superior  Largest turbinate
D. Supreme  Separate bone covered by thick mucous membrane

[ENT] T.09 – Anatomy and Physiology of the Nose and Paranasal Sinuses p.7
7. The middle meatus contains all of the following except: ANSWER:
A. Ethmoid bulla D. Sphenoethmoidal rescess
B. Uncinate process
Rest of middle meatus posteroinferiorly contains:
C. Hiatus semilunaris
 Ethmoidal bulla
D. Sphenoethmoidal recess
 Uncinate Process
 Semilunar hiatus

[ENT] T.09 – Anatomy and Physiology of the Nose and Paranasal Sinuses p.8
8. Which of the following parts of the nasal septum unites ANSWER:
with the cribriform plate B. Perpendicular Plate of Ethmoid
A. Vomer
Perpendicular Plate of Ethmoid
B. Perpendicular plate of ethmoid
 Forms upper 1/3 or more of the nasal septum
C. Quadrangular cartilage
 Unites superiorly with cribriform plate of the ethmoid
D. Crest of maxillary bone

[ENT] T.09 – Anatomy and Physiology of the Nose and Paranasal Sinuses p.10

1 | 3 EARS, NOSE, AND THROAT Recalls No. 11 Editors | ENT TRANS TEAM
9. All of the following are part of kiesselbach plexus except: ANSWER:
A. Ethmoidal artery C. Lesser palatine artery
B. Superior labial artery
Arteries important in the Kieselbach area: LEGS
C. Lesser palatine artery  L- superior Labial artery
D. Sphenopalatine artery  E- anterior and posterior Ethmoidal artery
 G- Greater palatine artery
 S- Sphenopalatine artery

[ENT] T.09 – Anatomy and Physiology of the Nose and Paranasal Sinuses p.11
10. The space between the bulla ethmoidalis and uncinate ANSWER:
process is called: D. Hiatus Semilunaris
A. Agger nasi Hiatus semilunaris inferioris
 The gap between the ethmoid bulla and the free edge of the uncinate
B. Olfactory cleft
process defines the hiatus semilunaris
C. Frontonasal duct
D. Hiatus semilunaris [ENT] T.09 – Anatomy and Physiology of the Nose and Paranasal Sinuses p.12
11. The osteomeatal unit is composed of the following except: ANSWER:
A. Ethmoidal bulla D. Middle turbinate
B. Uncinate process Composed of:
 Maxillary sinus ostia
C. Hiatus semilunaris
 Anterior ethmoid cells and their ostia
D. Middle turbinate
 Ethmoid infundibulum
 Hiatus semilunaris
 Middle meatus
 Uncinate process

[ENT] T.09 – Anatomy and Physiology of the nose and Paranasal Sinuses p. 14
12. The primary drainage pathway of the maxillary sinus ANSWER:
A. Hiatus semilunaris A. Hiatus semilunaris
B. Uncinate process
Nasal Cavity Openings/Drainage: Middle meatus
C. Ethmoid infundibulum
 Middle ethmoidal air cells – bulla
D. Ethmoid bulla
 Ant. Ethmoidal air cells – Hiatus Semilunaris
 Max. Sinus – Hiatus Semilunaris
 Frontal Sinus – Infundibulum

[ENT] T.09 – Anatomy and Physiology of the nose and Paranasal Sinuses p. 9
13. Which of the following does NOT supply the medial and ANSWER:
lateral walls of the nasal cavity? D. Infraorbital artery
A. Sphenopalatine artery
Important arteries in the Kiesselbach area:
B. Anterior and posterior ethmoidal arteries
L – Superior Labial artery
C. Greater palatine artery E- anterior and posterior Ethmoidal artery
D. Infraorbital artery G – Greater palatine artery
S – Sphenopalatine artery

[ENT] T.09 – Anatomy and Physiology of the nose and Paranasal Sinuses p. 11
14. Depressor muscle ANSWER:
A. Procerus B. Alar nasi
B. Alar nasalis
Depressor muscles: Elevator muscles:
C. Levator labii alaequae nasi
 Alar nasi  Procerus
D. Anomalous nasi
 Depressor septi nasi  Levator labii alaeque nasi
 Anomalous nasi

[ENT] T.09 – Anatomy and Physiology of the nose and Paranasal Sinuses p. 3
15. Boundaries of external nasal valve except: ANSWER:
A. Anterior head of inferior turbinate A. Anterior head of inferior turbinate
B. Nasal floor
External nasal valve:
C. Lateral crus
 Bounded by the lateral crus or the lower lateral cartilage laterally
D. Medial crus
 Medially, by the medial crus of the lateral cartilage
 Inferiorly, by the nasal floor
Internal nasal valve:
Bounded by the caudal border of the upper lateral cartilage, septum and
the inferior head of the inferior turbinate.

[ENT] T.09 – Anatomy and Physiology of the nose and Paranasal Sinuses p. 5
16. Crescent shaped bone curved downward/backwards ANSWER:
attached to the perpendicular process of the palatine bone C. Uncinate process
and the ethmoid process of the inferior turbinate: Uncinate process:
A. Hiatus semilunaris  Latin ―”processus uncinatus”—hook outgrowth
 Sharp ridge of bone sagittally oriented
B. Ethmoid bulla
 Crescent shaped bone curved downward/backwards
C. Uncinate process
 Attaches to perpendicular process of the palatine bone and the
D. Ethmoid infundibulum ethmoid process of the inferior turbinate

[ENT] T.09 – Anatomy and Physiology of the Nose and Paranasal Sinuses p.12

2 | 3 EARS, NOSE, AND THROAT Recalls No. 11 Editors | ENT TRANS TEAM
17. Two-dimensional, sagittally oriented, crescent-shaped ANSWER:
cleft that represents the shortest distance between the free A. Hiatus semilunaris
posterior margin of the uncinate and the anterior surface of
Hiatus semilunaris inferioris:
the ethmoid bulla:
 Two-dimensional, sagittally oriented, Crescent-shaped cleft
A. Hiatus semilunaris
 Represents the shortest distance between the free posterior margin of
B. Ethmoid bulla the uncinated process and the anterior surface of the ethmoid bulla
C. Uncinate process  Passageway or ―” door” for gaining access to the ethmoid
D. Ethmoid infundibulum infundibulum.

[ENT] T.09 – Anatomy and Physiology of the Nose and Paranasal Sinuses p.12
18. Three-dimensional space bordered medially by the UP ANSWER:
and laterally by the LP with the maxillary ostium in its floor: D. Ethmoidal infundibulum
A. Hiatus semilunaris
Ethmoidal infundibulum
B. Ethmoid bulla
 3D space bordered medially by the uncinated process and laterally by
C. Uncinate process the lamina papyracea (LP) with the maxillary ostium in its floor
D. Ethmoid infundibulum  Lateral to LP

[ENT] T.09 – Anatomy and Physiology of the Nose and Paranasal Sinuses p.12
19. Posteroinferior portion of the nasal septum: ANSWER:
A. Perpendicular plate C. Vomer
B. Septal cartilage
C. Vomer  Vomer – posterior inferior part of the nasal septum
 Perpendicular plate of ethmoid – posterior superior part of septum
D. Membranous septum
 Septal cartilage/Quadrangular cartilage – anterior part of septum

[ENT] T.09 – Anatomy and Physiology of the Nose and Paranasal Sinuses p.10
20. Antrum of Highmore: ANSWER:
A. Frontal sinus B. Maxillary sinus
B. Maxillary sinus
Maxillary sinus also known as Antrum of Highmore
C. Ethmoid sinus
D. Sphenoid sinus https://medicine.en-academic.com/77657/antrum_of_Highmore

3 | 3 EARS, NOSE, AND THROAT Recalls No. 11 Editors | ENT TRANS TEAM
RECALLS: EARS, NOSE, AND THROAT
Q12 ● Dr. Jason Cabrera ● May 29, 2022 ● 2nd Semester

Diseases of the Nose, Paranasal sinuses, and Nasopharynx


QUESTION ANSWER/RATIONALE
1. All of the following describes an angiofibroma except: ANSWER:
A. Occurs in male adolescents 1. B. Originates at the superior margin of the greater palatine foramen
B. Originates at the superior margin of the greater palatine
2. B. Anterior bowing of the posterior maxillary sinus wall
foramen
C. Can extend intracranially in about 10-36% 3. B. Surgical
D. Presents with unilateral nasal obstruction and epistaxis
2. Classic pathognomonic radiographic finding of 4. A. I
angiofibroma
A. Anterior bowing of the anterior maxillary sinus wall
B. Anterior bowing of the posterior maxillary sinus wall Juvenile angiofibroma
C. Posterior bowing of the anterior maxillary sinus wall  Histologically benign, locally aggressive
D. Posterior bowing of the posterior maxillary sinus wall  Male adolescents
 Origin: Superior margin of sphenopalatine foramen
 Intracranial extension 10%-36%
3. Mainstay treatment of angiofibroma:  Clinical Features:
A. Medical o Occurs in males 10-25 y.o.; Unilateral nasal obstruction, Epistaxis,
B. Surgical Facial swelling, Proptosis, Diplopia
C. Chemotherapy  Radiology: Classic radiographic feature
D. Radiotherapy o Anterior bowing of posterior maxillary sinus wall
4. Chandler classification of angiofibroma where in the tumor (Pathognomonic)
is confined to the nasopharyngeal vault:  Management: Surgery is the mainstay of treatment
A. I  Chandler classification
o I – Tumor confine to nasopharyngeal vault
B. II
o II – Tumor extending to nasal cavity or sphenoid sinus
C. III o III – Tumor extending into antrum, ethmoid sinus, orbit
D. IV o IV – Intracranial tumor

[ENT] T.10 – Diseases of the Nose, Paranasal Sinuses, and Nasopharynx p.14-15
5. Which of the following is/are true about the acute bacterial ANSWER:
rhinosinusitis (ABRS) may be made in adults with: D. All of the above
A. Symptoms of a viral upper respiratory infection (URI) that
Acute Bacterial Rhinosinusitis
have not improved after 10 days or worsen after 5 to 10
 Symptoms of a viral upper respiratory infection (URI) that have not
days improved after 10 days or worsen after 5 to 10 days.
B. Symptoms may include nasal drainage, nasal congestion,  Some or all of the following symptoms: nasal drainage, nasal
facial pressure/pain, postnasal drainage, hyposmia/anosmia, congestion, facial pressure/pain, postnasal drainage,
fever, cough, fatigue, maxillary dental pain, and ear hyposmia/anosmia, fever, cough, fatigue, maxillary dental pain, and ear
pressure/fullness pressure/fullness.
C. Duration of symptoms is less than 12 weeks  European Position Paper on Rhinosinusitis and Nasal Polyps (EPOS) --
D. All of the above all cases lasting for < 12 weeks with complete resolution of symptoms
 Most common bacterial species:
o Streptococcus pneumoniae
o Haemophilus influenza
o Moraxella catarrhalis
 Other streptococcal spp, anaerobic bacteria
 Staphylococcus aureus

[ENT] T.10 – Diseases of the Nose, Paranasal Sinuses, and Nasopharynx p.7
6. Primary treatment of ABRS except: ANSWER:
A. Empiric antibiotic therapy D. Antihistamine
B. Nasal saline irrigation Management of ABRS
C. Intranasal corticosteroids  Empiric antibiotic therapy
 Nasal saline irrigation – adjunctive treatment
D. Antihistamine
 INCS (Intranasal corticosteroids)

[ENT] T.10 – Diseases of the Nose, Paranasal Sinuses, and Nasopharynx p.7
7. Work up for patients with ABRS with inadequate response ANSWER:
to treatment may include the following: D. All of the above
A. CT of the Paranasal Sinuses
Failure of 2nd line antibiotic treatment warrants further work-up
B. Sinus or meatal culture
 CT of the Paranasal Sinuses
C. Immune system studies
 Sinus or meatal culture
D. All of the above  Immune system studies
[ENT] T.10 – Diseases of the Nose, Paranasal Sinuses, and Nasopharynx p.7
8. Chronic rhinosinusitis (CRS) is defined as inflammation of ANSWER:
the nasal cavity and paranasal sinuses and/or the A. 12 weeks
underlying bone that has been present for at least: Chronic rhinosinusitis
 Inflammation of the nasal cavity and paranasal
A. 12weeks
 Sinuses and/or the underlying bone that has been present for at least
B. 13 weeks
12 weeks
C. 14 weeks
D. 15weeks [ENT] T.10 – Diseases of the Nose, Paranasal Sinuses, and Nasopharynx p. 13

1 | 3 EARS, NOSE, AND THROAT Recalls No. 12 Editors | ENT TRANS TEAM
9. What do you call when there is sudden deterioration of the ANSWER:
patient’s condition with either worsening of baseline A. Acute Exacerbation of CRS
symptoms or development of additional symptoms in
cases of CRS? Symptoms lasting for less than 4
Acute rhinosinusitis (ARS)
weeks with complete resolution
A. Acute exacerbation of CRS
duration between 4 and 12
B. Recurrent ABRS Subacute RS
weeks
C. Noth Symptoms lasting for more than
D. CRS Chronic RS (CRS) (with or
12 weeks without complete
without nasal polyps)
resolution of symptoms
≥ 4 episodes per year, each
lasting ≥ 7-10 days with
Recurrent ARS
complete resolution in between
episodes
sudden worsening of baseline
Acute exacerbation of CRS: CRS with return to baseline
after treatment

ENT] T.10 – Diseases of the Nose, Paranasal Sinuses, and Nasopharynx p. 6


10. CRS without nasal polyp, being an inflammatory disease, ANSWER:
should be primarily treated with: B. Intranasal corticosteroid
A. Antibiotic
Management of CRS without NP
B. Intranasal corticosteroid  INCS (intranasal corticosteroid) - primary treatment
C. Nasal saline irrigation  Nasal saline irrigation
D. Leukotriene receptor antagonist  Short-term antibiotics (less than 4 weeks; amoxicillin-clavulanic acid,
cefuroxime axetil and ciprofloxacin)
 Long-term: Low dose macrolide therapy (>12 weeks), may be given
together INCS especially when there is inadequate response in INCS
alone
 Surgery, if it does not improve after 2-3 months of INCS treatment

ENT] T.10 – Diseases of the Nose, Paranasal Sinuses, and Nasopharynx p. 13


111. Allergic rhinitis is treated based on the following: ANSWER:
A. severity of symptoms D. All of the above
B. frequency of symptoms
Based on severity and frequency, hence the ARIA classification: Mild,
C. co-morbidities
moderate, or severe / intermittent or persistent
D. all of the above

Based on comorbidities – No surgical for allergic rhinitis but is indicated in


the management of comorbid conditions

[ENT] T.10 - Diseases of the Nose, Paranasal sinuses, and Nasopharynx p.12
12. Theories of choanal atresia formation except: ANSWER:
A. Persistence of retropharyngeal membrane A. Persistence of retropharyngeal membrane
B. Abnormal persistence of bucconasal membrane
Four Basic Theories
C. Abnormal mesoderm forming adhesions in nasochoanal
 Persistence of buccopharyngeal membrane
region
 Abnormal persistence of bucconasal membrane
D. Misdirection of neural crest cell migration  Abnormal mesoderm forming adhesions in nasochoanal region
 Misdirection of neural crest cell migration

[ENT] T.10 - Diseases of the Nose, Paranasal sinuses, and Nasopharynx p.1
13. Paradoxical cyanosis: ANSWER:
A. Nasal glioma C. Choanal atresia
B. Nasal dermoid cyst
Choanal atresia presents with paradoxical cyanosis
C. Choanal atresia
 Cyanosis present at rest and improves with exertion
D. Encephalocele
 Opposite pattern relative to cyanosis with a cardiac cause

[ENT] T.10 - Diseases of the Nose, Paranasal sinuses, and Nasopharynx p.1

2 | 3 EARS, NOSE, AND THROAT Recalls No. 12 Editors | ENT TRANS TEAM
14. Mc Govern nipple: ANSWER:
A. Nasal glioma C. Choanal atresia
B. Nasal dermoid cyst
McGovern nipple: used for airway stabilization of Choanal atresia
C. Choanal atresia
D. Encephalocele [ENT] T.10 - Diseases of the Nose, Paranasal sinuses, and Nasopharynx p.1
15. Epithelium-lined cavities or sinus tracts filled with keratin ANSWER:
debris, hair follicles, sweat glands, and sebaceous glands: B. Nasal Dermoid Cyst
A. Nasal glioma
Nasal dermoid cyst
B. Nasal dermoid cyst  Epithelium-lined cavities or sinus tracts filled with keratin debris, hair
C. Choanal atresia follicles, sweat glands, and sebaceous glands
D. Encephalocele  Arise from ectodermal elements of fetal trilaminar septum which fail to
degenerate

[ENT] T.10 - Diseases of the Nose, Paranasal sinuses, and Nasopharynx p.2
16. This is due to abnormal closure of the fonticulus frontalis ANSWER:
which leads to an ectopic rest of glial tissue left A. Nasal glioma
extracranially:
Nasal glioma
A. Nasal glioma
 Rare congenital lesion composed of dysplastic glial cells which have lost
B. Nasal dermoid cyst their intracranial connections and present as an extra nasal or intranasal
C. Choanal atresia mass
D. Encephalocele  Abnormal closure of the fonticulus frontalis can lead to an ectopic
rest of glial tissue if left extracranially

[ENT] T.10 - Diseases of the Nose, Paranasal sinuses, and Nasopharynx p. 2


17. Most common site of PNS mucocele: ANSWER:
A. Maxillary B. Frontal
B. Frontal
Mucocele – Pathogenesis
C. Ethmoid
 Frontal sinus – most common site of occurrence
D. Sphenoid
 Least common site – sphenoid sinus

[ENT] T.10 - Diseases of the Nose, Paranasal sinuses, and Nasopharynx p. 7


18. Furunculosis: ANSWER:
A. Group A Beta hemolytic streptococcus B. Staphylococcus aureus
B. Staphylococcus aureus
Furunculosis:
C. Group Beta streptococcus
 Infection spreads to deeper tissues and forms a central core of purulent
D. Moraxella catarrhalis liquefaction (furuncle)
 Extensive and invasive infection of sebaceous glands or hair follicles
with some involvement of subcutaneous tissues
 CA: Staphylococcus aureus

[ENT] T.10 - Diseases of the Nose, Paranasal sinuses, and Nasopharynx p. 6


19. Acute fulminant fungal rhinosinusitis: ANSWER:
A. Acute invasive fungal RS A. Acute invasive fungal RS
B. Chronic invasive Fungal RS
Acute Invasive Fungal RS is also known as Acute fulminant fungal RS
C. Chronic granulomatous fungal RS
D. Allergic fungal RS [ENT] T.10 - Diseases of the Nose, Paranasal sinuses, and Nasopharynx p. 7
20. Charcot Leyden crystals: ANSWER:
A. Acute invasive fungal RS D. Allergic Fungal RS
B. Chronic invasive Fungal RS
Allergic Fungal RS - Criteria of Bent and Kuhn
C. Chronic granulomatous fungal RS
 Eosinophilic mucin (Charcot-Leyden crystals)
D. Allergic fungal RS
 Noninvasive fungal hyphae
 Nasal polyposis

[ENT] T.10 - Diseases of the Nose, Paranasal sinuses, and Nasopharynx p. 8

3 | 3 EARS, NOSE, AND THROAT Recalls No. 12 Editors | ENT TRANS TEAM
ENT: BASIC OTORHINOLARYNGOLOGY MIDTERM QUIZZES
QUESTION BANK
S.Y. 2020-2021

OUTER AND MIDDLE EAR ANATOMY, ANATOMY AND PHYSIOLOGY OF THE INNER EAR, DISEASES OF THE INNER EAR
Item Question Rationale Answer
1 Most common syndromic hereditary SNHL characterized hearing loss The most common syndromic form of hereditary SNHL, B
and goiter Pendred syndrome (PS). The condition is autosomal
A. Eagle Syndrome recessive, and affected individuals also have goiter. The
B. Pendred Syndrome hearing loss is usually congenital and severe to profound,
C. Usher Syndrome although progressive mild to moderate SNHL is sometimes
D. Waaldenburg Syndrome seen.
Chapter 148, p.2295 Cummings 6th Ed.
Slide 6, Diseases of the Inner Ear PPT by Dr. Tolentino
2 Features of PC-BPPV except: The pattern of response is characteristic: A?
A. Downbeating and torsional nystagmus 1) the nystagmus is a combined vertical upbeating and a
B. >1min latency rotary (torsional) component that beats toward the
C. Cupulolithiasis downward ear, and pure vertical nystagmus is not BPPV
D. Rotary 2) a latency of onset of nystagmus, on
the order of seconds, is often apparent
3) the duration of nystagmus is short (<1 minute)
4) vertiginous symptoms are invariably associated
5) the nystagmus disappears with repeated testing, that is, it
is fatigable
6) symptoms often recur with the nystagmus in the opposite
direction upon return of the head to the upright position.

Canalithiasis of the posterior semicircular canal is the most


frequent cause of BPPV.
Chapter 165, p.2551 Cummings 6th Ed.

Features of PC-BPPV:
• Canalithiasis mechanism
• Nystagmus duration
• Vertical and torsional nystagmus
• Reversal of nystagmus
• Fatigability of nystagmus
• Classic eye movements in Dix-Hallpike maneuver
✓ Combined upbeating and torsional
✓ Latency: seconds
✓ Nystagmus: <1 min
✓ Vertiginous symptoms
✓ Nystagmus is fatigable
✓ Symptoms recur
Slide 25-26, Diseases of the Inner Ear PPT by Dr.
Tolentino
3 It separates the IAC into anterior and posterior compartments D
A. Spiral ligament
B. Reissner’s membrane
C. Falciform crest
D. Bill’s bar

Slide 35, Anatomy and Physiology of the Inner Ear PPT by


Dr. Tolentino
4 Arrest at 6th wk AOG results to a partial bony and membranous Arrest at the seventh week of gestation yields a cochlea that C
labyrinth with a 1.5 turn of the cochlea has only 1.5 turns. This is the most common type of cochlear
A. Scheibe malformation, accounting for more than 50% of all such
B. Michel deformities.
C. Mondini Chapter 192, p.2985 Cummings 6th Ed.
D. Santorini Slide 7, Diseases of the Inner Ear PPT by Dr. Tolentino
ENT: BASIC OTORHINOLARYNGOLOGY MIDTERM QUIZZES
QUESTION BANK
S.Y. 2020-2021
5 Site of tumor spread that passes through an incomplete ossification at The incomplete ossification of the anterior bony canal B
the anterior EAC. produces an opening into the infratemporal region, known as
A. Superficial parotid gland the foramen of Huschke, which may also serve as a means
B. Deep lobe of parotid gland for extension of malignant tumors from the
C. Mastoid EAC to the deep lobe of the parotid gland. Naturally
D. IJV occurring defects in the cartilaginous portion of the EAC,
known as the fissures of Santorini, also provide avenues of
spread to the superficial lobe of the gland.
Chapter 127, p.1981 Cummings 6th Ed.
6 Most common neoplastic cause of SNHL Vestibular schwannoma is the most common neoplasm that D
A. Meningioma results in SNHL. Best known as acoustic neuroma, vestibular
B. SCCA schwannomas originate from the vestibular nerves within the
C. Astrocytoma CPA or the internal auditory canal.
D. Vestibular Schwannoma Chapter 150, p.2329 Cummings 6th Ed.
Vestibular Schwannoma – most common neoplastic cause of
SNHL.
Slide 16, Diseases of the Inner Ear PPT by Dr. Tolentino
7 Serves as communication between perilymphatic fluid and CSF At the basal turn of the cochlea is the cochlear aqueduct, a D
A. Modiolus bony channel that allows communication between the
B. Spiral lamina perilymphatic fluid and cerebrospinal fluid of the
C. Helicotrema subarachnoid space in the posterior fossa.
D. Cochlear aqueduct Chapter 128, p.1988 Cummings 6th Ed.
8 Divides the scala media and scala vesibuli into two separate The scala tympani and the scala media are separated by the D
compartments basilar membrane, whereas the scala media and the scala
A. Basilar membrane vestibuli are separated by the Reissner membrane.
B. Stria vascularis Chapter 128, p.1996 Cummings 6th Ed.
C. Spiral ligament
D. Reissner’s membrane
9 Gelatinous mucopolysaccharide mass within a keratin meshwork that The cupula is a gelatinous mass that consists of B
extends to the roof and otoliths being displaced to this area is one of mucopolysaccharides within a keratin meshwork, which
the pathophysiology of BPPV extends from the surface of the cristae to the roof and lateral
A. Crista walls of the membranous labyrinth to form a fluid-tight
B. Cupula partition.
C. Ampulla Chapter 130, p.2013 Cummings 6th Ed.
D. Stereocilia
10 Development of the external ear commences at: Embryology of External Ear B
A. 2nd – 4th wk AOG • Starts at 4th-6th wk AOG
B. 4th – 6th wk AOG • 1st branchial arch → 1st - 3rd hillock
C. 6th – 8th wk AOG • 2nd branchial arch → 4th - 6th hillock
D. 8th – 10th wk AOG Slide 12, Outer and Middle Ear Anatomy PPT by Dr.
Tolentino
11 Divides the scala media and scala tympani into two separate The scala tympani and the scala media are separated by the A
compartments basilar membrane, whereas the scala media and the scala
A. Basilar membrane vestibuli are separated by the Reissner membrane.
B. Stria vascularis Chapter 128, p.1996 Cummings 6th Ed.
C. Spiral ligament
D. Reissner’s membrane
12 Which does not articulate with the temporal bone? The temporal bone articulates with the sphenoid, parietal, B
A. Sphenoid occipital, and zygomatic bones and therefore contributes to
B. Squamosa the cranial, skull base, and facial structure.
C. Zygoma Chapter 127, p.1977 Cummings 6th Ed.
D. Parietal bone
13 Which part of the ossicular chain is most prone to necrosis? The presence of a single nutrient vessel within the long A
A. Lateral process of incus process of incus and the absence of collateral circulation
B. Short process of incus renders this segment of bone susceptible to aseptic necrosis.
C. Stapedial footplate Chapter 127, p.1983 Cummings 6th Ed.
D. Umbo
14 All statements are correct about the vestibular apparatus except: Posterior and lateral to the vestibule are the mastoid air cells. C
A. It is enclosed in the petrous temporal bone Chapter 130, p.2009 Cummings 6th Ed.
B. There are 2 sensory organs for linear acceleration
C. Mastoid air cells are posterior and medial to it
D. The functional pair of right anterior SCC is the left posterior
SCC
15 Which of the following statements is true? The EAC is about 2.5 cm in length and comprises a lateral C
A. The EAC is 1/4 fibrocartilage and 3/4 bony cartilaginous (membranous) portion and a medial bony
B. The EAC is 3/4 fibrocartilage and 1/4 bony portion. The membranous portion accounts for the lateral
ENT: BASIC OTORHINOLARYNGOLOGY MIDTERM QUIZZES
QUESTION BANK
S.Y. 2020-2021
C. The EAC is 1/3 fibrocartilage and 2/3 bony third of the EAC, whereas the bony portion forms the medial
D. The EAC is 2/3 fibrocartilage and 1/3 bony two thirds.
Chapter 127, p.1980-1981 Cummings 6th Ed.
16 Anterior boundary of middle ear Boundaries of Middle Ear D
A. Epitympanum Superior / Roof– epitympanum, tegmen tympani
B. Hypotympanum Inferior / Floor – hypotympanum, IJV
C. Aditus Lateral – Tympanic membrane, scutum
D. Huguier’s Canal Anterior – Eustachian tube, canal for the TT muscle
(Huguier’s Canal)
Posterior – aditus
Medial – promontory (basal turn of cochlea), oval and round
windows, horizontal portion of CNVII, LSCC
Slide 19, Outer and Middle Ear Anatomy PPT by Dr.
Tolentino
17 Most common cause of nongenetic hearing loss According to the Centers for Disease Control and Prevention D
A. HSV (CDC), CMV is the most common congenital viral infection
B. VZV and the most common cause of nongenetic hearing loss in
C. Rubella the United States.
D. CMV Chapter 153, p.2361 Cummings 6th Ed.
18 One spontaneous vertigo lasting at least 20 mins + audiogram Definite Meniere Disease C
documented hearing loss + tinnitus/aural fullness, other factors • Two or more definitive spontaneous episodes of vertigo
excluded: lasting at least 20 minutes
A. Possible Meniere • Audiometrically documented hearing loss on at least
B. Probable Meniere one occasion
C. Definite Meniere • Tinnitus or aural fullness in the suspected ear
D. Certain Meniere • Other causes excluded
Chapter 165, p.2556 Box 165-1 Cummings 6th Ed.
19 Nonpharmacologic treatment for migraine associated vertigo Treatment of Migraine-Associated Vertigo: D
A. Low salt diet ✓ Migraine control diet
B. Regular sleep ✓ Regular sleep
C. Aerobic exercises ✓ Aerobic exercise
D. All of the above Slide 36, Diseases of the Inner Ear PPT by Dr. Tolentino
20 Serves as communication between the scala vestibuli and scala The scala tympani and the scala vestibuli join together at the C
tympani apex of the cochlea to form the helicotrema.
A. Modiolus Chapter 129, p.1996 Cummings 6th Ed.
B. Spiral lamina
C. Helicotrema
D. Cochlear aqueduct
21 Calcification or elongation of the styloid process which is Elongation or angulation of the styloid process can produce A
characterized by throat and neck pain radiating to the ear. the triad of odynophagia, dysphagia, and foreign-body
A. Eagle Syndrome sensation in the throat, known as Eagle syndrome, as a result
B. Pendred Syndrome of the compression of cranial nerves or the internal carotid
C. Usher Syndrome artery.
D. Waaldenburg Sydrome Chapter 127, p.1980 Cummings 6th Ed.
22 Which of the following is not a part of the temporal bone? The temporal bone consists of four embryologically distinct D
A. Petrosa components: these are squamous, mastoid, petrous, and
B. Squamosa tympanic.
C. Tympanic Chapter 127, p.1977 Cummings 6th Ed.
D. Sphenoid
23 Cochleotoxic drugs except: Sensorineural Hearing Loss (SNHL) – Ototoxicity A
A. Streptomycin Aminoglycosides – entry to HC → death → ROS
B. Kanamycin • Cochleotoxic: kanamycin, tobramycin, amikacin,
C. Neomycin neomycin, dihydrostreptomycin
D. Tobramycin • Neomycin toxicity – rapid and profound
• Risk factors: renal disease, prolonged use, increase
serum levels, age, + loop diuretics
Slide 10, Diseases of the Inner Ear PPT by Dr.
Tolentino
24 Major blood supply to the auricle and EAC: Blood Supply: C
A. Anterior auricular artery ❖ External Carotid Artery: major blood supply to auricle
B. Posterior auricular artery & EAC
C. External carotid artery ❖ Posterior auricular artery: mastoid & auricle
D. Deep temporal artery ❖ Superficial Temporal Artery: EAC, auricle
Slide 14, Outer and Middle Ear Anatomy PPT by Dr.
Tolentino
ENT: BASIC OTORHINOLARYNGOLOGY MIDTERM QUIZZES
QUESTION BANK
S.Y. 2020-2021
25 Maneuver used in the diagnosis of BPPV In the presence of a suggestive history, the diagnosis of A
A. Dix Hallpike BPPV is made by observing the classic eye movements in
B. Simont association with the Dix-Hallpike maneuver, which is
C. Epley carried out with the patient positioned on an examination
D. Frenzel table such that when placed supine, the head extends over the
edge of the table.
Chapter 165, p.2551 Cummings 6th Ed.
26 All statement is true except: The scala vestibuli and scala tympani are filled with B
A. perilymph is high in Na and low potassium perilymph, which has a composition similar to that of the
B. perilymph is between membranous labyrinth extracellular fluid (high in sodium, and low in potassium).
C. endolymph high potassium and low sodium The scala media is filled with endolymph, which has a
D. maintained by stria vascularis similar composition to the intracellular fluid (low in sodium,
high in potassium). The maintenance of such a large
electrochemical gradient is accomplished by the stria
vascularis, which resides on the outer wall of the scala
media, away from the modiolus.
Chapter 129, p.1997 Cummings 6th Ed.
27 First central auditory center to receive binaural innervation The SOC is the first central auditory center to receive B
A. Cochlear nerve binaural innervation, and it maintains the tonotopic
B. Superior Olivary Complex organization seen in the cochlea and cochlear nucleus.
C. Lateral Lemniscus Chapter 128, p.1992 Cummings 6th Ed.
D. Inferior Colliculus
28 All statements are correct except: Inner hair cells are flask shaped, receive extensive afferent A
A. Inner hair cells are active transducers innervation, and receive indirect efferent innervation.
B. Inner hair cells are flasked shaped Outer hair cells are cylindrical and receive direct afferent and
C. Outer hair cells are cylindrical efferent innervation. Approximately 3500 inner hair cells are
D. Outer hair cells are arranged in 3 rows arranged in one row on the modiolar side of the tunnel of
Corti, and 12,000 outer hair cells fall in three rows on the
strial side.
Chapter 128, p.1989 Cummings 6th Ed.
29 Which muscle is attached to the mastoid? The site of sternocleidomastoid insertion is indicated by a B
A. Trapezius rough and irregular surface at the mastoid tip. Medial to the
B. Posterior belly of digastric mastoid tip, the posterior belly of the digastric muscle is
C. Anterior belly of digastric inserted in a sulcus that terminates anteriorly at the
D. Omohyoid stylomastoid foramen.
Chapter 127, p.1978 Cummings 6th Ed.

Mastoid: SCM, Posterior belly of digastric


Slide 7, Outer and Middle Ear Anatomy PPT by Dr.
Tolentino
30 Safe sound intensity to work in 1 hour Occupational Safety and Health Administration C
A. 95 dB Standard for Hearing:
B. 100 dB Duration Hearing Level
C. 105 dB 8 hrs 90 dB
D. 110 dB 4 hrs 95 dB
2 hrs 100 dB
1 hr 105 dB
30 mins 110 dB
15 mins 115 dB

Slide 14, Diseases of the Inner Ear PPT by Dr. Tolentino


CRANIOMAXILOFACIAL TRAUMA AND ANATOMY, PHYSIOLOGY, AND DISEASES OF THE ORAL CAVITY
Item Question Rationale Answer
1 Functions of the oral cavity EXCEPT: Functions of oral cavity: E
a. analysis of material before swallowing • Analysis of material before swallowing
b. mechanical swallowing of food • Mechanical processing by the teeth, tongue, and palatal
c. lubrication surfaces
d. limited digestion • Lubrication
e. NOTA • Limited digestion
Oral Cavity ppt, Dr. Vitamog slide 5
2 Space between cheeks and gums The region between the internal mucosa of the cheek and the B
a. buccal mucosa teeth is the vestibule of the mouth.
b. vestibule
c. buccinator space Boies Fundamentals of Otolaryngology 6th Edition Chapter
d. Stensen’s space 14 , Page 275
e. Fissure
ENT: BASIC OTORHINOLARYNGOLOGY MIDTERM QUIZZES
QUESTION BANK
S.Y. 2020-2021
3 A small papilla on vestibular mucosa opposite this structure marks the The parotid duct opens opposite the upper second molar B
opening of the duct of the Parotid gland Boies Fundamentals of Otolaryngology 6th Edition Chapter
a. upper 1st molar 14 , Page 275
b. upper 2nd molar
c. upper 3rd molar
d. lower 2nd molar
e. lower 3rd molar
4 Located on the posterior 1/3 of the dorsal surface of tongue Divided into anterior two third and posterior one third by a E
a. filifom V-shaped sulcus terminalis.
b. fungiform Oral Cavity ppt, Dr. Vitamog slide 19
c. vallate
d. foliate
e. NOTA
5 All EXCEPT______ papillae have taste buds on their surface The papillae in general increase the area of contact between A
a. filifom the surface of the tongue and the contents of the oral cavity.
b. fungiform All except the filiform papillae have taste buds on their
c. vallate surfaces.
d. foliate Oral Cavity ppt, Dr. Vitamog slide 19
e. lymphatic nodules
6 Intrinsic muscles act in altering the shape of the tongue which consist Intrinsic Muscle: C
of the ff EXCEPT No bony attachment.
a. longtitudinal fibers Consist of: Longitudinal fibers, Transverse fibers, Vertical
b. transverse fibers fibers
c. diagonal fibers Function: Alter the shape of the tongue
d. vertical fibers Oral Cavity ppt, Dr. Vitamog slide 26
e. NOTA

7 These happen during Esophageal stage of swallowing EXCEPT Esophageal stage: D


a. begins by relaxing the cricopharyngeal sphincter • purely reflexive
b. time for transit is 10-15 sec. • begins by relaxing the cricopharyngeal sphincter
c. peristaltic waves play active roles • time taken for esophageal transit is 10-15 seconds
d. both voluntary and reflexive • Primary / secondary / tertiary peristaltic waves play
e. hyoid bone, soft palate, and tongue return to their original active roles in this phase
positions • the hyoid bone, soft palate and tongue return to their
’original positions’
Oral Cavity ppt, Dr. Vitamog slide 61
8 Multiple melanotic macules on the skin, lower lip and buccal mucosa Peutz-Jegher: Polyps are nonneoplastic. Patients carry an A
which carry an increased risk of both intestinal and extraintestinal increased risk of both intestinal and extraintestinal
malignancies malignancies. Multiple melanotic macules on the skin, lower
a. Peutz-Jegher lip and buccal mucosa.
b. Plummer Vinson Diseases of Oral Cavity ppt, Dr Vitamog slide 17
c. Sturge-Weber
d. Fordyce granules
e. Melkerson- Rosenthal
9 This is a rare condition consisting of IDA glossitis, and dysphagia with Plummer Vinson Disease: B
increased risk for SCCa of oral cavity and esophagus Clinical- Rare condition consisting of iron-deficiency
a. Peutz-Jegher anemia, glossitis and dysphagia. Oral manifestations include
b. Plummer Vinson a red, smooth, atrophic dorsum of the tongue.
c. Sturge-Weber Increased risk for squamous cell carcinoma of the oral cavity
d. Fordyce granules and esophagus.
e. Melkerson- Rosenthal Etiology- hypochromic, microcytic anemia, that is
consistent with iron deficiency
T/P- Treatment usually addresses the iron-deficiency anemia
via dietary iron supplementation
Diseases of Oral Cavity ppt, Dr Vitamog slide 18
10 This disease entity is caused by autoantibodies directed against the Etiology – Autoantibodies directed against the glycoprotein C
glycoprotein component of desmosomes. Positive Nikolsky sign component of desmosomes
a. Lichen Planus Clinical – Initial manifestation frequently involves the oral
b. Herpes simplex cavity; collapsed vesicles and ulcerations randomly
c. Pemphigus vulgaris distributed (desquamative gingivitis). Induction of blister on
d. Erythema multiforme perilesional mucosa or skin (Nikolsky’s sign).
e. Aphthous stomatitis T/P – Systemic steroids combined with steroid-sparing
immuno-modulating agents (e.g. azathioprine).
Exacerbation and remissions common; complete remission
is rare
Diseases of Oral Cavity ppt, Dr Vitamog slide 33
ENT: BASIC OTORHINOLARYNGOLOGY MIDTERM QUIZZES
QUESTION BANK
S.Y. 2020-2021
11 True of BCCa EXCEPT • Lower lip – mostly SCC A
a. lower lip mostly BCC • Upper lip- mostly BCC
b. rodent ulcer • Risk factors same with SCC
c. prolonged exposure • Prolonged sun exposure
d. risk factors same with SCC • ‘”rodent ulcer”
e. upper lip mostly BCC • Treatment: Surgery with or without RT
Diseases of Oral Cavity ppt, Dr Vitamog slide 75
12 A spillage of salivary secretions into the connective tissue following Etiology – Trauma to the region of the salivary gland. B
rupture of an excretory duct Clinical – A spillage of salivary secretions into the
a. ranula connective tissue following the rupture of an excretory duct.
b. mucocele T/P – No treatment is required for most mucoceles as they
c. sialolithiasis will rupture and heal. Some lesions will require surgical
d. acinic cell excision.
e. pyogenic granuloma Diseases of Oral Cavity ppt, Dr Vitamog slide 56
13 Although asymptomatic, this lesion which is commonly located on the Etiology – Unique aspect is its association with transmitted C
palate can interfere with eating and is associated with HIV acquired HIV virus. The causative factor remains unknown,
a. exostoses although viruses such as CMV, Epstein-Barr virus and
b. torus palatinus hepatitis B virus has been linked to its pathogenesis. Human
c. Kaposi sarcoma papilloma virus genetic sequences have been recovered from
d. Adenocystic carcinoma biopsies of Kaposi Sarcoma lesions.
e. Pleomorphic adenoma Clinical – Examination of the oropharynx may uncover
asymptomatic lesions on the palate, pharynx, or tongue.
Diseases of Oral Cavity ppt, Dr Vitamog slide 49
14 Parts of the midface EXCEPT Midface composed of: Zygoma, Orbit, Maxilla, Nose E
a. zygoma Maxillofacial trauma ppt, Dr. Vitamog slide13
b. orbit
c. maxilla
d. nose
e. NOTA
15 Horizontal buttress of the midface consists of Vertical buttress: D
a. zygomaticomaxillary • Zygomaticomaxillary – lateral
b. nasomaxillary • Nasomaxillary – medial
c. pterygomaxillary • Pterygomaxillary – posterior
d. supraorbital bar Horizontal buttress
e. AOTA • Alveolus
• Orbital floor
• Orbital rims
• Supraorbital bar
Maxillofacial trauma ppt, Dr. Vitamog slide14
16 When you encounter a trauma patient, things to prioritize are the ff B
EXCEPT
a. check vital signs
b. ask for ancillaries
c. control bleeding
d. history and PE
e. stabilize patient
17 Pyramidal fracture Le fort I – Horizontal – palate B
a. LeFort I Le fort II – pyramidal
b. LeFort II Le fort III – Craniofacial disjunction
c. LeFort III Maxillofacial trauma ppt, Dr. Vitamog slide23
d. LeFort IV
e. NOTA
18 Management of LeFort fracture are the ff EXCEPT • Reestablish facial height and width E
a. Re-establish facial height and width • IMF with ORIF of mandible
b. IMF with ORIF of mandible • Zygomatic arch reconstruction restores facial width and
c. reconstruction from stable to unstable bone projection
d. reconstruction from lateral to medial • Reconstruction continues from stable bone to
e. NOTA unstable and from lateral to medial
Maxillofacial trauma ppt, Dr. Vitamog slide24
19 Complex zygomaticomaxillary fracture includes the ff EXCEPT Simple: Isolated lateral fracture E
a. zygomatico-frontal suture Complex ZMC
b. infraorbital rim • Multiple complex Tetrapod fracture (Tripod Fracture)
c. zygomatic arc • Zygomatic-Frontal Suture
d. lateral orbital wall • Infraorbital Rim
e. NOTA • Zygomatic Arc
ENT: BASIC OTORHINOLARYNGOLOGY MIDTERM QUIZZES
QUESTION BANK
S.Y. 2020-2021
• Lateral Orbital Wall
Maxillofacial trauma ppt, Dr. Vitamog slide39
20 NOE fracture involves the ff EXCEPT NOE fractures -nasal, frontal, ethmoids, maxillary, lacrimal E
a. nasal bone bones
b. lacrimal bones Maxillofacial trauma ppt, Dr. Vitamog slide52
c. ethmoids
d. maxilla
e. NOTA
ENT: BASIC OTORHINOLARYNGOLOGY MIDTERM EXAM
QUESTION BANK
S.Y. 2020-2021

Item Question Rationale Answer


1 All statements are true regarding the embryology of the The external ear develops during the 6th week of gestation and is A
external ear except: completely developed by the 20th week.

a. Development starts at 4th-6th week AOG Distinct condensations of tissue, known as the hillocks of His, give rise
b. The 1st bronchial arch will form the first three hillocks to the tragus and most of the helix from the 1st branchial arch, and to
c. The 2nd brachial arch will form the second three the antihelix, antitragus, lobule, and inferior helix from the second
hillocks branchial arch.
d. The external ear will achieve its adult shape and size
by 20th week AOG Hillocks 1, 2, and 3 are part of the mandibular portion of the 1st
pharyngeal arch

Chap. 55, Pediatric Surgery 7th ed, p. 349, Chap. 19, Langman’s 13th ed, p.
125, Chapter 7, Williams OB 25th ed p. 1824, Chap 127, Cummings 5th ed.
2 All statements are true regarding the EAC except: The EAC is about 2.5 cm long and is composed of a lateral C
cartilaginous (membranous portion and a medial bony portion. The
a. It is approximately 2.5 cm long lateral portion accounts for the lateral third of the EAC, whereas the
b. Lateral 1/3 is made up of hair follicles, sebaceous and bony portion forms the medial two thirds.
sweat glands
c. Medial 1/3 is bony and has no subcutaneous tissue Otoscopic examination of the ™ is facilitated by first straightening the
d. The proper way to retract the ear during PE in a child external auditory meatus by gently pulling the auricle upward and
is downwards and backwards backward in the adult, and straight backward or backward and
downward in the infant.

p. 562, Chap 11, Snell Clinical Anatomy 9th ed., p. 1823, Chap 127,
Cummings 5th ed.
3 Vestigial lymph channels that drain into the superficial Naturally occurring defects in the cartilaginous portion of the EAC, A
parotid gland known as fissures of Santoritin, also provide avenues of spread to the
superficial lobe of the [parotid] gland.
a. Fissures of Santorini
b. Foramen of Huschke p. 1823, Chap 127, Cummings 5th ed.
c. Foramen of Luschka
d. Fissures of Morgagni
4 All statements are true regarding the boundaries of B
middle ear except:

a. The lateral boundary comprises the tympanic


membrane and scutum
b. The medial boundary comprises the horizontal semi
circular canal
c. The inferior boundary comprises the tegmen tympani
d. The superior boundary comprises the epitympanum

Anatomy of TB, OME PPT - Doc. Lacuesta slide 19


ENT: BASIC OTORHINOLARYNGOLOGY MIDTERM EXAM
QUESTION BANK
S.Y. 2020-2021
5 Which statement is true regarding the Eustachian tube? B

a. It connects the middle ear and the oropharynx


b. It is 2/3 cartilaginous and 1/3 bony
c. It is usually open
d. A and C is true

Anatomy of TB, OME PPT - Doc. Lacuesta slide 20


6 Which statement is true regarding the Eustachian tube? D

a. The Eustachian tube is approximately 18mm in


children and inclined at a 45 degree angle from the
horizontal plane
b. The Eustachian tube is approximately 35mm in adults
and inclined at a 90 degree angle from the horizontal
plane
c. Both statements are correct
d. Both statements are incorrect
Anatomy of TB, OME PPT - Doc. Lacuesta slide 21
7 What is the smallest bone in the body? At 3 mm x 2.5 mm, the "stapes" in the middle ear is the smallest named C
bone in the human body
a. Malleus
b. Incus
c. Stapes
d. none of the above
8 What is the innervation of the tensor tympani muscle? The tensor tympani is innervated by a branch of the fifth cranial nerve B

a. CN IV
b. CN V
c. CN VI
d. CN VII
9 Which one is correct? Central Auditory Pathway C
Cochlear afferent fibers synapse on neurons of the dorsal and ventral
a. CN VIII > cochlear nerve > superior olivary complex cochlear nuclei. These neurons give rise to axons that contribute to the
> lateral lemniscus > inferior colliculus > medial central auditory pathways. Some of the axons from the cochlear nuclei
lemniscus > auditory cortex cross to the contralateral side and ascend in the lateral lemniscus, the
b. CN VIII > cochlear nerve > inferior olivary complex main ascending auditory tract. Others connect with various ipsilateral
> lateral lemniscus > inferior colliculus > medial or contralateral nuclei, such as the superior olivary nuclei, which
geniculate body > auditory cortex projects through the ipsilateral and contralateral lateral lemnisci. Each
c. CN VIII > cochlear nerve > superior olivary complex lateral lemniscus ends in an inferior colliculus. Neurons of the inferior
> lateral lemniscus > inferior colliculus > medial colliculus project to the medial geniculate nucleus of the thalamus,
geniculate body > auditory cortex which gives rise to the auditory radiation. The auditory radiation ends
d. CN VII > cochlear nerve > superior olivary complex in the auditory cortex (areas 41 and 42), located in the transverse
> lateral lemniscus > inferior colliculus > medial temporal gyri in the temporal lobe.
geniculate body > auditory cortex

Berne & Levy 6th Ed, Chapter 8, page 144


10 Which part of the temporal bone houses the inner ear? The bony labyrinth lies within the petrous part of the temporal bone. C

a. mastoid Gray’s Anatomy 40th Ed, Chapter 37, page 633


b. styloid
c. petrous
d. squama
11 It partially divides the scala vestibuli and scala tympani Spiral lamina B
and is the point of attachment for the basilar membrane
-Partial division of scala vestibuli and scala tympani
a. Modiolus -Point of attachment for basilar membrane
b. Spiral lamina
c. Spiral limbus Doc Tolentino’s ppt on Inner Ear, page
d. Tectorial membrane
ENT: BASIC OTORHINOLARYNGOLOGY MIDTERM EXAM
QUESTION BANK
S.Y. 2020-2021
12 Medial attachment for the Reissner’s membrane Spiral limbus C

a. Modiolus ● Thickened band of periosteum


b. Spiral lamina ● Medial attachment of RM
c. Spiral limbus ● Gives rise to TM
d. Tectorial membrane
Doc Tolentino’s ppt on Inner Ear, page 7
13 Which statement is true regarding the anatomy of the `Organ of corti important structures: A/C
Organ of Corti?
● Tectorial Membrane - compliant gelatinous over the inner
a. The tectorial membrane is a gelatinous material over and outer cells
the inner and outer cells. ● Cells (inner and outer)
b. Reticular lamina serves as barrier from the perilymph ● Reticular Lamina - lightly interwoven matrix supporting the
which is toxic to hair cells outer hair cells
c. A is true
d. Both statements are true Inner Ear Anatomy and Physiology lecture, Dr. Tolentino, slide 7
14 Which statement is true regarding hair cell signal When there is movement of hair cells, particularly when the stereocilia D
transduction? are DEFLECTED IN THE DIRECTION OF THE TALLEST ROW →
causes the tip links to stretch → STRETCH SENSITIVE CATIONIC
a. Stereocilia are deflected in the direction of the tallest CHANNELS OPEN → large influx of cationic current → hair cell
row causing tip links to stretch DEPOLARIZATION
b. Cell hyperpolarization is the result of this stretch
which causes influx of cationic currents As the stereocilia are deflected AWAY from the tallest row →
c. Deflection of stereocilia from the tallest row causes relaxation of the tip links → decreased probability of ion channels
relaxation of the tip links thereby leads to opening → HYPERPOLARIZATION of hair cell
hyperpolarization of the hair cell
d. A and C are correct Inner Ear Anatomy and Physiology lecture, Dr. Tolentino, slide 12
15 All statements are true except: The Olivovochlear reflex protects the ear from acoustic trauma, and A
also discriminates transient sounds from background noise.
a. The afferent auditory system comprises reflexes the
protect the ear form acoustic trauma The olivocochlear reflex and Middle ear muscle reflex are both under
b. The stapedius muscle is responsible for primary EFFERENT auditory system.
sound-evoked potentials
c. The olivocochlear reflex discriminates transient Inner Ear Anatomy and Physiology, Dr. Tolentino, slide 17
sounds from background noise
d. It travels along the vestibular nerve to the cochlear to
innervate the outer hair cell
16 The cochlear nerve enters the CNS medialy via: Cochlear nerve enters the CNS medialy via the Foramen of Luschka C

a. Fissures of Santorini Dr. Tolentino, Inner Ear Lecture, Slide #20


b. Foramen of Huschke
c. Foramen of Luschka
d. Fissures of Morgagni
17 Point of entry of peripheral auditory information to Cochlear nucleus- is the point of entry of peripheral auditory B
central auditory system information to central auditory system

a. Cochlear nerve Dr. Tolentino, Inner Ear Lecture, Slide #20


b. Cochlear nucleus
c. Superior olivary complex
d. Medial geniculate body
18 Primary auditory cortex Primary auditory cortex - Brodmann area 41 A

a. Brodmann area 41 Secondary auditory cortex - Brodmann area 42


b. Brodmann area 42
c. Brodmann area 43 Dr. Tolentino, Inner Ear Lecture, Slide #25
d. Brodmann area 44
19 All statements are true regarding the embryology of the Embryology of Vestibular System C
vestibular system except: 4th-25th wk AOG
Ectoderm: otic placode 🡪 otic pit 🡪 otic vesicle/otocyst
a. Development starts at 4th until 25th wk AOG Otic vesicle differentiates to utriculosaccular chamber
b. Ectoderm forms the otic placode, pit and ultimately Mesoderm: bony otic capsule/labyrinth
the otic vesicle
c. The otic placode differentiates into a utriculosaccular Dr. Tolentino, Inner Ear Lecture, Slide #24
chamber
d. Mesoderm forms the labyrinth
ENT: BASIC OTORHINOLARYNGOLOGY MIDTERM EXAM
QUESTION BANK
S.Y. 2020-2021
20 Responsible for sensing angular acceleration 5 sensory organs: A
•3 semicircular canals angular accelerations or rotations
a. Semicircular canals •2 otoliths/maculae linear acceleration
b. Maculae
c. Both Dr. Tolentino, Inner Ear Lecture, Slide #30
d. Neither
21 Responsible for linear acceleration Macula: Sensitive to gravity and linear acceleration forces B

a. Semicircular canals Dr. Tolentino, Inner Ear Lecture, Slide #40


b. Maculae
c. Both
d. Neither
22 What is the position of the vestibular apparatus in Vestibular Apparatus - Between IAC anteromedially and ME laterally C
relation to the internal auditory canal?
Inner Ear Anatomy and Physiology slide 31 Dr. Tolentino PPT
a. Lateral
b. Medial
c. Anteromedial
d. Posteromedial
23 It divides the IAC into anterior and posterior Bill’s Bar- divides the IAC into anterior and posterior D
compartment
Bill bar is a bony anatomical landmark that divides the superior
a. Scarpa’s ganglion compartment of the internal acoustic meatus into an anterior and
b. Facial nerve posterior compartment.
c. Falciform crest
d. Bill’s Bar https://radiopaedia.org/articles/bill-bar-1, Inner Ear Anatomy and Physiology Dr.
Tolentino audio
24 Cause/s of mixed hearing loss CSOM- Chronic Suppurative Otitis Media- Mixed Hearing loss B

a. Genetic disorders Hearing loss in CSOM can result from perforation of the tympanic
b. Infections membrane, disruption of ossicular chain (conductive hearing loss),
c. Head trauma outer hair cell damage caused by the diffusion of bacterial toxins into
d. All of the above the inner ear (sensorineural hearing loss), or both (mixed hearing loss)

https://www.ncbi.nlm.nih.gov/pmc/articles/PMC7085926/, Diseases of the


Inner ear Dr. Tolentino audio
25 It is a syndromic hereditary SNHL with accompanying C
interstitial rhinitis
Syndromic
a. Pendred Syndrome
Pendred Syndrome- goiter+bilateral SNHL
b. Waardenburg Syndrome
c. Alport Syndrome
d. Usher Syndrome Waardenburg Syndrome- dystopia canthorum + broad nasal root +
eyebrow confluence + iridis + white forelock + SNHL

Alport Syndrome- interstitial nephritis + SNHL

Usher Syndrome- retinitis + SNHL +/- vestibular defects

Dr. Camille Tolentino. Ppt presentation “Diseases of the Inner Ear”. Slide#6
26 Complete absence of all bony and membranous labyrinth SNHL- Developmental A
Scheibe dysplasia- cochleosaccular dysplasia· Membranous labyrinth
a. Mondini aplasia only
b. Michel’s aplasia Mondini dysplasia- partial bony and membranoud labyrinth
c. Scheibe dysplasia Michel’s aplasia- complete absence of all bony and membranous
d. None of the above structures

Dr. Camille Tolentino. Ppt presentation “Diseases of the Inner Ear”. Slide#7
27 Known mechanism of loop diuretics in ototoxicity Loop diuretics- alter metabolism in the stria vascularis C
● Reversible SNHL
a. ROS accumulation
b. Cell death
c. Stria vascularis metabolism alteration Dr. Camille Tolentino. Ppt presentation “Diseases of the Inner Ear”. Slide#10
d. Decreased elimination
ENT: BASIC OTORHINOLARYNGOLOGY MIDTERM EXAM
QUESTION BANK
S.Y. 2020-2021
28 Audiometric finding for noise induced hearing loss On an audiogram, noise induced hearing loss (NIHL) will usually be D
seen first as a slight loss of hearing in the 4 kHz region.
a. 1,2,3 kHz loss with a 2 kHz notch This is recognized on an audiogram as a notch centered around 4000
b. 2,3,4 kHz loss with a 3 kHz notch Hz and it is the characteristic audiometric pattern of early NIHL.
c. 3,4,6 kHz loss with a 4 kHz notch
d. 4 and 6 kHz loss with a 5 kHz notch Higher frequencies 1st to be involved in NIHL

Dr. Lacuesta lecture Basic Audiometry Trans 11 pg 5,


https://www.audiometrictestingperth.com.au/noise-induced-hearing-loss/
29 All statements are true regarding differentiation of a. False - imbalance in central vertigo is severe A
central and peripheral origin of vertigo except b. True
c. True
a. Imbalances in central vertigo is mild d. True
b. Hearing loss in peripheral vertigo is frequent
c. Nystagmus in peripheral vertigo is unidirectional Trans 6 Diseases of Inner Ear pg 3 Dr. Tolentino’s slides
d. Neurologic symptoms in central vertigo is frequent
30 Repositioning maneuver using gravity as treatment of Benign paroxysmal positional vertigo (BPPV) produces characteristic D
BPPV symptoms of brief episodes of vertigo triggered by changes in head
position. BPPV can often be treated effectively with canalith
a. Dix Hallpike repositioning maneuvers such as the Epley maneuver.
b. Semonts
c. Brandt Daroff Cummings Otolaryngology Chapter 167 pg 3200
d. Epley
31 There is a 50% hearing impairment with this pathogen Congenital Rubella Syndrome- 50% hearing impairment B

a. CMV Diseases of the Inner Ear Lecture ENT by Dr. Camille Tolentino
b. Rubella
c. GBS
d. Mumps
32 Vestibulotoxic drug Ototoxic/Vestibulotoxic drugs: Streptomycin and Gentamycin A

a. Gentamicin Diseases of the Inner Ear Lecture ENT by Dr. Camille Tolentino
b. Kanamycin
c. Amikacin
d. Furosemide
33 NOE fracture involves the ff EXCEPT NOE: Nasal, frontal, ethmoids, maxillary, lacrimal bones D

a. nasal bone Craniomaxillofacial Trauma lecture by Dr. Maria Concepcion Vitamog


b. lacrimal bones
c. ethmoids
d. NOTA
34 Normal intercanthal distance B

a. 25mm
b. 30mm
c. 35 mm
d. 40mm

Dr. Vitamog’s Lecture. Craniomaxillofacial trauma, Slide #58


35 Management for nasal bone fracture is at optimum when Nasal fracture Treatment:Optimal management: within 1st 3 hours of A
done on/within: injury

a. 1st 3 hours Dr. Vitamog’s Lecture. Craniomaxillofacial trauma, Slide #66


b. 1st 6 hours
c. 1st 8 hours
d. within 1 week
ENT: BASIC OTORHINOLARYNGOLOGY MIDTERM EXAM
QUESTION BANK
S.Y. 2020-2021
36 True of mandibular fracture epidemiology D

a. body> condyle> angle> parasymphysis


b. angle> body> condyle> parasymphysis
c. angle> condyle> parasymphysis> body
d. condyle> body> angle> parasymphysis

Dr. Vitamog’s Lecture. Craniomaxillofacial trauma, Slide #70


37 True of TMJ except: TMJ-synovial diarthrodial joint; hinge, glide and rotate; articulating D
surface— fibrocartilage
a. synovial diarthrodial joint
b. hinge, glide and rotate Dr. Vitamog lecture, Craniomaxillofacial trauma, slide 71
c. articulating surface – fibrocartilage
d. NOTA
38 The mesiobuccal cusp of the permanent maxillary first Class I- mesiobuccal cusp of the maxillary 1st molar sitting within the A
molar occludes distal to the buccal groove of the mesio-buccal groove of the mandibular 1st molar.
permanent mandibular first molar. Class II- Maxillary molar more anterior- chin retruded
Class III- Maxillary molar more posterior- chin prognathic
a. Angle class I Dr. Vitamog lecture, Craniomaxillofacial trauma, slide 77
b. Angle class II
c. Angle class III
d. Angle class IV
39 Anterior vertical overlap Overbite is the vertical overlap of maxillary incisors over mandibular A
incisors
a. overbite
b. overjet Dr. Vitamog lecture, Craniomaxillofacial fractures, slide 79
c. crossbite
d. wear facet
40 Anterior crossbite is seen in? Angle’s Classification B
Class I- Mesio-buccal cusp of maxillary first molar sitting within the
a. Angle class I mesio-buccal groove of mandibular 1st molar
b. Angle class II Class II- Maxillary molar more anterior- chin retruded
c. Angle class III Class III- Maxillary molar more posterior- chin prognathic
d. Angle class IV
Doc Vitamog, Craniomaxillofacial Trauma, Slide 77
41 Most commonly injured nerve in mandibular fracture: The trigeminal nerve is the largest cranial nerve and gives rise to three C
branches: ophthalmic, maxillary, and mandibular. The mandibular
a. CN III nerve is the largest branch of the trigeminal nerve and its sensory and
b. CN IV motor roots exit the skull via the foramen ovale.
c. CN V
d. CN VI Electrophysiologic Investigation of Mandibular Nerve Injury (1991). Arch
Phys Med Rehabll Vol72
42 Best imaging view for mandibular fractures The “gold standard” imaging study for displaying the mandible is the D?
panoramic radiographic view (Panorex). Alternatively, a plain film
a. lateral oblique such as Towne’s view is specific for the condyles. The lateral oblique
b. posteroanterior view displays the condyle, coronoid, ramus, angle, and body.
c. anteroposterior
d. reverse towne Panoramic views with posteroanterior or reverse Towne's views are
likely to give a higher yield than the panoramic view alone.

Schubert, W. (2010, February 13). Radiographic diagnosis of mandibular


fractures: Mode and implications. Operative Techniques in Otolaryngology-
Head and Neck Surgery.
https://www.sciencedirect.com/science/article/abs/pii/S1043181002800549.
Cumming’s Otorinolaryngology 5th Edition Part 8, Chapter 189, Page 2699
43 Functions of the oral cavity EXCEPT FUNCTIONS OF THE ORAL CAVITY D
• Analysis of material before swallowing
a. analysis of material before swallowing • Mechanical processing by the teeth, tongue, and palatal surfaces
b. mechanical swallowing of food • Lubrication
c. lubrication • Limited digestion
d. NOTA Dr. Vitamog’s Oral Cavity PPT slide 5
ENT: BASIC OTORHINOLARYNGOLOGY MIDTERM EXAM
QUESTION BANK
S.Y. 2020-2021
44 Multiple melanotic macules on the skin, lower lip and Peutz - Jegher A
buccal mucosa which carry an increased risk of both Autosomal dominant hereditary hamartomatous polyposis syndrome
intestinal and extraintestinal malignancies Polyps are non - neoplastic → increase risk of both intestinal &
extraintestinal malignancies
a. Peutz-Jegher
b. Plummer Vinson Multiple melanotic macules on the skin, lower lip and buccal mucosa
c. Sturge-Weber Systemic symptoms: abdominal pain, rectal bleeding, & diarrhea
d. Melkerson- Rosenthal
Dr. Vitamog’s Diseases of the Oral cavity PPT slide 17
45 This disease entity is caused by autoantibodies directed Pemphigus Vulgaris C
against the glycoprotein component of desmosomes. - Autoantibodies against glycoprotein component of desmosomes
Positive Nikolsky sign - Rare in children
- Usually >50 y/o
a. Lichen Planus - Initial manifestation: involves the oral cavity; collapsed vesicles
b. Herpes simplex and ulcerations randomly distributed (desquamative gingivitis)
c. Pemphigus vulgaris - Nikolsky’s sign : induction of blister on perilesional mucosa or
d. Erythema multiforme skin

Dr. Vitamog’s Diseases of the Oral cavity PPT slide 33


46 True of BCCA EXCEPT Lower lip: squamous cell carcinoma predominates A
Basal cell carcinoma disproportionately arises on the upper lip.
a. lower lip mostly BCC
b. rodent ulcer Cummings Otolaryngology, Head and Neck surgery and oncology Page 1302
c. risk factors same with SCC
d. upper lip mostly BCC
47 The cheek is composed of this muscle: The cheek is composed of Buccinator muscle, covered laterally by the A
skin and medially by the mucous membrane
a. buccinators Hyoglossus and Genioglossus - are extrinsic muscles
b. hyoglossus Geniohyoid and mylohyoid - muscular diaphragm with the tongue on
c. genioglossus top of it to complete the floor of the oral cavity
d. geniohyoid
The Oral cavity, Dr. Lacuesta ppt Slide 13
48 Provides sensory innervation to the floor of mouth Lingual nerve – a branch of the mandibular (V3) division of the D
trigeminal nerve
a. greater palatine nerve
b. maxillary nerve The Oral cavity Dr. Lacuesta ppt Slide 28
c. buccal nerve
d. lingual nerve
49 Embryological remnant that marks the site of the upper Foramen cecum B
end of thyroglossal duct ● A pit that marks the apex of the sulcus terminalis
● An embryological remnant that mark the site of the upper end
a. sulcus terminalis of the thyroglossal duct
b. foramen cecum
c. foramen lacerum The Oral cavity Dr. Vitamog ppt Slide 19
d. terminalis rotundum
50 These papillae don’t possess taste buds on their surfaces All except filiform papillae have taste buds on their surfaces C

a. foliate The Oral cavity Dr. Vitamog ppt Slide 23


b. fungiform
c. filiform
d. spongiform
51 The tongue is composed of 2 types of muscles: extrinsic The tongue is composed of 2 types of muscles: A
and intrinsic ● extrinsic muscles
● intrinsic muscles
a. True
b. False The Oral cavity Dr. Vitamog ppt Slide 25
52 Extrinsic muscles alter the shape of the tongue Intrinsic muscle’s function: alter the SHAPE of the tongue B
Extrinsic muscle’s function: Function: Help in
a. True MOVEMENTS of the tongue
b. False

Dr. Maria Concepcion Vitamog’s Loom Video Lecture 1 on Anatomy and


Physiology of Oral Cavity (mark 12:48)
ENT: BASIC OTORHINOLARYNGOLOGY MIDTERM EXAM
QUESTION BANK
S.Y. 2020-2021
53 Which of the following equipment are used in indirect Examination of the Nasopharynx (Indirect Nasopharyngoscopy) A
nasopharyngoscopy?
● Utilize same light source
a. Nasopharyngeal mirror, sterile gloves, tongue ● Utilize a nasopharyngeal mirror size 0
depressor, head mirror, goose neck lamp ● Tongue is retracted as with oral examination and
b. Nasopharyngeal mirror, sterile gloves, sterile gauze, nasopharyngeal mirror slide over the depressor
head mirror, goose neck lamp ● Ask the patient to breathe through the nose to relax the soft
c. Laryngeal mirror, sterile gloves, sterile gauze, head palate
mirror, goose neck lamp ● Mirror is rotated around to visualize entirely the entire
d. Laryngeal mirror, sterile gloves, tongue depressor, nasopharynx
head mirror, goose neck lamp
Dr. Nicanor Lacuesta’s Google Meet Video Lecture on Basic ENT
Examination
54 Which of the following material are used in indirect Examination of the Hypopharynx and Larynx C
laryngoscopy?
● Utilize same light source
a. Nasopharyngeal mirror, sterile gloves, tongue ● Proper positioning of the patient is vital
depressor, head mirror, goose neck lamp ● Patient is asked to stick out his/her tongue which is grasped
b. Nasopharyngeal mirror, sterile gloves, sterile gauze, by the middle finger and the thumb while the index is used to push up
head mirror, goose neck lamp on the upper incisors of the patient
c. Laryngeal mirror, sterile gloves, sterile gauze, head ● Laryngeal mirror of adequate size is warmed or application
mirror, goose neck lamp of anti-fogging solution is done
d. Laryngeal mirror, sterile gloves, tongue depressor, ● Mirror is gently slid, avoiding posterior third of the tongue or
head mirror, goose neck lamp posterior pharyngeal wall and pressed upward against the soft palate
● If patient gags, ask the patient to pant/ apply topical
anesthesia spray

Dr. Nicanor Lacuesta’s Google Meet Video Lecture on Basic ENT


Examination
55 How do you retract the pinna to straighten the external Remember that the external ear canal is not straight. To straighten it C
auditory canal in adults? for examination, grasp the pinna to retract it backward and upward in
adults and downward in infants.
a. inferiorly, posteriorly, laterally
b. Superiorly, anteriorly, laterally Adams, G.L., Boies, L.R., & Hilger, P.A., (1997). Boies Fundamentals of
c. Superiorly, posteriorly, laterally Otolaryngology. 6th Ed, Ch 1, p5-6
d. Inferiorly, posteriorly, laterally
56 If the examiner’s dominant eye is the right, where should If the dominant eye is the right eye, light source is placed on the left C
the lamp be placed in doing a basic ENT examination? side of the patient’s head because the mirror is placed on the right eye
of the examiner.
a. At the examiner’s right side, facing the patient
b. At the examiner’s left side, facing the patient Dr. Lacuesta lecture, Basic ENT Examination (audio) - source: ENT - Trans 5
c. At the examiner’s right, facing him/her - Basic ENT Examination, page 1
d. At the examiner’s left, facing him/her
57 What is the optimal distance of the patient’s head from The most useful size head mirror has a 3 ½ -inch diameter with a 1/2 - D
the head mirror? inch hole in the center and focal length of about 14 inches.

a. 10 inches Adams, G.L., Boies, L.R., & Hilger, P.A., (1997). Boies Fundamentals of
b. 13 inches Otolaryngology. 6th Ed, Ch 1, P. 3
c. 12inches
d. 14 inches
58 Which of the following is true regarding the proper way Speculum C
of using a nasal speculum? ● Speculum must be removed without closing it
● Held on the same side of the nasal cavity examined
a. It is held on the opposite side of the nasal cavity being ● Index finger is used to grasp the nasal ala with the speculum
examined
b. The speculum is closed or released prior to removal Dr. Lacuesta’s lecture, Basic ENT Examination
from the nasal vestibule
c. The forefinger is used to hold the nasal ala against the
speculum
d. The patient’s head is tilted slightly downward
59 How should the examiner be seated compared to the Patient is seated with the head slightly higher than that of the C
patient? examiner’s head
Eye level of the examiner should be at the level of what he will examine
a. The heads of both the patient and the examiner should on the patient
be at the same height
ENT: BASIC OTORHINOLARYNGOLOGY MIDTERM EXAM
QUESTION BANK
S.Y. 2020-2021
b. The head of the examiner should be higher than that Examiner may stand or sit, must not stoop down and must be
of the patient’s comfortable
c. The head of the examiner should be lower than that of Dr. Lacuesta’s lecture, Basic ENT Examination
the patient
d. The patient can lie down with the examiner stooping
down to examine the patient
60 Which of the following is a good substitute for a head Head mirror substitute: focusable light on a headband A
mirror?
Dr. Lacuesta’s lecture, Basic ENT Examination
a. A head lamp that can be focused
b. A handheld flashlight
c. A goose neck lamp directed onto the examined area
d. A handheld light source held by an assistant
61 Which of the following is true about using a head mirror - 100 watt or stronger unfrosted light bulb mounted on a goose C
in doing a basic ENT examination? neck stand without a reflector (aka droplight)
- Focal length: 14 inches
a. The light bulb ideally 50 watts or more mounted on a - Examiner directs the area to be examined into the field of
goose neck lamp view and
b. The focal length of the head mirror is 16 cm - avoids repositioning himself
c. The area to be examined should be moved into the
light Dr. Lacuesta’s lecture, Basic ENT Examination
d. The examiner has the liberty to move his/her head
around
62 Which of the following is used to move the tongue out Examination of the hypopharynx and larynx C
of your visual field when you do an indirect
laryngoscopy using a laryngeal mirror Patient is asked to stick out his/her tongue which is grasped by the
middle finger and the thumb while the index finger is used to push up
a. A wooden tongue depressor on the upper incisors of the patient.
b. A steel tongue depressor
c. A gloved hand to grasp the tongue Dr. Lacuesta’s lecture, Basic ENT Examination
d. A sterile gauze to grasp the tongue
63 Which of the following is used to depress the tongue The mirror is warmed and introduced into the oral cavity, whereas the A
when you do a posterior rhinoscopy using a tongue is depressed with a tongue depressor. Tongue is retracted
nasopharyngeal mirror? downward as with oral examination and nasopharyngeal mirror slide
over the depressor and tuck it right under the soft palate or the uvula so
a. A tongue depressor you can see upwards toward the nasopharynx
b. A gloved hand
c. Sterile gauze Doc Lacuesta’s Powerpoint and Lecture, Basic ENT Examination
d. Another laryngeal mirror
64 The head mirror has a hole at the middle which has a Head mirror has 3 ½ inch with a ½ inch hole at the center B
diameter of?
Doc Lacuesta’s Powerpoint and Lecture, Basic ENT Examination
a. 1 inch
b. ½ in
c. ¾ in
d. ¼ in
65 The diameter of the head mirror is usually Head Mirror: 3 ½ inch mirror with a ½ inch hole diameter at the center C

a. 2 ½ in Doc Lacuesta’s Powerpoint and Lecture, Basic ENT Examination


b. 3 in
c. 3 ½ in
d. 4 in
66 In pneumatic otoscopy, when there is middle ear effusion Perforations and middle ear effusions are common causes of nonmobile B
what can be observed? tympanic membranes.

a. Inward movement of the tympanic membrane Part 2, Chapter 8, p. 97, Cummings Otolaryngology Head & Neck Surgery, 5th
b. No movement of the tympanic membrane ed
c. Shifting of air-fluid levels
d. Bulging of the tympanic membrane
67 Which fingers are usually utilized in using a nasal Nasal speculum should be used without discomfort to the patient. It is B
speculum? held with the thumb and middle finger; index finger is used to grasp the
nasal ala with the speculum.
a. Thumb and index finger
b. Thumb and the middle finger Doc Lacuesta. Powerpoint: Basic ENT Examination .
c. The thumb and the palm of the hand
ENT: BASIC OTORHINOLARYNGOLOGY MIDTERM EXAM
QUESTION BANK
S.Y. 2020-2021
d. The thumb and the ring finger
68 In doing an indirect laryngoscopy, what can be done to If patient gags, ask the patient to pant (breathe rapidly through the D
avoid a gag reflex mouth) or apply a topical anesthesia spray e.g. Lidocaine spray.

a. Reassure the patient Doc Lacuesta. Powerpoint and Lecture: Basic ENT Examination.
b. Perfect technique in doing the examination
c. Ask the patient to hold his/her breath
d. Application of topical anesthesia
69 What is the first turbinate that you can visualize when Anterior Rhinoscopy: C
you do an anterior rhinoscopy?
● Students are usually capable to observe the position of the
a. Superior turbinate nasal septum with possible deformities, inferior nasal turbinate and the
b. Middle turbinate quality of nasal mucosa
c. Inferior turbinate
d. Supreme turbinate https://www.slideshare.net/dilu172/examination-of-nose
70 In clearing the ears of impacted cerumen, when is aural IMPACTED CERUMEN B
toilette or aural irrigation not advisable? Water irrigation or aural toilet - Not done if TM is perforated
Doctor Lacuesta’s lecture on Diseases of External Ear
a. When there is diffuse swelling in the ear canal
b. When there is a suspected or known tympanic
membrane perforation
c. In pediatric patients as the experience can prove to be
traumatic
d. In dry, hard cerumen since it cannot be readily
removed
71 Which of the following cannot be used as IMPACTED CERUMEN D
cerumenolytics? Soften dry or hard cerumen using ear drops, oil, or hydrogen peroxide

a. Commercially prepared ear drops


b. Hydrogen peroxide Doctor Lacuesta’s lecture on Diseases of External Ear
c. Oil
d. Sterile saline solution

72 In lacerations of the external ear, which of the following Management: C


is included in the management? o Thorough wound cleaning should be done
o Only the skin is sutured using non-absorbable suture
a. Layered repair of the laceration (to include all layers) o Lacerated cartilage approximated under skin
b. Secondary intention healing ▪ Do not suture the cartilage &/or perichondrium
c. Antibiotic therapy o Antibiotic therapy
d. Analgesic therapy o Advise daily wound care

Doc Laquesta Disease on External Ear


73 Hematoma in the external ear, if left untreated can lead If untreated, hematoma leads to cauliflower ears. C
to this outcome
Dr Lacuesta's PPT: Diseases of the External Ear
a. Resorption of the underlying cartilage
b. Suppurative chondritis
c. Cauliflower-ear deformity
d. Necrosis of the overlying skin
74 Antibiotic treatment in hematoma formation in the Management of hematoma: Anti-staphylococcal antibiotics C
external ear is directed at?
Dr. Lacuesta's PPT: Diseases of the External Ear
a. Atypical bacteria
b. Gram negative
c. Staphylococcal
d. Anaerobic bacteria
75 Which should be done first in the treatment of Management: B
hematomas in the external ear? o Evacuation of hematoma
▪ Simple incision & drainage
a. Pressure dressing o Pressure dressing
b. Drainage ▪ Rubber drain may be placed to make sure all of the hematoma is
c. Observation evacuated
d. Antibiotic therapy o Anti-staphylococcus antibiotics

Doc Lacuesta's PPT : Diseases of the External Ear, Slide # 13


ENT: BASIC OTORHINOLARYNGOLOGY MIDTERM EXAM
QUESTION BANK
S.Y. 2020-2021
76 In 25% of burns this condition can develop in the External ear burn occur in 90% of patients with C
external ear. facial burns
● 25% develop suppurative chondritis
a. Necrotizing fasciitis
b. Atrophy Dr. Lacuesta’s PPT: Diseases of the External Ear, Slide #15
c. Suppurative chondritis
d. Cauliflower ear
77 Which of the following is true about the management of Management for Frostbite involves rapid warming and antibiotics B.
frostbite in the external ears? initially done with delayed surgical debridement.

a. Immediate surgical debridement is done Doc Lacuesta’s PPT


b. Rapid warming and antibiotics initially done
c. Expectant observation
d. Gradual heating of the exposed part
78 Which of the following is the mechanism by which Frostbite - soft tissues and cartilage are frozen. Injury is secondary to A
damage is caused by frostbite in the external ear? direct cellualr damage and microvascular insult leading to local
ischemia.
a. Microvascular insult leading to local ischemia
b. Generation of free radicals Doc Lacuesta’s PPT
c. Loss of innervation leading to atrophy
d. Loss of nutritional supply
79 Which of the following is true about - Infection of perichondrium/cartilage A/D?
perichondritis/chondritis? - Perichondritis may follow any infection or hematoma
- Pain over auricle and deep in canal
a. Always with a precedent cause - May be spontaneous (overt diabetes)
b. Characterized by pain localized in the external auricle - Result of trauma to auricle
c. Can be autoimmune in etiology
d. Can be secondary to trauma
Dr. Lacuesta’s PPT on Diseases of the External Ear slide 18
80 Which of the following is true in the management of Visualization is mandatory for removal C.
foreign bodies in the external auditory canal? After removal, additional injuries must be assessed like lacerations or
abrasion which should be treated accordingly
a. Removal can be done blindly Sedation or general anesthesia may be required if the patient is
b. After extraction of the foreign body there is no longer uncooperative
any needed precaution
c. Inspection after removal should be done to assess for Dr. Lacuesta’s PPT on Diseases of the External Ear Slide 20
any additional injuries
d. Even if the patient is uncooperative there is no
indication for general anesthesia or sedation
81 Furunculosis in the external auditory canal must be Furunculosis A
managed by which of the following? ● Outer third of EAC
● Staphylococcus aureus or S. albus
a. Systemic antibiotics and analgesics ● Affectation of pilosebaceous follicles
b. Topical antibiotics and anti-inflammatory ear drops ● Incision and drainage is the best option
c. avoid incising or draining the abscess as this may ● Systemic antibiotics and analgesics
spread infection to the confluent areas
d. antibiotics are targeted at anaerobic bacteria DISEASES OF THE EXTERNAL EAR Nicanor B. Lacuesta Jr. MD, DPBO-
HNS PPT. p.24
82 A 36-year old female came in for pain and pruritus over Otomycosis : Aspregillus niger infection C
the right ear for 3 days. There is purulent discharge and
hearing loss. Patient is a known swimmer and is
currently residing in the Philippines. On otoscopy, there
is a noted whitish and cotton-like material on the middle
2/3 of the EAC. What is the most probable diagnosis?

a. Foreign body DISEASES OF THE EXTERNAL EAR Nicanor B. Lacuesta Jr. MD, DPBO-
b. Diffuse otitis externa HNS PPT. p.30
c. Otomycosis
D.Perichondritis
83 Swimmer’s ear is a condition which is commonly caused Bacterial infections account for over 90% of cases of AOE, and fungal C
by which of the following bacteria? infections account for the rest. Pseudomonas aeruginosa,
Staphylococcus epidermidis, and S. aureus represent the first, second,
a. Staphylococcus and third most common bacterial isolates from AOE, respectively.
b. Anaerobic bacteria
c. Pseudomonas species
ENT: BASIC OTORHINOLARYNGOLOGY MIDTERM EXAM
QUESTION BANK
S.Y. 2020-2021
d. Aspergillus niger species Cummings Otolaryngology: Head & Neck Surgery 6th Ed., Chapter 137,
p.2117
84 Swimmer’s ear can be managed by which of the Topical therapy is the treatment of choice for AOE, because oral B
following? antibiotics have been shown not to be effective.

a. Systemic antibiotics Cummings Otolaryngology: Head & Neck Surgery 6th Ed., Chapter 137,
b. Topical corticosteroids p.2117
c. Antifungal preparations
d. Systemic antiviral medication
85 A 60 year-old female presented with otalgia and aural Herpes Zoster Oticus B
discharge for 3 days. On physical examination there was • Ramsay-Hunt Syndrome
noted vesicles in the auricle, EAC, and face of the • Caused by chickenpox virus
patient. There is also noted decreased movement of the • Infection of the geniculate ganglion and other cranial nerve
facial muscles in the ipsilateral side. What is the most ganglia
probable causative agent? • Painful vessicles in EAC and auricle
• Seventh nerve involvement presents with cutaneous herpes and
a. Pseudomonas species facial palsy
b. Chicken pox virus
c. Staphylococcus aureus
d. Aspergilus niger Dr. Lacuesta, Disease of external ear ppt slide #31
86 What is the most probable diagnosis for the case above? Herpes Zoster Oticus A
• Ramsay-Hunt Syndrome
a. Ramsay-hunt syndrome • Caused by chickenpox virus
b. Otomycosis • Infection of the geniculate ganglion and other cranial nerve
c. Diffuse otitis externa ganglia
d. Perichondritis • Painful vessicles in EAC and auricle
• Seventh nerve involvement presents with cutaneous herpes and
facial palsy

Dr. Lacuesta, Disease of external ear ppt slide #31


87 Treatment of herpes zoster oticus includes which of the Herpes zoster oticus treatment C
following?
● Symptomatic treatment
a. Systemic antibiotics for infection ● Systemic steroid for facial palsy
b. Surgical debridement
c. Systemic steroids for facial palsy Dr. Nicanor Lacuesta lecture ppt on the Diseases of the External Ear - Slide 31
d. Topical keratolytics
88 This condition can be precipitated by the neomycin Contact dermatitis is precipitated by ear plug use and hypersensitivity B
component in common otic preparations. to neomycin component of otic preparations.

a. diffuse otitis externa Dr. Nicanor Lacuesta lecture ppt on the Diseases of the External Ear - Slide 33
b. contact dermatitis
c. perichondritis
d. otomycosis
89 a 50 year old male complains of lesions on the external Contact dermatitis: Erythematous, edematous, pruritic lesions, with C
ear which is described as raised, erythematous with a vesicles or exudates (flaky). Common among hearing aid and ear plug
grayish scale and thick lichenification. He is a know users (e.g., surfers, competitive gun shooters, people with perforated
frequent swimmer and uses a behind the ear hearing aid. TM who would like to swim). New hearing aids →hypoallergenic
What is the most probable diagnosis? material

a. Seborrheic dermatitis Doc Lacuesta's PPT : DISEASES OF THE EXTERNAL EAR


b. Psoriasis
c. Contact dermatitis
d. Otomycosis
90 A 18 year old female came in for lesions on her right ear. Psoriasis A
On inspection there were erythematous papules forming • External ear involved in 1/5th of cases
large patches with scaling. What is your probable • Etiology is unknown
diagnosis? • Multifactorial
• Erythematous papules from large patches with thick scale
a. Psoriasis • Epithelial hyperplasia with hyperkeratosis
b. Seborrheic dermatitis • Management:
c. Contact dermatitis o Topical steroids
d. otomycosis o UV phototherapy
o Severe cases: cyclosporin or methotrexate → prevent hypermitotic
activity
ENT: BASIC OTORHINOLARYNGOLOGY MIDTERM EXAM
QUESTION BANK
S.Y. 2020-2021
Doc Lacuesta's PPT : DISEASES OF THE EXTERNAL EAR
91 What is the mainstay treatment of seborrheic dermatitis? Management: Topical steroids, Keratolytics A

a. Topical steroids Doc Lacuesta's PPT : DISEASES OF THE EXTERNAL EAR


b. Systemic steroids
c. UV treatment
d. Topical antibiotics
92 For severe cases of psoriasis which of the following can Management: B
be used
● Topical steroids
a. Betamethasone ● UV phototherapy
b. Cyclosporin ● Severe cases: cyclosporine or methotrexate
c. UV phototherapy
d. Avoidance of offending substance Doc Lacuesta’s PPT: DISEASES OF THE EXTERNAL EAR
93 The following condition is caused by Group A beta Erysipelas caused by Group A, beta-hemolytic streptococcus . B
hemolytic streptococci Skin: bright red; well demarcated, advancing margin

a. perichondritis
b. Erysipelas Dr. Lacuesta’s ppt and lecture, Diseases of external ear slide # 43
c. Relapsing polychondritis
d. Necrotizing otitis externa
94 This condition in the external ear can be fatal. Necrotizing otitis externa- potentially lethal infection. C
Typically seen in diabetics (poorly controlled) and
a. psoriasis immunocompromised patients.
b. Relapsing polychondritis
c. Necrotizing otitis externa Dr. Lacuesta’s ppt and lecture, Diseases of external ear slide # 38
d. Ramsay hunt syndrome
95 What is the most common etiologic agent of necrotizing Necrotizing external otitis - potentially lethal infection and common C
otitis externa? cause is pseudomonas aeruginosa

a. Beta hemolytic bacteria Reference: Dr. Lacuesta ppt, Diseases of the external ear
b. Streptococcus species
c. Pseudomonas species
d. Malassezia furfur
96 Which of the following is associated with seborrheic Seborrheic Dermatitis - erythematous, raised with a greasy scale and D
dermatitis associated with malassezia furfur

a. Beta hemolytic bacteria Reference: Dr. Lacuesta ppt. Diseases of the external ear
b. Streptococcus species
c. Pseudomonas species
d. Malassezia furfur
97 A 80 year old farmer complains of a mass on his right SQUAMOUS CELL CA comprised 80% of all malignant tumors of the C
auricle for 2 years. On inspection, there is a noted 5x3 auricle and EAC
darkly pigmented lesion on the antihelix with a central •Metastasis uncommon
area of necrosis and a raised whitish margin and apparent •Management:Small tumors: local wide excision :Include the margin of
erosion of the underlying cartilage. What is the mode of normal tissue to ensure removal of the entire carcinoma
treatment?

a. Systemic antibiotics DOC LACUESTA'S PPT: DISEASES OF EXTERNAL EAR, slide # 50


b. Symptomatic treatment
c. Wide excision with frozen section
d. Minimal excision
98 What is your probable diagnosis of the above-mentioned SQUAMOUS CELL CA comprised 80% of all malignant tumors of the C
case? auricle and EAC
•Metastasis uncommon
a. Malignant melanoma •Management:Small tumors: local wide excision :Include the margin of
b. Basal cell carcinoma normal tissue to ensure removal of the entire carcinoma
c. Squamous cell carcinoma
d. Cylindroma DOC LACUESTA'S PPT: DISEASES OF EXTERNAL EAR, slide #50
99 This condition in the external ear can also involve the RELAPSING POLYCHONDRITIS B
larynx in 90% of cases - Unknown etiology
- Progressive destruction of cartilage
a. Perichondritis - Nose, ears, and larynx involved in 90% of cases
b. Relapsing polychondritis - Resembles inflammed cauliflower ear
c. Squamous cell carcinoma - Steroids to control acute attacks and suppress recurrences.
ENT: BASIC OTORHINOLARYNGOLOGY MIDTERM EXAM
QUESTION BANK
S.Y. 2020-2021
d. Basal cell carcinoma Reference: Dr. Lacuesta, Diseases of the external ear, Slide #41
100 This condition of the external ear is an acute superficial ERYSIPELAS B
cellulitis that extends beyond the external ear into the - Acute superficial cellulitis
surrounding tissues. - Group A, beta hemolytic streptococci
- Skin: bright red; well demarcated, advancing margin
a. Seborrheic dermatitis - Rapid treatment with oral or IV antibiotics if insufficient
b. Erysipelas response
c. Impetigo
d. Psoriasis Reference: Dr. Lacuesta, Diseases of the external ear, Slide #43

- GOD BLESS! LABAN MEDISINA! PARA SA BAYAN -


ENT: BASIC OTORHINOLARYNGOLOGY FINALS TERM QUIZZES
QUESTION BANK
S.Y. 2020-2021

Item Question Rationale Answer


1 The following paranasal sinuses drain into Nasal Cavity Openings: Middle Meatus B
the middle meatus, except: ● Middle ethmoidal air cells: bulla
● Anterior ethmoidal air cells: hiatus semilunaris
A. Frontal sinus ● Maxillary sinus: hiatus semilunaris
B. Posterior ethmoid ● Frontal sinus: infundibulum
C. Anterior ethmoid Ostia of posterior ethmoidal cells open into supreme meatus (75%)
D. Maxillary sinus
The Nose and Paranasal Sinuses Trans - Dr. Cabrera, page 9
2 Which of the following drains into the C
inferior meatus:

A. Sphenoid Sinus
B. Posterior ethmoids
C. Nasolacrimal duct
D. None of the above

Slide #17 Anatomy of Nose & Paranasal Sinuses Module - Dr. Cabrera
3 The following are components of the nasal D
septum, except:

A. Vomer
B. Maxillary crest
C.Perpendicular plate of the ethmoid
D. Cribriform plate of the ethmoid

Anatomy of Nose & Paranasal Sinuses Module - Dr. Cabrera


4 Which of the paranasal sinuses is the only The maxillary and sphenoidal sinuses are present in a rudimentary form at birth; B
sinus routinely present at birth: they enlarge appreciably after the eighth year and become fully formed in
adolescence.
A. anterior ethmoid
B. maxillary sinus Snell’s Clinical Anatomy by Regions 9th ed, Chapter 11, page 641
C. Frontal sinus
D. Posterior ethmoid
5 All of the following are part of the lateral The lateral boundaries of the nasal cavity are: D
border of the nasal cavity except: ● Nasal bone
● Turbinates
A. turbinates ● Maxillary bone
B. horizontal plate of palatine bone ● Palatine bone
C. maxillary bone ● Ethmoid
D. none of the above ● Sphenoid

The Nose and Paranasal Sinuses lecture, Dr. Cabrera


6 The largest turbinate/conchae Inferior Turbinate - largest A

A. inferior Dr. Cabrera, The Nose and Paranasal Sinuses Ppt (1st yr), Slide #42
B. middle
C. superior
D. supreme
7 The middle meatus contains all of the Rest of middle meatus posteroinferiorly contains D
following except: ▪ Ethmoidal bulla
▪ Uncinate process
A. ethmoid bulla ▪ Semilunar hiatus
B. uncinate process
ENT: BASIC OTORHINOLARYNGOLOGY FINALS TERM QUIZZES
QUESTION BANK
S.Y. 2020-2021
C. hiatus semilunaris Dr. Cabrera, Nose and Paranasal Sinuses Lecture, Video IMG_0005MP4 38:56
D. sphenoethmoidal recess
8 Which of the following parts of the nasal Perpendicular plate of ethmoid - Unites sup with cribrifom plate of ethmoid. B
septum unites with the cribriform plate:

A. vomer
B. perpendicular plate of ethmoid
C. quadrangular cartilage
D. crest of maxillary bone

Nose and Paranasal Sinuses Dr. Cabrera ppt


9 All of the following are part of kiesselbach The source of 90% of anterior nosebleeds is within Kiesselbach's plexus (also C
plexus except: known as Little’s area) on the anterior nasal septum. There are five named vessels
whose terminal branches supply the nasal cavity:
A. ethmoidal artery
B. superior labial artery ● Anterior ethmoidal artery
C. lesser palatine artery ● Posterior ethmoidal artery
D. sphenopalatine artery ● Sphenopalatine artery
● Greater palatine artery
● Superior labial artery
https://www.ncbi.nlm.nih.gov/books/NBK435997
10 The space between the bulla ethmoidalis The space between the anterior wall of ethmoid bulla and the free edge of uncinate D
and uncinate process is called: process is called the hiatus semilunaris; it opens anterosuperiorly into a cavity
called the ethmoid infundibulum.
A. agger nasi
B. olfactory cleft
C. frontonasal duct https://www.intechopen.com/books/paranasal-sinuses/paranasal-sinus-anatomy-what-the-
surgeon-needs-to-know
D. hiatus semilunaris
11 The osteomeatal unit is composed of the Osteomeatal complex represents the final common pathway for drainage and D
following except: ventilation of the frontal, maxillary and anterior ethmoid air cells. Delicate air cells
around semilunar hiatus, namely Haller cells, agger nasi cells, concha bullosa, bulla
A. ethmoidal bulla ethmoidalis and uncinate process constitutes anatomical variants of osteomeatal
B. uncinate process complex.
C. hiatus semilunaris
D.middle turbinate The osteomeatal complex (OMC) is the collection of structures that aids in mucus
drainage and airflow between the maxillary sinus, the anterior ethmoid air cells,
and the frontal sinus. It is located on the lateral wall of the nasal cavity and has
several well defined borders. The OMC contains several important landmarks, such
as the ethmoid bulla, hiatus semilunaris, ethmoidal infundibulum, frontonasal duct
(recess) and uncinate process.

https://www.kenhub.com/en/library/anatomy/the-ostiomeatal-complex,
https://www.ncbi.nlm.nih.gov/pmc/articles/PMC4575670/
12 The primary drainage pathway of the The maxillary sinuses are the largest of the sinuses. They are located laterally and A
maxillary sinus: slightly inferiorly to the nasal cavities. They drain into the nasal cavity at the hiatus
semilunaris, underneath the frontal sinus opening.
A. Hiatus semilunaris
B. Uncinate process The Paranasal Sinuses. https://teachmeanatomy.info/head/organs/the-nose/paranasal-
C. ethmoid infundibulum sinuses/
D. Ethmoid bulla
13 Which of the following does NOT supply BLOOD SUPPLY OF LATERAL NASAL WALL D
the medial and lateral walls of the nasal
cavity? • Anterior septal branch, anterior lateral nasal branch, external nasal branch of
anterior ethmoidal artery
A. sphenopalatine artery • Alar branches of lateral nasal branch (of facial artery) o Supplies the anterior part
B. anterior and posterior ethmoidal arteries • Septal and lateral nasal branches of posterior ethmoidal artery
C. greater palatine artery • Posterior lateral nasal branches of sphenopalatine artery
D. infraorbital artery
Blood supply of the Nasal cavity
ENT: BASIC OTORHINOLARYNGOLOGY FINALS TERM QUIZZES
QUESTION BANK
S.Y. 2020-2021
External carotid artery and its branches: sphenopalatine, greater palatine, superior
labial and lateral nasal arteries
Internal carotid artery and its branches: anterior and posterior ethmoidal arteries

https://www.kenhub.com/en/library/anatomy/nasal-cavity, Dr. Cabrera Anatomy of Nose &


Paranasal Sinuses
14 Depressor muscle: Elevator Muscles: B
- Procerus
A. Procerus - Levator Labii Alaeque Nasi
B. alar nasalis - Anomalous Nasi
C. Levator labii alaequae nasi
D. Anomalous nasi Depressor Muscles:
- Alar Nasalis
- Depressor Septi Nasi

Doctor Cabrera The Nose and Paranasal Sinuses Module, Video 1, 14:55
15 Boundaries of external nasal valve except: External Nasal Valve Boundaries: A
lateral crus(lateral);
A. anterior head of inferior turbinate medial crus(medial);
B. nasal floor nasal floor(inferior)
C. lateral crus
D. medial crus Internal Nasal Valve:
Narrowest portion of the nasal cavity where air enters
Boundaries: caudal border of the upper lateral cartilage, septum, and the anterior
head of the inferior turbinates

Reference: Dr. Cabrera Anatomy of Nose & Paranasal Sinuses Video 1 (25:38)
16 Crescent shaped bone curved Uncinate process latin “processus uncinatus” - hook outgrowth C
downward/backwards attached to the
perpendicular process of the palatine bone Hiatus semilunaris inferioris - two dimensions;, sagitally oriented, crescent-shaped
and the ethmoid process of the inferior cleft. Passageway or door for gaining access to the ethmoid infundibulum
turbinate:
Ethmoid bulla - largest and most consistent air cell in the anterior ethmoid group.
A. hiatus semilunaris Posterior to the uncinate process
B. ethmoid bulla
C. uncinate process Ethmoid infundibulum - 3D space bordered medially by the UP and laterally by
D. ethmoid infundibulum the lamina papiracea with the maxillary ostium in its floor

Dr. Cabrera lecture on the Nose and Paranasal sinuses Anatomy (from 1st yr lec) Slide 83
17 Two-dimensional, sagittally oriented, hiatus semilunaris A
crescent-shaped cleft that represents the ● Two-dimensional, sagittally oriented, crescent-shaped cleft that
shortest distance between the free posterior represents the shortest distance between the free posterior margin of the
margin of the uncinate and the anterior uncinate and the anterior surface of the ethmoid bulla
surface of the ethmoid bulla:
Doctor Cabrera The Nose and Paranasal Sinuses Module
A. hiatus semilunaris
B. ethmoid bulla
C. uncinate process
D. ethmoid infundibulum
18 Three dimensional space bordered Ethmoid Infundibulum - 3D space bordered medially by the UP and laterally by D
medially by the uncinate process and the lamina papiracea (LP) with the maxillary ostium in its floor
laterally by the lamina papyracea with the
maxillary ostium in its floor: Dr. Cabrera’s Video Lecture on Nose and Paranasal Sinuses (video 2, mark 32:42)

A. hiatus semilunaris
B. ethmoid bulla
C . uncinate process
D. ethmoid infundibulum
19 Separates anterior and posterior ethmoids: The Basal lamella of the MT separates anterior and posterior ethmoids. A
A. basal lamella
B. ethmoid bulla Dr. Cabrera’s video lecture on Nose and Paranasal sinuses (video 2, mark 40:40)
C. sphenoethmoid recess
D. agger nasi
20 Roof of ethmoid: Fovea ethmoidalis is also known as roof of ethmoid B

A. cribriform plate Dr. Cabrera’s lecture, The Nose and Paranasal Sinuses
B. fovea ethmoidalis
ENT: BASIC OTORHINOLARYNGOLOGY FINALS TERM QUIZZES
QUESTION BANK
S.Y. 2020-2021
C. crista galli
D. lateral lamella
21 Greater curvature of middle turbinate is CONCHA BULLOSA B
concave to middle meatus: Aerated middle turbinate (unilateral / bilateral)
May obstruct middle meatus osteomeatal complex → sinusitis
A. concha bullosa
B. paradoxical middle turbinate PARADOXICAL MIDDLE TURBINATE
C. ethmoid bulla Greater curvature of middle turbinate is concave to middle meatus
D. basal lamella
ETHMOID BULLA
Rounded elevation below middle concha
Formed by projection of middle ethmoidal air cells

BASAL LAMELLA
Separates the anterior and posterior ethmoids

Dr. Cabrera, The Nose and Paranasal Sinuses Ppt (1st year), Slide # 54
22 Posteroinferior portion of the nasal septum: The unpaired vomer bone, often referred to simply as the vomer, is triangular- C
shaped and forms the posterior-inferior part of the nasal septum.
A. perpendicular plate
B. septal cartilage https://courses.lumenlearning.com/openstax-anatomyandphysiology/chapter/7-2-the-skull/
C. vomer
D. membranous septum
23 Antrum of Highmore: The pyramid-shaped maxillary sinus (or antrum of Highmore) is the largest of B
the paranasal sinuses, and drains into the middle meatus of the nose through the
A. Frontal sinus osteomeatal complex.
B. maxillary sinus
C. ethmoid sinus https://medical-dictionary.thefreedictionary.com/antrum+of+Highmore
D. sphenoid sinus
24 Lymphatic drainage of ethmoid sinus: Ethmoid Sinus Primary Lymphatic Drainage: A/B
● Submandibular lymph nodes - anterior and middle ethmoid sinuses
A. submandibular nodes ● Retropharyngeal lymph nodes - posterior ethmoid sinus
B. retropharyngeal nodes
C. parapharyngeal nodes Gupta, D.K. (2015). Nose and Paranasal Sinus. Retrieved from
D. deep lateral cervical nodes https://www.slideshare.net/DeepakKumarGupta2/nose-and-paranasal-sinus
25 The only sinus that does not arise as an The sphenoid sinus originates in the sphenoid bone at the center of the head. It D
outpouching from the lateral nasal wall: arises not from an outpouching of the nasal cavity but from the nasal embryonic
lining
A. Frontal sinus
B. maxillary sinus https://emedicine.medscape.com/article/1899145-overview
C. ethmoid sinus
D. sphenoid sinus
26 All of the following describes an • Histologically benign B
angiofibroma except: • Locally aggressive
• Male adolescents
a. Occurs in male adolescents • Less than 0.05%
b. Originates at the superior margin of the • Origin: Superior margin of sphenopalatine foramen
greater palatine foramen • Intracranial extension 10%-36%
c. Can extend intracranially in about 10-
• Unilateral nasal obstruction &Epistaxis
36%
d. Presents with unilateral nasal obstruction
and epistaxis
27 Classic pathognomonic radiographic Classic radiographic feature: B
finding of angiofibroma o Anterior bowing of posterior maxillary sinus wall (Pathognomonic)
o Erosion of sphenoid, hard Palate, medial wall of maxillary sinus
a. anterior bowing of the anterior maxillary o Displacement of nasal septum
sinus wall
b. anterior bowing of the posterior
maxillary sinus wall Reference: Diseases of the nose - Dr. Cabrera Lecture
c. posterior bowing of the anterior
maxillary sinus wall
d. posterior bowing of the posterior
maxillary sinus wall
28 Mainstay treatment of angiofibroma: ● Treatment for Juvenile Nasopharyngeal Angiofibroma includes: B
radiation, surgery and hormonal therapy
a. medical ○ Surgery: preferred treatment
ENT: BASIC OTORHINOLARYNGOLOGY FINALS TERM QUIZZES
QUESTION BANK
S.Y. 2020-2021
b. surgical ○ Radiation: for residual disease or when surgery is not possible
c. chemotherapy
d. radiotherapy Boies Fundamentals of Otolaryngology, 6th Ed. Part 4, Chapter 17 p. 336,
29 Which of the following is/are true about the Acute bacterial rhinosinusitis (ABRS) D
acute bacterial rhinosinusitis (ABRS) may
be made in adults with: – symptoms of a viral upper respiratory infection (URI) that have not improved
after 10 days or worsen after 5 to 10 days.
a. symptoms of a viral upper respiratory
infection (URI) that have not improved – some or all of the following symptoms: nasal drainage, nasal congestion, facial
after 10 days or worsen after 5 to 10 days pressure/pain, postnasal drainage, hyposmia/anosmia, fever, cough, fatigue,
b. symptoms may include nasal drainage, maxillary dental pain, and ear pressure/fullness.
nasal congestion, facial pressure/pain,
postnasal drainage, hyposmia/anosmia,
– European Position Paper on Rhinosinusitis and Nasal Polyps (EPOS) -- all cases
fever, cough, fatigue, maxillary dental
lasting for < 12 weeks with complete resolution of symptoms
pain, and ear pressure/fullness
c. duration of symptoms is less than 12
REFERENCE : Dr. Jesson Cabrera, Diseases of the Nose , Slide 47
weeks
d. all of the above
30 Primary treatment of ABRS except: Management of ARBS: D

a. empiric antibiotic therapy ● Empiric antibiotic therapy for low risk drug resistance
b. nasal saline irrigation ● Nasal Saline irrigation
c. intranasal corticosteroids ● INCS
d. antihistamine ● Penicillin alternative if allergic (doxy, levo, moxifloxacin)
● 2nd line antimicrobial for high risk and failure of initial management
● + further workup of CT, culture, and immune studies

Diseases of the Nose. pdf Dr.Cabrera Slide 52


31 Work up for patients with ABRS with A
inadequate response to treatment may • Imaging Studies ?? – reserved for patients with persistent symptoms, recurrent
include the following: ABRS or complications

a. CT of the Paranasal Sinuses Diseases of the Nose . PDF. JessonDeo Cabrera, RMT, MD ,P.51
b. Sinus or meatal culture
c. Immune system studies
d. all of the above
32 Chronic rhinosinusitis (CRS) is defined as A
inflammation of the nasal cavity and In general, CRS refers to symptoms that last 12 weeks or longer without symptom-
paranasal sinuses and/or the underlying free periods.
bone that has been present for at least:
Cummings Otolaryngology: Head & Neck Surgery 6th Ed., Chapter 196, p.3038
a. 12weeks
b. 13 weeks
c. 14 weeks
d. 15weeks
33 What do you call when there is sudden • Chronic RS (CRS) (with or without nasal polyps): – Symptoms lasting for more A
deterioration of the patient’s condition with than 12 weeks without complete resolution of symptoms
either worsening of baseline symptoms or • Recurrent ARS: – ≥ 4 episodes per year, each lasting ≥ 7-10 days with complete
development of additional symptoms in resolution in between episodes
cases of CRS? • Acute exacerbation of CRS: – sudden worsening of baseline CRS with return to
baseline after treatment
a. acute exacerbation of CRS
b. recurrent ABRS Diseases of the Nose ppt, Dr. Cabrera, slide #43
c. both
d. CRS
34 CRS without nasal polyp, being an According to the most recent US guidelines, both topical corticosteroids and nasal B
inflammatory disease, should be primarily saline irrigations are recommended as initial medical therapies for affected
treated with: patients. Intranasal corticosteroids can decrease nasal polyp size, lessen sinonasal
symptoms, and improve patient quality of life.
a. antibiotic
b. intranasal corticosteroid https://www.ncbi.nlm.nih.gov/pmc/articles/PMC4939220
c. nasal saline irrigation
d. leukotriene receptor antagonist
ENT: BASIC OTORHINOLARYNGOLOGY FINALS TERM QUIZZES
QUESTION BANK
S.Y. 2020-2021
35 Allergic rhinitis is treated based on the D
following:

a. severity of symptoms
b. frequency of symptoms
c. co-morbidities
d. all of the above

36 Chandler classification of angiofibroma JUVENILE NASOPHARYNGEAL ANGIOFIBROMA STAGING A


where in the tumor is confined to the (CHANDLER)
nasopharyngeal vault: • Stage I: tumor confined to the nasopharynx
• Stage II: tumor extends to the nasal cavity or sphenoid
a. I • Stage III: tumor involves the maxillary sinus, ethmoid sinus, infratemporal fossa,
b.II orbit, cheek, and cavernous sinus
c. III • Stage IV: tumor is intracranial
d. IV
Review of Otolaryngology by Cummings Chapter 5 Pg 93
37 Theories of choanal atresia formation FOUR BASIC THEORIES: A
except: 1. Persistence of buccopharyngeal membrane
2. Abnormal persistence of bucconasal membrane
a. Persistence of retropharyngeal 3. Abnormal mesoderm forming adhesions in nasochoanal region
membrane 4. Misdirection of neural crest cell migration
b. Abnormal persistence of bucconasal
membrane Dr. Cabrera ppt Nose and PNS , slide # 6
c. Abnormal mesoderm forming adhesions
in nasochoanal region
d. Misdirection of neural crest cell
migration
38 Paradoxical cyanosis: Choanal Atresia Signs and Symptoms under bilateral is paradoxical cyanosis which C
present at rest and improves with exertion. Opposite pattern relative to cyanosis
a. nasal glioma with cardiac cause.
b. nasal dermoid cyst
c. choanal atresia Reference: Diseases of the nose - Dr. Cabrera Lecture, Slide 8
d. encephalocele
39 Mc Govern nipple: Choanal Atresia treatment C
Bilateral:
a. nasal glioma – immediate management:
b. nasal dermoid cyst – airway stabilization with:
c. choanal atresia • oral airway
d. encephalocele • Intubation if ventilation isrequired
• Perforation of atresia plate
• McGovern nipple

Dr. Cabrera ppt. Nasal Diseases.slide 9


ENT: BASIC OTORHINOLARYNGOLOGY FINALS TERM QUIZZES
QUESTION BANK
S.Y. 2020-2021
40 . Epithelium-lined cavities or sinus tracts Nasal dermoid cyst B
filled with keratin debris, hair follicles, • Usually present at birth and may remain unrecognized
sweat glands, and sebaceous glands: • Epithelium-lined cavities or sinus tracts filled with keratin debris, hair
follicles, sweat glands, and sebaceous glands
a. nasal glioma • Arise from ectodermal elements of fetal trilaminar septum which fail to
b. nasal dermoid cyst degenerate
c. choanal atresia
d. encephalocele Reference: ENT - Trans 10 - Diseases of the Nose - Dr. Cabrera p.2

41 This is due to abnormal closure of the Nasal Glioma A


fonticulus frontalis which leads to an
ectopic rest of glial tissue left ● Abnormal closure of the fonticulus frontalis can lead to an ectopic rest
extracranially: of glial tissue if left extracranially

a. nasal glioma Diseases of the Nose, Dr. Cabrera Trans, page 3


b. nasal dermoid cyst
c. choanal atresia
d. encephalocele
42 Furstenburg sign positive: D

a. nasal glioma
b. nasal dermoid cyst
c. choanal atresia
d. encephalocele

Dr. J. Cabrera lecture - Diseases of the Nose


43 Most common site of PNS mucocele: Frontal sinus – most common site of occurrence B

a. maxillary Dr. J. Cabrera lecture - Diseases of the Nose


b. frontal
c. ethmoid
d. sphenoid

44 Anatomical boundary of anterior and Maxillary sinus ostium A


posterior epistaxis: ● Anterior epistaxis - epistaxis anterior to the maxillary sinus ostium
● Posterior epistaxis - epistaxis posterior to the maxillary sinus ostium
a. maxillary ostium
b. uncinate process Dr. J. Cabrera lecture - Diseases of the Nose
c. ethmoid bulla
d. hiatus semilunaris

45 Furunculosis: B
Staphylococcus aureus or S. albus affectation of the pilosebaceous follicles.
a. Group A Beta hemolytic streptococcus Recurrent Furunculosis - challenges and management: a review - NCBI - NIH
b. staphylococcus aureus https://www.ncbi.nlm.nih.gov/pmc/articles/PMC3934592
c. Group Beta streptococcus
d. Moraxella catarrhalis
46 Primary treatment of acute rhinosinusitis Treatment: B
except: - Supportive measures to relieve nasal obstruction and prevent sinusitis
- Antibiotics (Only for secondary bacterial infections)
ENT: BASIC OTORHINOLARYNGOLOGY FINALS TERM QUIZZES
QUESTION BANK
S.Y. 2020-2021
a. bed rest - Antihistamines
b. co-amoxiclav - Topical vasoconstrictors
c. cetirizine - Oral decongestants Antipyretic/analgesic
d. adequate hydration - Bed rest (Best treatment for uncomplicated colds)
- Adequate hydration
- Chamomile steam inhalation

Dr. Cabrera, Diseases of the Nose, Slide #46


47 Acute fulminant fungal rhinosinusitis: Acute Invasive Fungal Rhinosinusitis A
• AKA acute fulminant fungal rhinosinusitis
a. Acute invasive fungal RS
b. Chronic invasive Fungal RS Dr. Lacuesta. Diseases of the Nose Lecture Slide #56
c. Chronic granulomatous fungal RS
d. Allergic fungal RS
48 There is tissue invasion by fungal elements C
greater than 4 weeks duration, with
mucosal inflammatory cell infiltrate:

a. Acute invasive fungal RS


b. Chronic invasive Fungal RS
c. Chronic granulomatous fungal RS
d. Allergic fungal RS

Diseases of the Nose slide 60 DR CABRERA PPT


49 Charcot Leyden crystals: Criteria of Bent and Kuhn for AFS D
– Eosinophilic mucin (Charcot- Leyden crystals)
a. Acute invasive fungal RS – Noninvasive fungal hyphae
b. Chronic invasive Fungal RS – Nasal polyposis
c. Chronic granulomatous fungal RS –characteristic radiologic findings
d. Allergic fungal RS
Dr. Jesson Cabrera. Ppt presentation “Diseases of the Nose”. Slide#62
50 Atrophic Rhinitis organism: Primary: B
• Klebsiella ozaenae
a. Klebsiella pneumoniae
b. Klebsiella ozaenae Dr. Cabrera’s slides NOSE GENERAL LECTURE ppt slide 75
c. Haemophilus influenzae
d. mycobacterium leprae
51 Prior to making a basic ENT examination A general history and physical examination should be included in every head and B
what should be done first? neck evaluation.
a. Have all the necessary
equipment ready Doc Lacuesta’s audio: Should be completed prior to performing basic ENT examination
b. Make a thorough general history
and physical examination Reference: The Head and Neck History and Examination page 3, Boies: Fundamentals of
Otolaryngology: A Textbook of Ear, Nose, and Throat Disease 6/e Adams
c. Apply topical anesthesia
d. Acquire an informed consent
52 How is the head mirror placed on the Head mirror is placed over the dominant eye of the examiner A
examiner’s head?
a. Over the dominant eye of the Reference: Doc Lacuesta’s ppt: Basic ENT Examination page 1
examiner
b. Wherever the light source is
positioned
c. Over the forehead to have
binocular sight
d. Positioning is not important
53 What is the patient’s head position in Patient B.
relation to that of the examiner? ▪ Seated with the head slightly higher than that of the examiner’s head
a. Patient’s head should be at the ❖ Apex of patient’s head should be at least ~4-5 inches higher
same level as that of the examiner than examiner’s head
b. Patient’s head should be higher
than that of the examiner Reference: Doc Lacuesta’s ppt: Basic ENT Examination page 1
ENT: BASIC OTORHINOLARYNGOLOGY FINALS TERM QUIZZES
QUESTION BANK
S.Y. 2020-2021
c. Patient’s head should be lower
than that of the examiner
d. Head position is not important as
the examiner can always tilt the head
upwards
54 What is the optimal position for the Examiner may stand or sit, must not stoop down and must be comfortable. C
examiner?
a. The examiner should be seated Trans 5.0 Basic ENT Examination page 1
b. Examiner must be standing
c. Examiner can either sit or stand
as long as they maintain optimal head
level in respect to that of the patient’s
and will not slouch
d. Examiner must stoop down to
have a commanding view of the field to
be examined
55 What is the optimal distance of examiner’s Focal length of 14 inches must be maintained at all times. C
head from that of the patient?
a. Not important so as long as the Trans 5.0 Basic ENT Examination page 1
examiner can see the field being
examined
b. At the examiner’s comfort
c. At the focal length of the head
mirror being used
d. Approximately 6 inches
56 When doing an anterior rhinoscopy, on Internal (anterior) rhinoscopy B?
which side should the speculum be held? Speculum
a. On the hand opposite to where o Held on the same side of the nasal cavity examined
the head mirror is placed so as not to Ex: examine the patient’s left nasal cavity → examiner uses right hand & vice versa
block the angle of reflection of the light
b. On the contralateral hand of the
nasal cavity being examined (i.e., when
examining the right nostril the speculum
should be held by the examiner at the
left hand)
c. At the examiner’s dominant hand
d. On the hand where the examiner
is more comfortable with
57 In doing a posterior rhinoscopy, which of Indirect nasopharyngoscopy, utilize a nasopharyngeal mirror size 0 B
the following materials / instrument is not
used? BASIC ENT EXAMINATION.slides for lecture.
a. Tongue depressor
b. Small laryngeal mirror
c. Head mirror
d. Sterile gauze to hold the tongue
58 In doing an indirect laryngoscopy, which ● Classical indirect laryngoscopy requires a laryngeal mirror, light source,
of the following materials/instrument is not head mirror and gauze sponge.
used?
a. Gauze Basic Otorhinolaryngology, A Step-by-Step Learning Guide, Part 4, Ch. 17.2, Methods of
b. Light source Examination
c. Tongue depressor
d. A laryngeal mirror
59 In doing otoscopy of the right ear on which The otoscope is always held by the physician in the hand that correlates with the A
hand should the otoscope be held? side of the ear to be examined, ie. the patient's right ear is examined with the
a. Right hand otoscope in the examiner's right hand; the left ear is examined with the otoscope in
b. Left hand the examiner's left hand.
c. Dominant hand
d. Whichever the examiner is Source: ENT Clinical Practice.
comfortable with https://www.racgp.org.au/afpbackissues/2005/200510/200510chang.pdf
60 In doing an indirect laryngoscopy which Mirror is gently slid, avoiding posterior third of the tongue or posterior pharyngeal C
area should be avoided to prevent a gag wall and pressed upward against the soft palate
reflex?
a. Anterior tongue BASIC ENT EXAMINATION of Dr. Lacuesta Trans 5 p4, under Examination of the
b. Soft palate Hypopharynx and Larynx
c. Posterior pharyngeal wall
d. Tonsils
ENT: BASIC OTORHINOLARYNGOLOGY FINALS TERM QUIZZES
QUESTION BANK
S.Y. 2020-2021
61 What is the focal length of your head Head Mirror focal length is 14 inches. C
mirror?
a. 10 inches BASIC ENT EXAMINATION.slides for lecture. Dr. Nicanor Lacuesta. p.4
b. 12 inches
c. 14 inches
d. 16 inches
62 How would you straighten the external To visualize the ear canal, gently grasp the pinna and elevate it superiorly and A
auditory canal of a 21-year old? posteriorly to straighten the canal and allow atraumatic insertion of the otoscopic
a. By pulling on the auricle upward, speculum.
backward, and outward
b. By pulling the pinna downward Cumming’s Otolaryngology Head & Neck Surgery 6th Ed. Chapter 4, p.50
and forward
c. By using the speculum of my
otoscope
d. By pulling on the auricle
downward and backward
63 What can be utilized in place of a head A focusable light on a headband may be substituted for the described head mirror D
mirror and a gooseneck lamp? and light source.
a. A penlight
b. Your otoscope Ch 1, pp 3, Boies Fundamentals of Otolaryngology 6th Ed
c. A drop light
d. A head light
64 When you have focused the head mirror on Examiner directs the area to be examined into the field of view and avoids C
the area you wish to examine how will you repositioning himself.
move the next area to be examined?
a. By moving the lamp to change Dr. Lacuesta's PPT: Basic ENT Examination (Trans 1)
the angle of reflection
b. Placing my head mirror on the
other eye
c. By moving the head of the
patient
d. By turning my head
65 What is the proper way of handling your ● Nasal speculum is held with the thumb and middle finger on the same B
nasal speculum? side of the nasal cavity examined
a. It is held on the opposite side of ● Index finger → grasp the nasal ala with the speculum
the nasal cavity being examined ● Hand should be placed on the patient’s nose to be able to tilt and move
b. The index finger is used to grasp the head whenever needed
the nasal ala against the speculum ● Speculum must be removed without closing it
c. The speculum is closed or
released prior to removal from the nasal Dr. Lacuesta's PPT: Basic ENT Examination (Trans 1)
vestibule
d. The patient’s head is tilted
slightly downward
66 Treatment of seborrheic dermatitis would Management: B
include: ● Topical steroids
a. Topical antibiotics ● Keratolytics
b. Keratolytics
c. Systemic antifungal Dr. Lacuesta’s PPT: Diseases of the external ear
d. Systemic antibiotics
67 What is the treatment for small exostosis in Excision for symptomatic cases. B
the external auditory canal?
a. Observation For small (Di sinabi kung symptomatic hehe) Although most canal exostoses are
b. Excision asymptomatic, patients with canal obstruction greater than 80% can have recurrent
c. Topical steroids episodes of otitis media and related hearing loss.
d. Systemic antibiotics Dr. Lacuesta’s ppt and lecture, S R Whitaker et al. Treatment of external auditory canal
exostoses., https://pubmed.ncbi.nlm.nih.gov/9473067/
68 This comprises 80% of all malignant Squamous cell carcinoma - comprise 80% of all malignant tumors of the auricle B
tumors in the auricle and external auditory and EAC. Metastasis is uncommon.
canal
a. Basal cell carcinoma Reference: Dr. Lacuesta ppt. Diseases of the external ear
b. Squamous cell carcinoma
c. Malignant melanoma
d. Adenoid cystic carcinoma
69 These tumors in the external ear arises Adenoid Cystic Adenoma is Frequently called CYLINDROMA B
from the ceruminous glands and can appear ● arises from ceruminous glands
as yellowish masses ● yellowish smooth masses with numerous dilated vessels
ENT: BASIC OTORHINOLARYNGOLOGY FINALS TERM QUIZZES
QUESTION BANK
S.Y. 2020-2021
a. Exostosis
b. Cylindroma Doc Lacuesta's PPT, Diseases of the External Ear, slide # 54
c. Melanomas
d. Cholesteatomas
70 This group of external ear deformities GROUP I Minor malformations A
exhibit a normal auricle but may exhibit ● Commonly seen in craniofacial dysostoses
some slight variation ● Auricle usually normal but may exhibit some variation
a. Group I ● EAC may be normal but occasionally hypoplastic in its entire length
b. Group II ● TM can be normal with regular canal
c. Group III ● Handle of malleus is often deformed and in abnormal position
d. Group IV ● Inner ear is abnormal in rare cases

Doc Lacuesta's PPT, Diseases of the External Ear, Slide #60


71 In this classification of external ear C
deformity, the ossicles are often absent and
the semicircular canals are involved and
are associated with craniofacial deformities
a. Group I
b. Group II
c. Group III
d. Group IV

Trans 7 - Diseases of the external ear - Dr. Lacuesta, page 7


72 Which of the following is true about type I B
first branchial cleft anomalies?
a. From ectoderm of 1st branchial
arch
b. Represents a duplication of
membranous EAC
c. Direction of the deformity is
posterior, superior, and medial conchal
cartilage
d. Never opens into the EAC itself

Dr. Lacuesta’s lecture on Diseases of the External Ear slide 7


73 This condition of the external ear arises KERATOSIS OBTURANS C
from accumulation of squamous debris ● Diffuse disorder of entire circumference of the medial EAC skin
which makes the ear painful, erythematous, ● Accumulation of squamous debris in the EAC
have granulations and occasional hearing ● Often mistaken as an impacted cerumen
loss ● Bilateral and occur in young age group
a. Canal cholesteatoma ● Primarily affects adults (30-60 y/o)
b. Seborrheic dermatitis ● Painful with hyperemia, granulation, and occasional hearing loss
c. Keratosis obturans
d. Impacted cerumen Doc Lacuesta's PPT, Diseases of the External Ear
74 What is the recommended treatment of the Management of Keratosis Obturans includes: D
condition above? ● Regular debridement
a. Topical antibiotics ○ Atraumatic removal of epithelial debris
b. Antihistamines ○ May need initial softening w/ otic drops / general anesthesia
c. Cerumenolytics ○ May be lifelong
d. Debridement and steroids ● Topical steroids

ENT Trans 7, page 5


75 This condition of the external ear is caused Canal cholesteatoma is a cystic collection of squamous epithelium. Unilateral, with A
by a cystic collection of squamous pain, purulent otorrhea, bony erosion, and periostitis
ENT: BASIC OTORHINOLARYNGOLOGY FINALS TERM QUIZZES
QUESTION BANK
S.Y. 2020-2021
epithelium
a. Canal cholesteatoma External Auditory Canal Cholesteatoma: Clinical Imaging Spectrum | American Journal of
b. Seborrheic dermatitis Neuroradiology http://www.ajnr.org/content/24/4/751
c. Keratosis obturans
d. Impacted cerumen
76 What are possible complications of the Canal cholesteatoma C
condition above? - Unilateral, with pain, purulent otorrhea, bony erosion and periostitis
a. Secondary bacterial infection
b. Hearing loss Dr. Lacuesta, Diseases of the External Ear, Slide # 46
c. Periostitis and bone loss
d. Soft tissue erosion
77 What would be the primary treatment for Canal Cholesteatoma B
the condition above? -Managed by frequent debridement plus topical
a. Cerumenolytics antibiotics
b. Topical antibiotics and -Surgery reserved for complicated cases and failure of
debridement medical therapy
c. Surgery
d. Keratolytics Dr. Lacuesta, Diseases of the External Ear Lecture, Slide #46
78 What would be the treatment of small Small tumors: local wide excision C
squamous cell carcinoma of the external ▪ Include the margin of normal tissue to ensure removal of the
ear? entire carcinoma
a. Complete resection of the ear ▪ Pie incision: if it involves underlying tissue
b. Radiation therapy
c. Wide excision
d. En bloc resection with lymph
node dissection

Diseases of the External Ear slide 50 DR. LACUESTA PPT


79 How is basal cell carcinoma different from Squamous Cell Carcinoma C
squamous cell carcinoma in terms of ● Meatastasis uncommon
clinical progression?
a. Basal cell carcinoma Basal Cell Carcinoma
metastasizes ● Does not metastasize but locally aggressive and may invade cartilage and
b. Squamous cell carcinoma is bone
locally more aggressive
c. Squamous cell carcinoma can Dr. Nicanor Lacuesta. Ppt presentation “Diseases of the External Ear”. Slide# 50-52
metastasize
d. Basal cell carcinoma is locally
less aggressive
80 How many of the cases of aural atresias and Aural atresia and stenoses C
stenoses are bilateral? - ⅓ of cases are bilateral
a. 90%
b. ½ of all cases Dr. Lacuesta’s ppt DISEASES OF THE EXTERNAL EAR slide 57
c. ⅓ of all cases
d. ⅔ of all cases
81 Which of the following is not included in Management of Acute Otitis Media B
the management of acute otitis media? ● Antimicrobial therapy
a. Antimicrobial ○ Amoxicillin/Ampicillin
b. Aural toilette ○ Erythromycin and Sulfisoxazole
c. Supportive therapy ○ Trimethoprim- sulfamethoxazole
d. Tympanocentesis/myringotomy ○ Amoxicillin-clavulanat
○ Cephalosporins
● Supportive management
○ Analgesic
○ Decongestants
○ Antihistamines
○ Antipyretics
○ Local heat
ENT: BASIC OTORHINOLARYNGOLOGY FINALS TERM QUIZZES
QUESTION BANK
S.Y. 2020-2021
● Tympanocentesis/Myringotomy- if medical treatments fail

Diseases of the Middle Ear ENT Lecture by Dr. Lacuesta


82 Which of the following is a sign/symptom Signs and Symptoms: D
of acute otitis media? ● TM opaque, bulging, or congested
a. A pearly white TM on otoscopy ● limited or no mobility of TM on
b. Vertigo ● pneumatic otoscopy
c. Hearing loss ● otorrhea
d. Fever ● otalgia
● fever

Dr. Lacuesta Lecture. Disease of the Middle Ear. Slide # 6


83 Which of the following bacteria is not Microbiologic agents in Acute Otitis Media C
involved in acute otitis media? ● S. pneumoniae
a. Streptococcus pyogenes ● H. influenzae
b. Staphylococcus aureus ● B. catarrhalis
c. Pseudomonas aeruginosa ● S. pyogenes
d. Haemophilus influenza ● S. aureus
● S. epidermidis

Dr. Lacuesta’s Lecture Slides, Diseases of the Middle Ear, slide #8


84 This stage of AOM is characterized by the Stage of Exudation- outpouring of fluid from the dilated permeable capillaries B
formation of effusion in the middle ear
a. Hyperemia Stage of exudation- Outpouring of fluid from the dilated vessels of the
b. Exudation mucoperiosteum – serous in nature containing fibrin, red cells, and polymorphs.
c. Suppuration Exudate fills the tympano- mastoid compartment and due to the pressure exerted
d. Coalescence into the middle ear

The second stage is exudation. Serum, fibrin, red cells, and polymorphonuclear
leukocytes escape into mucous secreted by goblet cells. This exudate fills the
middle ear cavity under pressure. The tympanic membrane becomes very
thickened and bulging, resulting in a conductive hearing loss, pain, and otalgia

Doc Lacuesta’s lecture on Diseases of the Middle Ear, slide 11, Doc Lazo’s lecture on
Disorders of ears, nose and throat, slide 25, Unit Four: Middle Ear Disease Diagnosis. (n.d.).
https://app1.unmc.edu/medicine/heywood/otology/unit4-middle-ear-disease-diagnosis.cfm.
85 Which of the following subareas are not Long standing inflammations of the middle ear cleft, eustachian tube, tympanum, B
affected by chronic otitis media? attic, and the mastoid air-cell system
a. Middle ear cleft
b. Auditory canal Dr. Lacuesta’s ppt
c. Eustachian tube
d. Mastoid air cells
86 This surgical procedure in the removal of Surgical Approach D
cholesteatoma is done en aural or trans ● Atticotomy: transcanal
canal ● Simple mastoidectomy
a. Canal wall-up mastoidectomy ● Canal wall-up procedure (intact wall) with or without facial recess
b. Canal wall-down mastoidectomy approach
c. Myringotomy ● Canal wall-down procedure (canal down): radical or modified radical
d. Atticotomy mastoidectomy, Body procedure

Cummings otolaryngology Otology, Neur-otology and Skull Base Surgery Page 1970
87 Which is an otoscopic finding in cases of Otoscopic findings: cholesteatoma formation: D
cholesteatoma formation? ● Retraction pockets along the posterosuperior portion of the tympanic
a. Clear foul-smelling discharge membrane ( attic cholesteatoma)
b. Bulging tympanic membrane ● Whitish to yellowish flakes
c. Erythematous tympanic ● Erosion of the ossicular bones
membrane ● Yellowish to brownish, foul smelling discharge
d. Aural polyps ● Presence of granulation tissue and/or aural polyp

Doc Lacuesta’s lecture on Diseases of the Middle Ear ppt slide # 27


ENT: BASIC OTORHINOLARYNGOLOGY FINALS TERM QUIZZES
QUESTION BANK
S.Y. 2020-2021
88 What is the proper indication to remove a C
ventilation tube in myringotomy?
a. When there is no more secretions
noted in the middle ear
b. A maximum of 6 weeks duration
prior to removal
c. The tube will eventually fall off
on its own
d. After the completion of
antimicrobial therapy

Doc Lacuesta’s lecture (Audio) on Diseases of the Middle Ear, slide 19 & 21
89 Which of the following conditions is found D
in an “unsafe ear”?
a. Anterior of central tympanic
membrane perforation
b. Infected eustachian tube
c. Well-pneumatized mastoid air
cells
d. Diseased attic or antrum

Doc Lacuesta’s lecture on Diseases of the Middle Ear, slide 34


90 What is the period of time for otitis media • acute - 3 weeks B
with effusion to become subacute? • Subacute- 3 months
a. 3 weeks • Chronic- 6 months
b. 3 months
c. 6 weeks Doc Lacuesta’s lecture on Diseases of the Middle Ear
d. 6 months
91 What is the most common symptom of Otitis media with effusion in children, the most common symptom is hearing loss. A
otitis media with effusion in children?
a. Hearing loss Trans 8.1 Diseases of the Middle Ear page 4
b. Pain
c. Ear fullness
d. Popping sounds in the ear

92 In children, what is the indication in otitis Air in middle ear (bubbles) is a good sign, showing partial function of the C
media with effusion for it to be treated eustachian tube
surgically?
a. Presence of air bubbles Thickened tympanic membrane - must treat surgically
b. Erythematous tympanic
membrane Reference: Doc Lacuesta’s ppt: Diseases of the Middle Ear page 49
c. Thickened tympanic membrane
d. Hearing loss
93 With regards to injuries to the ear, which of Symptoms of severe injury: D
the following symptoms points to ● Presence of vertigo
involvement of the inner ear? ● Nausea
a. Brief pain ● Disequilibrium
b. Air escape on Valsalva ● Facial nerve paralysis
c. Vertigo ● Sensorineural hearing loss signifies involvement of the inner ear
d. Hearing loss
Reference: Doc Lacuesta’s ppt: Diseases of the Middle Ear page 5
94 In otosclerosis of the otic capsule, which Otosclerosis is a disease in the bony otic capsule causing changes that fixates the C
structure is fixated? stapes causing progressive hearing loss.
a. Malleus
b. Incus Trans 8.1 Diseases of the Middle Ear page 5
c. Stapes
d. All of the ossicles
95 Pure tone audiometry of otosclerosis has Cahart’s notch in pure tone audiometry increase threshold at 2k Hz B
hearing loss at what frequency?
a. 500 Hz Doc Lacuesta’s lecture on Diseases of the Middle Ear, slide 71
b. 2000 Hz
c. 4000 Hz
d. 8000 Hz
96 Palatogenesis begins at the end of: Palatogenesis begins at end of 5 th week B
a. 4th week
ENT: BASIC OTORHINOLARYNGOLOGY FINALS TERM QUIZZES
QUESTION BANK
S.Y. 2020-2021
b. 5th week Doc Pasahol’s ppt slide 13
c. 6th week
d. 7th week
97 What gender is cleft lip more common? Boys> Girls A
a. Male
b. Female
c. Both are equally affected Dr. Maria Luisa Pasahol, PPT presentation “Cleft lip and Palate” slide # 30
98 What prenatal diagnostic consideration in Ultrasonography and 3D ultrasonography enables in utero diagnosis of clefts C
cleft lip and palate is important especially especially in 3rd trimester
in the 3rd trimester?
a. Amniotic fluid exam
b. Genetic analysis
c. 3D ultrasonography
d. AOTA

Cleft Lip and Palate DR PASAHOL PPT


99 Why is cleft palate more common in Cleft Palate Epidemiology D
females? -Fusion of palatine shelves 1 week later in girls vs boys - higher incidence of cleft
a. Fusion of palatine shelves is 1 palate in girls
month later in boys than girls
b. Fusion of palatine shelves is 1 Dr. Pasahol, Cleft Lip and Palate Lecture, Slide #31
week later in boys than girls
c. Fusion of palatine shelves is 1
month later in girls than boys
d. Fusion of palatine shelves is 1
week later in girls than boys
100 Cleft lip and palate deformities are the The etiology of clefting is multifactorial and prevalence varies among ethnic D
most common congenital defect of the groups and within families
head. The most common etiology is:
a. Genetic Cummings Otolaryngology Head & Neck Surgery 5th Ed., Part 8, Chapter 186, Page 2659
b. Teratogens
c. Prenatal vitamin deficiency
d. Multifactorial
101 The aim of this first surgery for the patient The goals of cheiloplasty involve reconstruction of the normal anatomy and A
with cleft lip is to restore the entire cleft function of the lip, correction of the nasal deformity, and construction of the floor
defect to as near a normal anatomy as of the nose and proper alignment of the maxillary segment.
possible.
a. Cheiloplasty Cleft Lip and Palate lecture, Dr. Pasahol, https://uichildrens.or/health-library/cleft-palate-
b. Alveoloplasty surgical-procedures
c. Primary rhinoplasty
d. Palatoplasty
102 This is formed when the paired maxillary By 6th and 7th week B
prominences grow medially toward the ● Paired maxillary prominences grow medially toward the paired medial
paired medial nasal prominences. nasal prominences - UPPER LIP
a. Nasal tip ● Maxillary prominences fuse to form the LATERAL ASPECT OF
b. Upper lip UPPER LIP
c. Columella
d. Nasal ala Doc Pasahol’s ppt on Cleft Lip and Palate, page 9
103 The incisive foramen divides the palate ● Incisive foramen – divides the palate into primary palate and secondary D
into primary and secondary palate. The palate
following structures are found posterior to ● Primary palate
the incisive foramen. ○ Anterior to the incisive foramen
a. Premaxilla ○ Consists of: Premaxilla, Lip, Nasal tip, columella
b. Lip ● Secondary palate
c. Columella ○ develops after completion of the primary palate
d. NOTA ○ Extends from incisive foramen anteriorly to uvula posteriorly

Doc Pasahol’s ppt on Cleft Lip and Palate


104 The age of development when there is ● Palatogenesis begins at end of 5th week C
complete fusion of the palate. ● Complete fusion at 12 weeks
ENT: BASIC OTORHINOLARYNGOLOGY FINALS TERM QUIZZES
QUESTION BANK
S.Y. 2020-2021
a. 4th week ● Medial nasal prominences fuse at the surface and deeper levels
b. 8th week ● Intermaxillary segment or primary palate
c. 12th week ○ Central maxillary alveolar arch
d. 16th week ○ 4 incisor teeth
○ Hard palate
● Formed by deeper level fusion of the medial nasal prominences

Doc Pasahol’s ppt on Cleft Lip and Palate slide 9


105 A male newborn baby is referred to you due In unaffected persons, the orbicularis oris muscle forms a complete sphincter B
to lip deformity. What muscle is affected in around the oral cavity, providing the substrate for proper form and function of the
complete cleft lip? lips and mouth. For proper correction of cleft lip deformities, it is essential not
a. Orbicualris oculi only to create symmetry of the lip superficially, at the skin level, but also to
b. Levator labii superioris recreate the complete orbicularis muscular sling, for long-lasting cosmetic and
c. Risorius functional results.
d. Orbicularis oris Cummings Otolaryngology Head & Neck Surgery 5th Ed., Part 8, Chapter 186, Page 2665
How is audiometric zero defined? Audiometric zero (ANSI) D
106 a. The total absence of sound ● The median average threshold of a large sample of young adults with no
b. The median average hearing hearing complaints, no history of ear disease, & no recent colds
threshold of the general population
c. The median average hearing Audiology - Dr. Lacuesta recorded lecture at 41:52
threshold of young children with no history
of ear infection or recent colds
d. The median average hearing
threshold of young adults with no history of
hearing problem, ear infection, or recent
colds
107 What is the maximum decibels for a normal C
hearing threshold?
a. 10 dB
b. 15 dB
c. 20 dB
d. 25 dB

Probst, R.P., Grevers, G., & Iro, H. (2006). Basic Otorhinolaryngology. Chapter
8.4, page 177.
108 What is the criteria for mixed type of A
hearing loss on an audiometry?
a. When bone conduction
thresholds are reduced but are still better
than air conduction by 10 dB or more
b. When bone conduction
thresholds are better than air conduction
by 10 dB or more & are normal
c. When bone conduction
thresholds are the same as air conduction Audiology - Dr. Lacuesta lecture/ Trans 11 page 4
and neither is normal
d. NOTA
109 This test is used to measure the compliance ● Tympanometry provides a graphic representation of the impedance B
of the tympanic membrane changes caused by applied air pressure in the external ear canal.
a. Otoacoustic emission testing ● The tympanogram is a graphic representation of compliance changes
b. Tympanometry as the applied air pressure is varied over a negative-to-positive range.
c. Pure tone audiometry
d. Speech discrimination test Probst, R.P., Grevers, G., & Iro, H. (2006). Basic Otorhinolaryngology. Chapter 8.4, page
185.
110 Which of the following is not used as a Tuning Fork Tests uses the examiner’s ear as the reference comparison, therefore D
reference comparison when doing a tuning necessitating that the examiner have normal hearing. Under it, Weber’s Test uses
fork test? the patient’s normal ear for basis of lateralization. Schwabach’s Tests requires a
a. Examiner’s normal ear normal reference as comparison.
b. Patient’s normal ear
c. A third person with normal ear Dr. Lacuesta, Audiology Recorded Lecture, Time Stamp 23:12
d. A decibel meter
111 In Weber’s test, when the patient reports the Weber's Test Lateralization B
sound to lateralize to the better ear, the loss To the poorer ear - Conductive
ENT: BASIC OTORHINOLARYNGOLOGY FINALS TERM QUIZZES
QUESTION BANK
S.Y. 2020-2021
is described to be: To the better ear - Sensorineural
a. Conductive Midline - Normal, bilateral sensorineural, equal conductive hearing loss
b. Sensorineural
c. Mixed Dr. Lacuesta, Audiology Video Lecture, Time Stamp 23:55
d. Normal hearing
112 In Rinne’s test, if air conduction is longer B
than bone conduction, the loss is described
to be:
a. Conductive
b. Sensorineural Dr. Lacuesta, Audiology Video Lecture, Time stamp 28:55
c. Mixed
d. Normal
113 In Schwabach’s test, a prolonged result has A
a hearing loss that is described as:
a. Conductive
b. Sensorineural
c. Mixed
d. Normal

Audiology - Dr. Lacuesta recorded lecture at 31:11


114 In Bing’s test, a positive result can be Bing test B
interpreted as: a tuning fork test in which the vibrating fork is held against the mastoid process
a. Conductive hearing loss and the auditory meatus is alternately occluded and left open; an increase and
b. Normal hearing decrease in loudness (positive Bing) is perceived by the normal ear and in
c. Mixed hearing loss sensorineural hearing loss, whereas lack of a difference in loudness (negative
d. NOTA Bing) is the perception in conductive hearing loss

https://medical-dictionary.thefreedictionary.com/Bing+test
115 In Bing’s test, a negative result can be Positive Bing = sensorineural B
interpreted as: Negative Bing = conductive
a. Sensorineural hearing loss
b. Conductive hearing loss Dr. Lacuesta’s ppt, Trans 11 pg 3
c. Mixed hearing loss
d. NOTA
116 This tuning for test compares the patient’s Schwabach’s Test compares the patient’s bone conduction against that of a C
bone conduction against that of a reference: reference.
a. Rinne’s
b. Webber’s Dr. Lacuesta’s Lecture. Trans Basic Audiology Page 3
c. Schwabach’s
d. Bing’s
117 What is the main advantage of pure tone Pure tone audiometry A
audiometry over tuning fork test in the It samples the octave series of the C scale (125,250,500,1000, 2000, 4000, 8000
evaluation of a patient’s hearing? Hz)
a. PTA sample the octave series of
the C scale (125, 250, 500, 1000, 2000,
4000, 8000 Hz) Dr. Lacuesta’s Lecture. Basic Audiology Slide #23, Lecture at 34:52
b. Tuning forks can maintain a
steady tonal intensity
c. The point of cessation of the tone
can be controlled and is exact in PTA
d. Tuning fork test can produce
“overtones”
118 How much of the presented words in The Speech Reception Threshold represents the intensity level at which a listener C
Speech Reception Threshold must be can repeat 50% of the speech material.
correct for it to be normal?
a. 100% Cummings Otolaryngology, 5th Ed, Chapter 133 Diagnostic Audiology Page 1888
b. 94-100%
c. 50%
d. 99-100%
119 What is the criteria of conductive hearing Conductive Hearing Loss: BC threshold better than AC by ≥10 db and are normal C
loss on an audiometry?
ENT: BASIC OTORHINOLARYNGOLOGY FINALS TERM QUIZZES
QUESTION BANK
S.Y. 2020-2021
a. When bone conduction Doc Lacueta, Audiology Recorded Lecture at 42:15
thresholds are the same as air conduction
and neither is normal
b. When bone conduction
thresholds are reduced but are still better
than air conduction by 10 dB or more
c. When bone conduction
thresholds are better than air conduction
by 10 dB or more & are normal
d. NOTA
120 What is the criteria for sensorineural When bone conduction thresholds are the same with air conduction & neither is C
hearing loss on an audiometry? normal, the loss is sensorineural
a. When bone conduction
thresholds are better than air conduction Doc Lacueta, Audiology Recorded Lecture at 44:09
by 10 dB or more & are normal
b. When bone conduction
thresholds are reduced but are still better
than air conduction y 10 dB or more
c. When bone conduction
thresholds are the same as air conduction
& neither is normal
d. NOTA
121 How many teeth are there in adult person? The normal adult complement of teeth is 32, with 8 in each quadrant of the maxilla A
a. 32 and mandible.
b. 34
c. 36 Cummings Otolaryngology Chapter 23 Maxillofacial trauma Page 323
d. 38
122 This provides the blood supply for the teeth Teeth Arterial supply: C
a. Facial artery ● Inferior alveolar artery
b. Ethmoid artery ● Anterior and posterior superior alveolar arteries (maxillary artery)
c. Maxillary artery
d. Ophthalmic artery Dr Lacuesta :The Oral Cavity, oropharynx and hypopharynx PPT slide #20
123 The lower teeth receive their innervation Teeth innervation - branches of the maxillary nerve, the lower teeth from branches D
from which nerve? of the mandibular nerve
a. Facial nerve
b. Ethmoid nerve Dr Lacuesta : The Oral Cavity, oropharynx and hypopharynx PPT slide #19
c. Maxillary nerve
d. Mandibular nerve
124 This structure smooths and fills in the Bichat fat pad (Buccal fat pad) is located between the buccinator muscle and the B
depression created a muscle of mastication overlying masseter muscle. It smoothes the cheek contour by filling in the
a. Masseter muscle depression at the anterior border of the masseter muscle.
b. Bichat fat pad
c. Buccinator muscle Dr. Lacuesta ppt lecture on the oral cavity, oropharynx, and hypopharynx, slide 14.
d. Subcutaneous fat
125 Which of the following muscle is not found The palatal muscles that form the framework of the soft palate are the tensor veli NOTA?
in the oral cavity? palatini and especially the levator veli palatini, which elevates the soft palate
a. Levator palatini muscle during swallowing to keep food from entering the nose. The muscles of the soft
b. Tensor palatini muscle palate are completed by the palatoglossus, which runs in the anterior faucial pillar
c. Palatopharyngeus muscle (palatoglossal arch), and by the palatopharyngeus muscle of the posterior faucial
d. Palatoglossus muscle pillar
**i am not sure, i think there is no
correct answer. All muscles Basic Otolaryngology, Part II, Chapter 4, page 81
mentioned above are found in the
oral cavity.
126 What type of tooth creates the greatest Greatest importance in chewing D
pressure for mastication? Located closest to the insertion of the masticatory muscles - allows very high
a. Incisors pressure to be developed between their occlusive surfaces
b. Canine
c. Premolars Dr. Lacuesta ppt lecture on the oral cavity, oropharynx, and hypopharynx, slide 30.
d. Molars
127 Which of the following is true about Primary infection: D
primary infection with HSV type 1? • usually acquired in early childhood and predominantly affects the oral
a. Does not usually present with mucosa as herpetic gingivostomatitis (aphthous stomatitis)
regional lymph node enlargement •preceded by fever and lethargy (flu like symptoms)
b. Has no prodrome •Can be accompanied by regional lymphadenitis
c. Nasal mucosa is never involved •nasal mucosa can be involved (herpetic rhinitis)
ENT: BASIC OTORHINOLARYNGOLOGY FINALS TERM QUIZZES
QUESTION BANK
S.Y. 2020-2021
d. Usually affects oral mucosa as •Severe forms: Pospischill Feyrter aphthoid, can occur in
herpetic gingivostomatitis immunocompromised children or as a sequel to measles, rubella, or
chickenpox can also occur in immunocompromised adults (HIV)

Dr. Lacuesta ppt lecture on the oral cavity, oropharynx, and hypopharynx, slide 45.
128 Secondary S. aureus infection in HSV • A feared complication is secondary bacterial superinfection by D
infection is otherwise known as Staphylococcus aureus or streptococci.
a. Herpes labialis • Also known as herpes impetiginatus, this infection frequently heals by
b. Herpes rhinitis scarring, in contrast to non-superinfected cases
c. Apthous stomatitis
d. Herpes impegtinatus Section 2, Chapter 4.4 Inflammations of the lips and oral cavity, Basic Otolaryngology ,
page 80
129 What if the treatment for HSV infection? Treatment for HSV: C
a. Oral prednisone ● Topical antiseptic to prevent superinfection
b. Amoxicillin ● Acyclovir - given for 5-7 days (if immunocompromised, 14 days)
c. Acyclovir
d. Ibuprofen Trans 13.1 : The Oral Cavity, pp 6
130 Which of the following is the difference • Gingivostomatitis is more painful and runs a longer clinical course. B
between herpangina and gingivostomatitis
(HSV)? • Herpangina resolves within 14 days.
a. Gingivostomatitis is less painful
b. Herpangina is self limiting Trans 13.1 : The Oral Cavity, pp 6.
c. Herpangina runs a longer course
d. Gingivostomatitis usually last for
14 days
131 Hand-foot-mouth disease is caused by? Hand-foot-and-mouth disease, most commonly caused by C
a. HSV Coxsackievirus
b. VZV
c. Coxsackie Virus Cumming Pediatric otolaryngology page 235
d. CMV
132 This viral disease occasional presents VARICELLA-ZOSTER VIRUS B
neuralgiform pain Accompanied by systemic signs (e.g., lethargy, fatigue, and occasional
a. HSV neuralgiform / neuropathic pain [piercing, burning sensation] in the distribution of
b. VZV the affected nerve
c. Coxsackie
d. CMV Dr. Lacuesta ppt lecture on the oral cavity, oropharynx, and hypopharynx, slide 52
133 This inflammatory disease has been linked Recurrent aphtous ulcer D
to iron, folic of vit.B12 deficiency ● May be caused by VZV, CMV
a. Herpangina ● Precipitating factors: minor trauma, hormonal changes, concomitant
b. Varicalla-zoster infection GIT disease, emotional stress
c. Primary herpes simplex infection ● Linked to iron, folic-acid or vitamin B12 deficiency
d. Recurrent aphtous ulcer
Basic Otorhinolaryngology, Part 2, Sec. 4, Ch. 4.4, p 83
134 This inflammatory disease can be Recurrent aphthous ulcer D
controlled by correcting deficiencies such
as iron deficiency Various precipitating factors have been identified : minor trauma, hormonal
a. Herpangina changes, concomitant GI diseases and emotional stress. Recurrent aphthous ulcers
b. Varicella-zoster infection have been linked to IRON, folic acid or B12 deficiency.
c. Primary herpes simplex infection
d. Recurrent aphtous ulcer Slide 60 : Deficits can be corrected by means of Iron, folic acid/ vitamin B12
replacement.

Dr. Lacuesta, Oral Cavity, Oropharynx and Hypopharynx slide, Number 58 and slide 60
135 Oral hairy leukoplakia is believed to be Oral hairy leukoplakia is caused by EBV, and its presence in an otherwise A
caused by this virus asymptomatic patient is a strong indicator of a diagnosis of HIV infection with
a. EBV moderate to severe immunosuppression
b. HSV
c. Coxsackie Cumming’s Otolaryngology Head & Neck Surgery 5th ed Ch 15 pg 225
d. CMV
136 Majority of the oropharynx and At the oropharynx this gives way to a stratified, nonkeratinized squamous B
hypopharynx is lined with what type of epithelium,which also lines the hypopharynx.
epithelium?
a. Ciliated pseudostratified Basic Otorhinolaryngology.Probst et al. Ch.5. p. 98
columnar epithelium
b. Nonkeratinized stratified
squamous epithelium
c. Keratinized stratified squamous
ENT: BASIC OTORHINOLARYNGOLOGY FINALS TERM QUIZZES
QUESTION BANK
S.Y. 2020-2021
epithelium
d. Simple columnar epithelium
137 Which portion would have fibers of the Most of the constrictor pharyngis muscle fibers run obliquely, the lowest portions C
pharyngeal muscles would have fibers that of the constrictorpharyngis inferior (cricopharyngeal part) run directly horizontally
run a transverse direction? anatomical weak spots in the pharyngeal wall.
a. Constrictor superioris
b. Constrictor medius THE ORAL CAVITY, OROPHARYNX, AND HYPOPHARYNX PPT by Dr.Lacuesta, p.137,
c. Constrictor inferioris Basic Otorhinolaryngology, Chapter 5, p.98-99
d. NOTA
138 This weak point of the pharyngeal WEAK POINTS IN THE PHARYNGEAL MUSCULATURE A
musculature occurs between the constrictor 1. Killian triangle: between the constrictor pharyngis inferior and the uppermost
pharyngis inferior and the uppermost fibers fibers of the cricopharyngeus muscle - common site for the formation of
of the cricopharyngeus muscle. hypopharyngeal diverticula
a. Killian triangle 2. Killian–Jamieson region: between the oblique and transverse fibers of the
b. Killian-Jamieson triangle constrictor pharyngis.
c. Lamier’s triangle 3. Laimer triangle, which is bounded above by the cricopharyngeus and below by
d. Killian-Jamieson Region the uppermost fibers of the esophageal musculature

Dr. Lacuesta Lecture; The oral cavity, oropharynx, and hypopharynx; Slide # 138
139 This weak point of the pharyngeal Weak points in the pharyngeal musculature: C
musculature is bounded above by the 1. Killian triangle - between the constrictor pharyngis inferior and the uppermost
cricopharyngeus and below by the fibers of the cricopharyngeus muscle
uppermost fibers of the esophageal 2. Killian–Jamieson region -between the oblique and transverse fibers of the
musculature constrictor pharyngis.
a. Killian triangle 3. Laimer triangle - bounded above by the cricopharyngeus and below by the
b. Killian-Jamieson triangle uppermost fibers of the esophageal musculature
c. Lamier’s triangle
d. Killian-Jamieson Region Dr. Nicanor Lacuesta's lecture ppt - Oral Cavity, Oropharynx, and Hypopharynx - Slide
#138.
140 Which of the tonsils is covered with The Palatine and Lingual tonsils are covered by stratified, nonkeratinized A&D
stratified squamous epithelium? squamous epithelium.
a. Palatine tonsils
b. Tubulopharyngeal tonsils Pharyngeal tonsil is covered mainly by multiple rows of ciliated epithelium,
c. Pharyngeal tonsils
d. Lingual tonsils Dr. Nicanor Lacuesta's lecture ppt - Oral Cavity, Oropharynx, and Hypopharynx - Slide
#144
141 In doing a chest radiogram in a patient who Chest / Abdomen radiograph is done to exclude a perforation of the esophagus or B
ingested caustic fluids, what are we actually stomach.
investigating?
a. Aspiration pneumonia Trans 13.2 : Oropharynx and Hypopharynx, pp 10.
b. Esophageal perforation
c. Esophageal stenosis
d. Extent of mucosal erosion
142 In accidental foreign body ingestion, where In oropharynx, Foreign bodies are commonly located at the tonsils or the base of C/B
are they usually embedded / found? the tongue.
a. Pyriform sinus
b. Vallecula Frequent lodging sites in the oropharynx are the tonsils, tongue base, valleculae,
c. Palatine tonsil and pyriform recesses, the tonsils being the most common site
d. Vestibular folds
The most common sites of throat FBs were the tongue root (42.29%), epiglottic
vallecula (19.40%), tonsil (18.21%), and piriform fossa (10.65%).

Ref: Clinical diagnosis and treatment of throat foreign bodies under video laryngoscopy.
https://journals.sagepub.com/doi/full/10.1177/0300060520940494, Dr. Lacuesta’s ppt slide
#153
143 This acute inflammation condition of the Present with a rash that begins on the trunk. The area around the mouth is spared D
pharynx is characterized by rash formation (“perioral pallor”). A pathognomonic feature is a bright red tongue with a
over the entire body with perioral pallor glistening surface and hyperplastic papillae “raspberry tongue” . The tonsils are
a. Streptococcal angina greatly swollen with a deep red color.
b. Plaut-Vincent angina
c. Diphtheria
d. Scarlet fever
144 This acute inflammation of the pharynx is Caused by Corynebacterium diphtheriae, transmitted by droplet inhalation or C
caused by a bacteria that produces a toxin skin-to-skin contact with an incubation period of 1–5 days.
that causes cell necrosis and ulcerations. • bacterium produces a special endotoxin that causes epithelial cell necrosis
a. Streptococcal angina and ulcerations
b. Plaut-Vincent angina
ENT: BASIC OTORHINOLARYNGOLOGY FINALS TERM QUIZZES
QUESTION BANK
S.Y. 2020-2021
c. Diphtheria Dr. Lacuesta Lecture; The oral cavity, oropharynx, and hypopharynx; Slide # 168
d. Scarlet fever
145 Diverticula in the hypopharynx is usually of HYPOPHARYNGEAL DIVERTICULUM(Zenker diverticulum) D
what type? Epidemiology
a. Traction diverticulum •most common diverticulum of the esopha geal inlet
b. Pulsion diverticulum •Most patients are middle aged or older, 3 : 1 preponderance of males over
c. Both can occur in the Females
hypopharynx
d. NOTA Dr. Lacuesta; The oral cavity, oropharynx, and hypopharynx; Slide # 209
146 This layer of the cervical fascia invests or The superficial cervical fascia (synonym: superior layer of cervical fascia) A
covers the sternocleidomastoid and underlies the platysma and subcutaneous fat, invests the entire neck, and encases
trapezius muscle hence the name ‘investing the sternocleidomastoid and trapezius muscles.
fascia’
a. Superficial fascia Basic Otorhinolaryngology A Step-By-Step Learning Guide, Chapter 16.1, Page 313
b. Middle fascia
c. Deep fascia
d. Neurovascular sheath
147 This layer of the cervical fascia is otherwise This fascial layer is attached to the hyoid bone and stretches superiorly to the B
known as the pretracheal fascia mandibular border and inferiorly to the manubrium sterni and clavicle. It is fused
a. Superficial fascia at the midline to the middle cervical fascia ( synonym: pretracheal layer of the
b. Middle fascia cervical fascia) this fascial layer stretches between the hyoid bone, the posterior
c. Deep fascia surface of the manubrium sterni, and the clavicle and extends laterally to the
d. Neurovascular sheath omohyoid muscle and scapula. It encases the infrahyoid muscles and forms a
general anterior boundary for the cervical viscera.

Basic Otorhinolaryngology A Step-By-Step Learning Guide, Chapter 16.1, Page 313


148 This layer of the cervical fascia is otherwise Deep cervical fascia (prevertebral fascia) C
known as the prevertebral fascia • Extends from the spinous processes of the cervical spine and forms a
a. Superficial fascia tube encasing the deep neck muscles
b. Middle fascia • Adherent posteriorly to the superficial cervical facia that encases the
c. Deep fascia trapezius muscle
d. Neurovascular sheath • Extends from the base of the skull all the way to the end of the spinal
column (prevertebral gravitational abscess)

Dr. Nicanor Lacuesta. Ppt presentation “The neck”. Slide#5


149 This layer of cervical fascia receives Neurovacular Sheath D
contribution from all fascia layers • Carotid artery, jugular veins, and vagus nerve are encased by its own
a. Superficial fascia connective tissue sheath – contribution from all fascial layers.
b. Middle fascia • Attached to the middle cervical fascia at the tendon between the
c. Deep fascia superior and inferior bellies of the omohyoid muscles
d. Neurovascular sheath
Dr. Lacuesta’s ppt on the Neck, Slide #6
150 This cervical fascia layer encases the Middle Cervical Fascia (Pretracheal Fascia) B
infrahyoid muscles ● Stretches from the hyoid to the posterior border of the mandible, and
a. Superficial fascia inferiorly to the posterior surface of the manubrium sterni and the clavicle.
b. Middle fascia ● Extends laterally to the omohyoid muscles and the scapula
c. Deep fascia ● Encases the infrahyoid muscles
d. Neurovascular sheath ● Forms the anterior boundary of the cervical fascia

The Neck, lecture by Dr. Lacuesta, page 4


151 Which of the following is not true about • Branchial cleft cyst and sinuses manifest at the age of 15 to 25 years A
branchial cleft cysts and sinuses?
a. Branchial cleft sinuses are • Result from the persistence or incomplete regression of the cervical
usually diagnosed later in life • sinus (cervical sinus theory)
b. Result from the persistence or
incomplete regression of the cervical • Sinuses that open internally are remnants of the second branchial
sinus • pouch
c. Sinuses that open internally are
remnants of the second branchial pouch • Sinuses that open externally are remnants of the second branchial cleft
d. Sinuses that open externally are
the remnants of the second branchial Dr. Lacuesta, The Neck ppt, Slide # 17, 20
cleft
ENT: BASIC OTORHINOLARYNGOLOGY FINALS TERM QUIZZES
QUESTION BANK
S.Y. 2020-2021
152 Which of the following is not true about B
branchial cleft sinuses?
a. Marked by clear amber colored
discharge
b. Open posterior to SCM, 1 or 2
fingerbreadths above the
sternoclavicular joint
c. Can have accessory cartilage at
the opening
d. If infected-can have purulent Dr. Lacuesta. The Neck PPT, Slide #22
discharge, erythema or infection
153 Which of the following is true about D
hemangiomas?
a. Incidence of 30% during the 1st
year of life
b. Premature babies are seldomly
affected
c. Superficial hemangiomas are
more common in males
d. NOTA

Dr. Nicanor Lacuesta. Ppt presentation “The neck”. Slide#34


154 What is a possible problematic outcome of Kasabach-Merritt Sequence (thrombopenia-hemangioma sequence) B
Kasabach-Merritt sequence? • Large hemangiomas in which thrombotic processes lead to DIC with
a. Clot formation consumption coagulopathy
b. Disseminated intravascular • Affects small infants exclusively
coagulopathy
c. Causes significant necrosis to Dr. Nicanor Lacuesta. Ppt presentation “The neck”. Slide#38
surrounding tissues due to pressure
d. Persistence beyond childhood
155 This dysontogenic tumor is formed by Dermoids = composed mainly of ectodermal elements with complete absence of D
encapsulation of ectodermal elements endodermal tissue
during development.
a. Teratoma Dr. Lacuesta ppt The Neck slide 46
b. Hamartomas
c. Embryonal tumors
d. Dermoids

- GOD BLESS! LABAN MEDISINA! PARA SA BAYAN -


RECALLS: EARS, NOSE, AND THROAT
Midterms Examination ● 2nd Semester

Midterms Examination
QUESTION ANSWER/RATIONALE
1. Which of the following equipment are used in indirect ANSWER:
mirror nasopharyngoscopy? C. Nasopharyngeal mirror, sterile gloves, tongue depressor, head
A. Nasopharyngeal mirror, sterile gloves, sterile gauze, head mirror, gooseneck lamp
mirror, gooseneck lamp
Equipments used in indirect nasopharyngoscopy
B. Laryngeal mirror, sterile gloves, sterile gauze, head mirror, o Gloves
gooseneck lamp o Head mirror
C. Nasopharyngeal mirror, sterile gloves, tongue depressor, o Gooseneck lamp
head mirror, gooseneck lamp o Nasopharyngeal mirror
D. Laryngeal mirror, sterile gloves, tongue depressor, head o Tongue depressors
mirror, gooseneck lamp [ENT] T.01 - Basic ENT Examination p.4
2. Which of the following materials are used in indirect mirror ANSWER:
laryngoscopy? B. Laryngeal mirror, sterile gloves, sterile gauze, head mirror,
A. Laryngeal mirror, sterile gloves, tongue depressor, head gooseneck lamp
mirror, gooseneck lamp
Equipments used in indirect laryngoscopy
B. Laryngeal mirror, sterile gloves, sterile gauze, head mirror, o Gloves
gooseneck lamp o Head mirror
C. Nasopharyngeal mirror, sterile gloves, sterile gauze, head o Gooseneck lamp
mirror, gooseneck lamp o Laryngeal mirror
D. Nasopharyngeal mirror, sterile gloves, tongue depressor, o Sterile gloves
head mirror, gooseneck lamp [ENT] T.01 - Basic ENT Examination p.4
3. To straighten the external auditory canal in adults, how is ANSWER:
the auricle retracted? B. Superiorly, posteriorly, laterally
A. Inferiorly, posteriorly, laterally
Pull backward and upward in adults
B. Superiorly, posteriorly, laterally
C. Superiorly, anteriorly, laterally
D. Inferiorly, posteriorly, laterally [ENT] T.01 - Basic ENT Examination p.2
4. What is the focal distance of the head mirror used in ENT ANSWER:
examination? 4. A. 14 inches
A. 14 inches 5. B. ½ in
B. 13 inches
Head mirror
C. 12inches  3 1⁄2 inch mirror with a 1⁄2 inch hole at the center
D. 10 inches  Focal length of 14 inches
5. The head mirror has a hole at the middle which has a  Should be placed as close as possible to the face – wider angle of view
diameter of?  View should be binocular
A. 1 inch  Examiner directs the area to be examined into the field of view and
B. ½ in avoids repositioning himself
C. ¾ in  Substitute – focusable light on a headband
D. ¼ in
[ENT] T.01 - Basic ENT Examination p.1
6. What is the best position for the patient when doing an ENT ANSWER:
examination? A. Sitting with back straight and slightly leaning forward
A. Sitting with back straight and slightly leaning forward
Position (Patient)
B. Lying down
 Seated with the head slightly higher than that of the examiner’s head
C. Sitting with a slouched back
 Slightly leaning forward and back straight
D. Any position, as long as both patient and examiner are  Legs should be uncrossed and flat on the floor
comfortable
[ENT] T.01 - Basic ENT Examination p.1
7. At what AOG does the external ear achieve its adult shape? ANSWER:
A. 12th wk D. 20th wk
B. 14th wk  7th week – cartilage formation
C. 16th wk  12th week – auricle formed
D. 20th wk  20th week: adult shape
 adult size by 9-year-old

[ENT] T.02 - Anatomy and Physiology of the External Ear p.3


8. Sensory innervation of the floor of EAC ANSWER:
A. CN X A. CN X
B. CN VII
External Auditory Canal Innervation
C. C3
 Anterior – C3
D. CN V
 Posterior – CN V
 Superior – CN VII
 Floor – CN X

[ENT] T.02 - Anatomy and Physiology of the External Ear p.3

1 | 7 EARS, NOSE, AND THROAT Midterms Examination Editors | ENT TRANS TEAM
9. Major blood supply to the EAC ANSWER:
A. External carotid artery A. External carotid artery
B. Superficial temporal artery
Major blood supply to auricle and EAC: External Carotid Artery
C. Internal carotid artery
D. Anterior auricular artery [ENT] T.02 - Anatomy and Physiology of the External Ear p.4
10. A possible route of spread for tumors in the EAC into the ANSWER:
parotid B. Fissure of Santorini
A. Foramen of Huschke
Two patterns of Cancer spread in the EAC
B. Fissure of Santorini
 Fissures of Santorini: vestigial lymph channels that drain into
C. Stylomastoid foramen superficial PG (parotid gland); Provide avenues of spread to the
D. Tympanomastoid suture line superficial lobe of the gland.
 Foramen of Huschke: Serve as a means for extension of malignant
tumors from the EAC to the deep lobe of the parotid gland.

[ENT] T.02 - Anatomy and Physiology of the External Ear p.4


11. In which situation is aural toilette or aural irrigation not ANSWER:
advisable when clearing impacted cerumen in the auditory C. When there is a suspected or known tympanic membrane
canal? perforation
A. In uncooperative patients
Impacted Cerumen
B. In pediatric patients as the experience can prove to be  Water irrigation can be done
traumatic o Not done if TM is perforated
C. When there is a suspected or known tympanic membrane o Do not do aural toilette (“flushing”) if there is a suspected TM
perforation perforation so always get a thorough patient’s history. Ask for
D. In dry, hard cerumen since it cannot be readily removed discharge, ear pain, and diagnosis of middle ear problems among
others.

[ENT] T.03 - Diseases of the External Ear p.1


12. In removing impacted cerumen from a patient’s ear, ANSWER:
which of the following is true? B. Hardened cerumen can be softened with oil
A. Blind curettage can be done with experienced clinicians
Hardened cerumen can be softened using ear drops (e.g., docusate
B. Hardened cerumen can be softened with oil
sodium, paradichlorobenzene), oil, or hydrogen peroxide.
C. Ear irrigation can be repeatedly done with elderly patients
D. Repeated ear irrigation can remove even hardened cerumen [ENT] T.03 - Diseases of the External Ear p.1
13. In lacerations of the auricle, which of the following ANSWER:
management is true? A. Antibiotics
A. Antibiotics
Management of lacerated/incised wounds of the auricle:
B. All layers should be sutured
 Cleaning
C. Secondary intention healing
 Suturing
D. Anti-inflammatory  Antibiotic therapy
 Daily wound care
[ENT] T.03 - Diseases of the External Ear p.2
14. Hematoma in the external ear, untreated can result in this ANSWER:
unwanted outcome A. Cauliflower-ear deformity
A. Cauliflower-ear deformity
Hematomas of the external ear are commonly seen in wrestlers and
B. Resorption of the underlying cartilage
boxers. If untreated it may lead to “cauliflower ears” (auricular
C. Suppurative chondritis chondropathy).
D. Necrosis of the overlying skin

[ENT] T.03 - Diseases of the External Ear p.2


15. A 40-year-old surfer has a gradually growing lesion on his ANSWER:
right auricle for 2 years. On inspection, there is a noted B. Wide excision with frozen section
5x3 darkly pigmented lesion on the antihelix with a central
Basal Cell Carcinoma
area of ulceration and a raised whitish margin and
 It is hyperpigmented and in the edges there are white lines that we call
apparent erosion of the underlying cartilage. What is the white growth.
mode of treatment?  Management: Wide surgical excision
A. Systemic antibiotics
B. Wide excision with frozen section
C. Symptomatic treatment
D. Cauterization [ENT] T.03 - Diseases of the External Ear p.7
16. This condition in the external ear can be life-threatening. ANSWER:
A. Necrotizing otitis externa A. Necrotizing otitis externa
B. psoriasis
Potentially lethal infection of EAC and surrounding structures.
C. Relapsing polychondritis
D. Ramsay hunt syndrome [ENT] T.03 - Diseases of the External Ear p.5
17. What is the treatment for small exostosis in the external ANSWER:
auditory canal? B. Excision
A. Observation
Excision for symptomatic cases
B. Excision
C. Topical steroids
D. Systemic antibiotic [ENT] T.03 - Diseases of the External Ear p.6

2 | 7 EARS, NOSE, AND THROAT Midterms Examination Editors | ENT TRANS TEAM
18. Furunculosis in the external auditory canal can be treated ANSWER:
by which of the following ways? B. Systemic antibiotics and analgesics
A. Topical antibiotics and anti-inflammatory ear drops
Incision and drainage are the best option
B. Systemic antibiotics and analgesics
Systemic antibiotics (antistaphylococcal) and analgesics
C. Avoid incising or draining the abscess as this may spread
infection to the confluent areas
D. Antibiotics are targeted at anaerobic bacteria [ENT] T.03 - Diseases of the External Ear p.3
19. Found at the posterior boundary of the middle ear ANSWER:
A. Tegmen 19. B. Aditus
B. Aditus 20. D. Eustachian tube
21. A. Tegmen
C. Internal Jugular Vein
22. C. Internal Jugular Vein
D. Eustachian tube
20. Located at the anterior boundary of the middle ear
A. Tegmen Boundaries of Middle Ear
B. Aditus  Superior/Roof – epitympanum, tegmen tympani
C. Internal Jugular Vein  Inferior/Floor – hypotympanum, Interior Jugular Vein
D. Eustachian tube  Lateral – Tympanic membrane, scutum
21. Superior boundary of the middle ear  Anterior – Eustachian tube, canal for the TT muscle (Huguier’s Canal)
A. Tegmen  Posterior – Aditus
B. Aditus  Medial – promontory (basal turn of cochlea), oval and round windows,
horizontal portion of CN VII, LSCC
C. Internal Jugular Vein
D. Eustachian tube
22. Inferior boundary of the middle ear
A. Tegmen
B. Aditus
C. Internal Jugular Vein
D. Eustachian tube
[ENT] T.03 - Diseases of the External Ear p.2
23. This is the medial wall of the tympanic cavity ANSWER:
A. Labyrinth A. Labyrinth
B. Sinus tympani
Tympanic Cavity: aka Tympanum or Middle ear
C. Tegmen tympani
 Divided into 3 depending on their position relative to the Tympanic
D. Scutum Membrane (TM). Arranged from superior to inferior portion:
Epitympanum, mesotympanum, and hypotympanum
o Mesotympanum (Medial to the Tympanic membrane)
 Summary of boundaries:
 Anteriorly are the Eustachian tube, carotid artery and the
semicanal of the tensor tympani muscle;
 Medially is the cochlear promontory;
 Cochlear promontory (corresponds to the basal turn of the
cochlea) forms the medial wall, marked posteriorly by the
oval window superiorly occupied by the stapes and round
window inferiorly
 **Cochlea = part of inner ear / labyrinth
 Posterosuperiorly is the oval window;
 Posteroinferiorly is the round window;
 A space in between and is further posterior to the round &
oval window is the sinus tympani.

[ENT] T.04 – Anatomy, Physiology, and Diseases of the Middle Ear p. 1-2
24. Which of the following types of acute otitis media is the ANSWER:
most common C. Bacterial otitis media
A. Viral myringitis
Four Types of AOM
B. Secretory otitis media
1) Bacterial Otitis Media
C. Bacterial otitis media ○ The usual and most common type of middle ear infection
D. Necrotizing otitis media 2) Viral Myringitis
3) Necrotic Otitis Media
4) Secretory Otitis Media

[ENT] T.03 - Diseases of the External Ear p.3


25. Which of the following antibiotics is used in the primary ANSWER:
treatment of acute otitis media A. Cefuroxime
A. Cefuroxime
Management of AOM
B. Clindamycin
 Antimicrobial therapy (7-10 days)
C. Ciprofloxacin o Amoxicillin/Ampicillin (first-line)
D. Metronidazole o Erythromycin and sulfisoxazole
o TMP-SMX
o Amoxicillin-clavulanate
o Cephalosporins

[ENT] T.03 - Diseases of the External Ear p.3

3 | 7 EARS, NOSE, AND THROAT Midterms Examination Editors | ENT TRANS TEAM
26. Which of the following is a condition that can be found in ANSWER:
a “safe ear”? C. Infected Eustachian Tube
A. Poorly aerated or sclerotic mastoid air cells
B. Peripherally located tympanic perforation
C. Infected Eustachian tube
D. Disease in attic or antrum

SAFE EAR

UNSAFE EAR

[ENT] T.03 - Diseases of the External Ear p.4


27. Which is considered a conservative treatment of chronic ANSWER:
otitis media? A. Removal of keratin debris
A. Removal of keratin debris
Conservative treatment of COM
B. Atticotomy
○ Removal of entrapped keratin: Direct or Saline irrigation
C. Myringotomy ○ Irrigation with 1:1 distilled white vinegar and 70% isopropyl alcohol for
D. Mastoidectomy stabilization
[ENT] T.03 - Diseases of the External Ear p.5
28. When is the highest incidence of acute otitis media? ANSWER:
A. 48-60 months of age C. 6-24 months of age
B. 5-36 months of age Incidence of AOM
○ Highest incidence occurs between 6 and 24 months of age
C. 6-24 months of age
○ Male predilection
D. 2-12 months of age ○ Crowding, poor hygiene, inadequate nutrition, delay in seeking medical
attention
[ENT] T.03 - Diseases of the External Ear p.3
29. Which of the following is a sign/symptom of acute otitis ANSWER:
media? A. Fever
A. Fever
Signs and Symptoms of AOM
B. A pearly white TM on otoscopy
 TM opaque, bulging, or congested
C. Vertigo
 Limited or no mobility of TM on pneumatic otoscopy
D. Hearing loss  Otorrhea
 Otalgia
 Fever
[ENT] T.03 - Diseases of the External Ear p.3
30. JL.R. 31/M came in at the OPD due to persistent dizziness ANSWER:
lasting for days accompanied by bouts of nausea and 30. B. Unidirectional horizontal
vomiting. He had flu-like symptoms a week prior. On 31. A. Truncal ataxia
32. A. Vestibular Neuritis
physical exam, nystagmus is…
33. C. Corticosteroids
A. Upward beating
B. Unidirectional horizontal
C. Bidirectional downward and torsional Vestibular Neuritis:
D. Absent  Inflammation of the vestibula nerve or the vestibular nuclei
31. (Refer to #30) Other symptoms noted include:  Vestibular neuritis typically presents with the sudden onset of severe
A. Truncal ataxia vertigo and vegetative symptoms. The dizziness lasts days, with
B. Hearing loss gradual, definite improvement throughout the course.
C. Inflamed tympanic membrane  Vertigo without hearing loss lasting to days to weeks
 Results as complication of upper respiratory tract infection (URTI)
D. None of the above
 Virus affects vestibular nuclei and/or vestibular nerve
32. (Refer to #30) What is your most probable diagnosis?
 Treatment for vestibular neuritis is steroids.
A. Vestibular Neuritis
B. BPPV
C. Vestibular migraine
D. Meniere’s
33. (Refer to #30) Treatment options
A. Vestibulosuppresants
B. Low salt diet
C. Corticosteroids
D. Diuretics [ENT] T.06 – Diseases of the Inner Ear p. 6

4 | 7 EARS, NOSE, AND THROAT Midterms Examination Editors | ENT TRANS TEAM
34. Passageway for nerve fibers in the cochlea ANSWER:
A. Modiolus 34. A. Modiolus
B. Helicotrema 35. C. Spiral lamina
36. D. cochlear aqueduct
C. Spiral lamina
37. B. Helicotrema
D. Cochlear aqueduct
35. Point of attachment for basement membrane
A. Modiolus COCHLEA
B. Helicotrema  Modiolus:
C. Spiral lamina o central core of the cochlea which is a highly porous bone -
D. Cochlear aqueduct passageway for nerve fibers from the internal auditory meatus to
36. Bony channel that allows communication between the hair cell synapses
 Spiral lamina
perilymphatic fluid and CSF
o coils around the center (around the modiolus)
A. Modiolus o Partial division of ST (scala tympani) and SV (scala vestibuli)
B. Helicotrema o Point of attachment for BM (basilar membrane)
C. Spiral lamina  Helicotrema
D. Cochlear aqueduct o at the apex
37. Communication between scala vestibuli and scala o Communication between SV and ST
tympani  Cochlear aqueduct
A. Modiolus o bony channel for the communication between perilymphatic fluid
B. Helicotrema and CSF
C. Spiral lamina
D. Cochlear aqueduct
[ENT] T.05 - Anatomy and Physiology of the Inner Ear p.1
38. Which statement is correct? ANSWER:
A. The fluid systems in the inner ear is responsible for cellular A. The fluid systems in the inner ear is responsible for cellular
depolarization creating a synaptic activation depolarization creating a synaptic activation
B. The perilymph is located within the membranous labyrinths
FLUID SYSTEMS IN THE INNER EAR
C. High K+, low Na+ is characteristic of perilymph  Perilymphatic system and the Endolymphatic system
D. The endolymph is located between osseous and  Crucial because it will create an environment that will result to
membranous labyrinths Mechanical displacement of BM traveling wave
 Cellular depolarization resulting to a synaptic activity

PERILYMPH ENDOLYMPH
Between osseus and
Within membranous labyrinths;
membranous labyrinths (with
maintained by cells of stria vascularis
scala tympani and vestibuli)
High K+, low Na+ = same with
Intracellular envt (Potassium
High Na+, low K+ = similar
maintains the large positive electric
to the Blood, CSF
gradient within the endolymph
compared to perilymph)
Scala vestibuli, Scala
tympani and internal spaces Scala media
of the organ of Corti
Endolymphatic system produces
Communicates with Endocochlear potential – large
cerebrospinal fluid via cochlear electrochemical gradient → +60 to
aqueduct +100 mV relative to the perilymph.

Accomplished by the stria vascularis


→ contains multiple active ion
Likely conduit for bacterial
channels and maintains the chemical
meningitis into the inner ear
composition of the endolymph and its
positive electrical potential
Disorders include endolymphatic
hydrops (Ménière disease) or wide
vestibular aqueduct

[ENT] T.05 - Anatomy and Physiology of the Inner Ear p. 2


39. What is the difference of Speech reception threshold ANSWER:
(SRT) and Speech discrimination threshold (SDT)? D. SRT uses spondee words
A. SRT has an expected score of 94 of 100%
 SDT has an expected score of 94 of 100%
B. SDT agrees closely with the average of pure tone at 500 Hz,
 SRT agrees closely with the average of pure tone at 500 Hz, 1000 Hz,
1000 Hz, and 2000 Hz and 2000 Hz
C. SRT is presented at 20 to 40 dB above SDT or the patient’s  SDT is presented at 20 to 40 dB above SDT or the patient’s most
most comfortable level comfortable level
D. SRT uses spondee words  SRT uses spondee words

[ENT] T.07 - Audiology p.7

5 | 7 EARS, NOSE, AND THROAT Midterms Examination Editors | ENT TRANS TEAM
40. Which of the following tests is considered the golden ANSWER:
standard test for auditory? B. Audiotory Braintem Evoked Response
A. Pure tone audiometry
Brainstem Evoked Response Audiometry (BERA)
B. Auditory Brainstem evoked response
 It measures the potential arising in the auditory nerve and brainstem
C. Play audiometry structures with a latency of approximately up to 10msec.
D. Cardiovascular audiometry  It is an accurate measure of the auditory function & is completely
objective
o the golden standard for audiometry
o Results cannot be cheated because the electric signal movements
will be measured
[ENT] T.07 - Audiology p.8
41. In Weber’s test, when the patient reports the sound to ANSWER:
lateralize to the poorer ear, the loss is described to be: A. Conductive
A. Conductive Weber’s Test Lateralization
B. Sensorineural  To the poorer ear - hearing loss is conductive
 To the better ear - hearing loss is sensorineural
C. Mixed
D. Normal hearing [ENT] T.07 - Audiology p.3
42. How is audiometric zero defined? ANSWER:
A. The total absence of sound D. The median average hearing threshold of young adults with no
B. The median average hearing threshold of the general history of hearing problem, ear infection or recent colds
population
Audiometric Zero (ANSI)
C. The median average hearing threshold of young children  The median average hearing threshold of young adults with no
with no history of ear infection or recent colds history of hearing problem, ear infection or recent colds.
D. The median average hearing threshold of young adults with  Each frequency has its separate zero with zero calibrated values built
no history of hearing problem, ear infection or recent colds into the audiometer.
[ENT] T.07 - Audiology p.5
43. What is the criteria of conductive hearing loss on an ANSWER:
audiometry? C. When bone conduction thresholds are better than air conduction
A. When bone conduction thresholds are the same as air by 10 db or more & are normal
conduction & neither is normal
Air conduction-Bone conduction Relationship
B. When bone conduction thresholds are reduced but are still  When BC thresholds are better than air conduction by 10 db or
better than air conduction by 10 db or more more and are normal, the loss is conductive
C. When bone conduction thresholds are better than air  When BC thresholds are the same as air conduction and neither is
conduction by 10 db or more & are normal normal, the loss is sensorineural
D. None of the above  When BC thresholds are reduced but are still better than air conduction
by 10 db or more, the loss is mixed or combined

[ENT] T.07 - Audiology p.5


44. What is the maximum decibels HTL for a normal hearing ANSWER:
threshold? D. 25 dB
A. 10 dB
B. 15 dB
C. 20 dB
D. 25 dB

 It is acceptable if the patient would hear deviation by 25 dB hearing


threshold.
 25-40 dB is mild hearing loss
 40-60 dB is moderate hearing loss
 60-80 dB is severe hearing loss
 >80 dB is profound hearing loss
[ENT] T.07 - Audiology p.5
45. All of the following are characteristics of primary bone ANSWER:
healing. EXCEPT D. Differentiation cascade takes place
A. No motion across the fracture Primary bone healing
B. increased amount of intracortical remodeling inside and in  No motion across the fracture
between fragment ends  Increased amount of intracortical remodeling inside and in between
C. doesn't proceed through the entire differentiation cascade fragment ends
D. differentiation cascade takes place  Doesn’t proceed through the entire differentiation cascade
 Achieved through rigid fixation
[ENT] T.08 - Maxillofacial Trauma p.13

6 | 7 EARS, NOSE, AND THROAT Midterms Examination Editors | ENT TRANS TEAM
46. Recommended management for mandibular fractures that ANSWER:
are favorable, nondisplaced, nonmobile, unchanged pre- A. Observation and soft diet
traumatic occlusion.
A. observation and soft diet
B. closed reduction with immobilization using MMF for 4-6
weeks
C. ORIF using titanium plates and screws
D. interdental wiring [ENT] T.08 - Maxillofacial Trauma p.13
47. A patient came in at the ER sustaining a low lateral blow ANSWER:
to the body of the mandible. The primary conventional x- F. All of the above
ray to request is/are?
Mandibular Series
A. AP
 AP
B. Oblique
 Oblique
C. High Townes  High Towne’s
D. Mandibular series
E. A, B, C
F. All of the above [ENT] T.08 - Maxillofacial Trauma p.4
48. Describes the normal intercanthal distance. EXCEPT ANSWER:
A. it is half of normal interpupillary distance B. 45mm
B. 45mm
C. approximately equal to each palpebral width
D. may indicate NOE fracture

[ENT] T.08 - Maxillofacial Trauma p.2


49. A 44 y/o Male came in the ER after hitting his motorcycle ANSWER:
to a narra tree. Alcohol intoxication and no helmet worn B. Drain hematoma
were elicited in the history. Upon further assessment, you
Incision and Drainage
noticed an expanding hematoma on the oral cavity. Your
next step would be?
A. Tracheostomy
B. Drain hematoma
C. Immobilize fractured mandible using bandage
D. Immobilize fractured mandible using arch bars and
interdental wirings
50. A 22 y/o Male sustained a nasal bone fracture after he ANSWER:
was accidentally hit during a basketball game 3 hours ago. F. B and C
Upon assessment, the nose appears to be asymmetrical Nasal Bone Fracture
and swollen. No active epistaxis noted. No septal  Observation
o Nondisplaced
hematoma. Your next step is? o No significant deformities
A. Closed reduction immediately o No clinical evidence of airway obstruction
B. Follow up after 2-3 days  Closed Reduction
C. Give analgesics, decongestants and nasal spray o Unilateral or bilateral fractures of the nasal bone with deformity
D. Open reduction o TIMING:
E. A and B  ASAP or 1-2h after injury: NO SWELLING YET
F. B and C  Wait after 2-3 days: If SWELLING IS PRESENT
G. A, B, C  Open Reduction
o If you have done closed reduction but there is still persistent
deformity
o Extensive fracture dislocation of the nasal bone and septum
o Fracture dislocation of the lower septum
o Open septal fractures
o Inadequate bony reduction due to deformity
o Combined deformities of alar and septal cartilages

[ENT] T.08 - Maxillofacial Trauma p.12

7 | 7 EARS, NOSE, AND THROAT Midterms Examination Editors | ENT TRANS TEAM
RECALLS: EARS, NOSE, AND THROAT
Final Examination ● 2nd Semester

Final Examination
QUESTION ANSWER/RATIONALE
1. What is the patient’s head position in relation to that of the ANSWER:
examiner? B. Patient’s head should be higher than that of the examiner
A. Patient’s head should be at same level as that of the Patient
examiner  Seated with the head slightly higher than that of the examiner’s
B. Patient’s head should be higher than that of the examiner head
C. Patient’s head should be lower than that of the examiner  Slightly leaning forward (not leaning on the chair) and back straight with
D. Head position is not important as the examiner can always their chin slightly protruding
tilt the head upwards  Legs should be uncrossed and flat on the floor.
 Patient’s feet should be close to the body as possible so that the
examiner can comfortably approach the patient.
[ENT] T.01 - Basic ENT Examination p.1.
2. What is the optimal distance of examiner’s head from that ANSWER:
of the patient? C. At the focal length of the head mirror being used
A. Not important so as long as the examiner can see the field
Focal length of 14 inches
being examined
 This is the optimum or best length in which you have a small area with
B. At the examiner’s comfort the brightest reflection you can get
C. At the focal length of the head mirror being used  This means that the examiner’s head is approximately 14 inches away
D. Approximately 6 inches from the patient.
[ENT] T.01 - Basic ENT Examination p.1
3. In doing an indirect laryngoscopy, what can be done to ANSWER:
avoid a gag reflex. D. Application of topical anesthesia
A. Reassure the patient
If patient gags (maybe anxious) – ask patient to breathe rapidly through
B. Perfect technique in doing the examination
the mouth to desensitize the entire area or you can ask the patient to pant
C. Ask the patient to hold his/her breath to dampen the gag reflex or apply a topical anesthesia spray
D. Application of topical anesthesia
[ENT] T.01 - Basic ENT Examination p.4
4. What is the first turbinate that you can visualize when you ANSWER:
do an anterior rhinoscopy? C. Inferior turbinate
A. Superior turbinate
B. Middle turbinate  Inferior Turbinate: large protruding mass like structure, moist, pinkish
and covered with mucus; the most distinct structure seen in internal
C. Inferior turbinate rhinoscopy
D. Supreme turbinate  Middle Turbinate: barely can be seen

[ENT] T.01 - Basic ENT Examination p.3


5. Which part of the temporal bone is a bulbous bony ANSWER:
structure shaped by the expansion of air filled spaces C. Mastoid
within.
Mastoid: A bulbous bony structure shaped by the expansion of air-filled
A. Petrous
spaces within
B. Tympanic
C. Mastoid
D. A&B [ENT] T.02 – Anatomy and Physiology of the External Ear p.2
6. Forms the lateral wall of the middle cranial fossa ANSWER:
A. Squamosa A. Squamosa
B. Tympanic
Squamosa: Forms the lateral wall of the middle fossa. Consists of a
C. Mastoid
plate of bone with an anterior extension known as the zygomatic process,
D. Petrous which forms the bony roof of the glenoid fossa.
[ENT] T.02 – Anatomy and Physiology of the External Ear p.1
7. Muscle attached to the mastoid ANSWER:
A. Trapezius B. Posterior belly of digastric
B. Posterior belly of digastric
Muscles attached to the Mastoid: SCM and Posterior belly digastric
C. Anterior belly of digastric
D. Platysma [ENT] T.02 - Anatomy and Physiology of the External Ear p.2
8. Major blood supply to the EAC ANSWER:
A. External carotid artery A. External carotid artery
B. Superficial temporal artery Major blood supply to auricle and EAC: External Carotid Artery
C. Internal carotid artery
D. Anterior auricular artery [ENT] T.02 - Anatomy and Physiology of the External Ear p.4
9. A 18 year old female came in for lesions on her right ear. ANSWER:
On inspection there were erythematous papules forming A. Psoriasis
large patches with scaling. What is your probable Psoriasis:
diagnosis?  Etiology is unknown
 Multifactorial causes
A. Psoriasis
 Erythematous papules from large patches with thick scale
B. Seborrheic dermatitis
 Epithelial hyperplasia with hyperkeratosis
C. Contact dermatitis
D. Otomycosis [ENT] T.03 – Diseases of the External Ear p.5

1 | 13 EARS, NOSE, AND THROAT Final Examination Editors | ENT TRANS TEAM
10. The following condition is caused by Group A beta ANSWER:
hemolytic streptococci. B. Erysipelas
A. Perichondritis Erysipelas
 Acute superficial cellulitis
B. Erysipelas
 It is a dermal infection that may involve the skin of the head and face,
C. Relapsing polychondritis
including the ear.
D. Necrotizing otitis externa  Group A, beta hemolytic streptococci
 Skin: Bright red; well-demarcated, advancing margin

[ENT] T.03 – Diseases of the External Ear p.5


11. This condition in the external ear can be fatal ANSWER:
A. Psoriasis 11. C. Necrotizing otitis externa
B. Relapsing polychondritis 12. C. Pseudomonas species
C. Necrotizing otitis externa
Necrotizing otitis externa
D. Ramsay-Hunt syndrome  An aggressive and potentially fatal infection originating in the external
12. What is the most common etiologic agent of necrotizing canal, with progressive spread along the soft tissues and bone of the
otitis externa? skull base, ultimately involving intracranial structures. It is also (more
A. Beta hemolytic bacteria accurately) known as necrotizing otitis externa.
B. Streptococcus species  Potentially lethal infection of EAC and surrounding structures
C. Pseudomonas species  Typically seen in diabetics and immunocompromised patients
D. Malassezia furfur  Pseudomonas aeruginosa is the usual culprit followed by S. aureus.
 Aspergillus sp are the most common fungal pathogens to cause this
disorder, but infections by more exotic molds have also been reported.

[ENT] T.03 – Diseases of the External Ear p.5


13. A 50 year old male complains of lesions on the external ANSWER:
ear which is described as raised, erythematous with a 13. A. Seborrheic dermatitis
grayish scale and thick lichenification. He is a known 14. D. Malasezzia furfur
frequent swimmer and uses a behind the ear hearing aid.
Seborrheic dermatitis
What is the most probable diagnosis?  Malasezzia furfur has a role
A. Seborrheic dermatitis  Erythematous
B. Psoriasis  Raised with a greasy scale
C. Contact dermatitis  Pink or orange and thick lichenification if chronic o Topical steroids
D. Otomycosis and keratolytic
14. Which of the following is associated with seborrheic
dermatitis?
A. Beta hemolytic bacteria
B. Streptococcus species
C. Pseudomonas species
D. Malassezia furfur [ENT] T.03 – Diseases of the External Ear p.4
15. Superior boundary of the middle ear ANSWER:
A. Tegmen 15. A. Tegmen
B. Aditus 16. B. Aditus
C. Internal Jugular Vein
Boundaries of Middle Ear
D. Eustachian tube  Superior/Roof – epitympanum, tegmen tympani
16. Posterior boundary of the middle ear  Inferior/Floor – hypotympanum, Interior Jugular Vein
A. Tegmen  Lateral – Tympanic membrane, scutum
B. Aditus  Anterior – Eustachian tube, canal for the TT muscle (Huguier’s Canal)
C. Internal Jugular Vein  Posterior – Aditus
D. Eustachian tube  Medial – promontory (basal turn of cochlea), oval and round windows,
horizontal portion of CN VII, LS

[ENT] T.03 - Diseases of the External Ear p.2


17. It is the conduit through which air is exchanged between ANSWER:
the middle ear space and upper aerodigestive tract. D. Eustachian tube
A. Duct of Rivinus
The ET is the conduit through which air is exchanged between the
B. Oval window
middle ear space and upper aerodigestive tract. It is angled by
C. Round window approximately 45 degrees from the middle ear to the nasopharyngeal
D. Eustachian tube opening at the torus tubarius.

Paul W. Flint MD, in Cummings Otolaryngology: Head and Neck Surgery, 2021
18. A possible route of spread for tumors in the EAC into the ANSWER:
parotid B. Fissure of Santorini
A. Foramen of Huschke
Two patterns of Cancer spread in the EAC
B. Fissure of Santorini
 Fissures of Santorini: vestigial lymph channels that drain into
C. Stylomastoid foramen superficial PG (parotid gland); Provide avenues of spread to the
D. Tympanomastoid suture line superficial lobe of the gland.
 Foramen of Huschke: Serve as a means for extension of malignant
tumors from the EAC to the deep lobe of the parotid gland.

[ENT] T.02 - Anatomy and Physiology of the External Ear p.4

2 | 13 EARS, NOSE, AND THROAT Final Examination Editors | ENT TRANS TEAM
19. Which of the following antibiotics is used in the primary ANSWER:
treatment of acute otitis media A. Cefuroxime
A. Cefuroxime Management of AOM
 Antimicrobial therapy (7-10 days)
B. Clindamycin
o Amoxicillin/Ampicillin (first-line)
C. Ciprofloxacin o Erythromycin and sulfisoxazole
D. Metronidazole o TMP-SMX
o Amoxicillin-clavulanate
o Cephalosporins
[ENT] T.03 - Diseases of the External Ear p.3
20. Which is considered a conservative treatment of chronic ANSWER:
otitis media? A. Removal of keratin debris
A. Removal of keratin debris
Conservative treatment of COM
B. Atticotomy
○ Removal of entrapped keratin: Direct or Saline irrigation
C. Myringotomy ○ Irrigation with 1:1 distilled white vinegar and 70% isopropyl alcohol for
D. Mastoidectomy stabilization
[ENT] T.03 - Diseases of the External Ear p.5
21. When is the highest incidence of acute otitis media? ANSWER:
A. 48-60 months of age C. 6-24 months of age
B. 5-36 months of age Incidence of AOM
○ Highest incidence occurs between 6 and 24 months of age
C. 6-24 months of age
○ Male predilection
D. 2-12 months of age ○ Crowding, poor hygiene, inadequate nutrition, delay in seeking medical
attention
[ENT] T.03 - Diseases of the External Ear p.3
22. Which of the following is a sign/symptom of acute otitis ANSWER:
media? A. Fever
A. Fever
Signs and Symptoms of AOM
B. A pearly white TM on otoscopy
 TM opaque, bulging, or congested
C. Vertigo
 Limited or no mobility of TM on pneumatic otoscopy
D. Hearing loss  Otorrhea
 Otalgia
 Fever
[ENT] T.03 - Diseases of the External Ear p.3
23. Passageway for nerve fibers in the cochlea ANSWER:
A. Modiolus 23. A. Modiolus
B. Helicotrema 24. C. Spiral lamina
25. D. cochlear aqueduct
C. Spiral lamina
COCHLEA
D. Cochlear aqueduct  Modiolus:
24. Point of attachment for basement membrane o central core of the cochlea which is a highly porous bone -
A. Modiolus passageway for nerve fibers from the internal auditory meatus to
B. Helicotrema the hair cell synapses
C. Spiral lamina  Spiral lamina
D. Cochlear aqueduct o coils around the center (around the modiolus)
25. Bony channel that allows communication between o Partial division of ST (scala tympani) and SV (scala vestibuli)
o Point of attachment for BM (basilar membrane)
perilymphatic fluid and CSF
 Helicotrema
A. Modiolus o at the apex
B. Helicotrema o Communication between SV and ST
C. Spiral lamina  Cochlear aqueduct
D. Cochlear aqueduct o bony channel for the communication between perilymphatic fluid
and CSF
[ENT] T.05 - Anatomy and Physiology of the Inner Ear p.1
26. Protects the inner ear from acoustic trauma, discriminate ANSWER:
transient sounds from background noise. A. Olivocochlear reflex
A. Olivocochlear reflex
EFFERENT AUDITORY SYSTEM
B. Vestibulocular reflex
 Middle ear muscle reflex – protective by the contraction of stapedius
C. Vestibulospinal reflex and tensor tympani muscles
D. Vestibulocollic reflex  Olivocochlear reflex – protect from acoustic trauma, discriminate
transient sounds from background noise

[ENT] T.05 - Anatomy and Physiology of the Inner Ear p.4


27. E.D. 39/M works as a fisherman came at the OPD due to ANSWER:
gradual decrease in hearing of both ears for 2 years C. Request for audiometry
R>L. History reveals he has been working as a rock star
Audiometry: Tool for hearing evaluation
for 25 yrs. No other signs and symptoms noted. Patient
has normal otoscopic findings. What is the next best
thing to do?
A. Request for CT Scan
B. Observe
C. Request for audiometry
D. Request for hearing aid fitting
[ENT] T.07 – Audiology p. 3

3 | 13 EARS, NOSE, AND THROAT Final Examination Editors | ENT TRANS TEAM
28. (Refer to #27) Audiometry showed a notching at 4 kHz. ANSWER:
What is your diagnosis? D. Noise induced HL
A. Conductive HL
4000 hz notching is an audiometric signature for noise-induced hearing
B. Mixed HL
loss.
C. Sensorineural HL
D. Noise induced HL [ENT] T.07 – Audiology p. 6
29. (Refer to #27) What will you advise your patient? ANSWER:
A. Change career D. Advise to wear earplugs and minimize noise exposure
B. Advise to stop dynamite fishing
C. Advise hearing aid
D. Advise to wear earplugs and minimize noise exposure
30. What is the allowable duration of unprotected exposure to ANSWER:
bomb, gunshot explosions according to OSHA? D. <15 mins
A. 1-2 hrs  Gunshot: 140 dB SPL
B. 15-30 mins  Permissible exposure for continuous noise under traumatic injury
Level in dB SPL Maximum Exposure Duration (hr)
C. 30-45 mins
90 8
D. <15 mins
95 4
100 2
105 1
110 0.5
115 0.25
[ENT] T.07 – Audiology p. 1, 11
31. Which of the following tests is considered to be the most ANSWER:
accurate measurement of auditory function and is B. Brainstem Evoked Response Audiometry
completely objective?
Brainstem Evoked Response Audiometry (BERA)
A. Pure tone audiometry
 It measures the potential arising in the auditory nerve and brainstem
B. Brainstem Evoked Response Audiometry structures with a latency of approximately up to 10msec.
C. Play audiometry  It is an accurate measure of the auditory function & is completely
D. Cardiovascular audiometry objective
o the golden standard for audiometry
o Results cannot be cheated because the electric signal movements
will be measured

[ENT] T.07 - Audiology p.8


32. How is audiometric zero defined? ANSWER:
A. The total absence of sound D. The median average hearing threshold of young adults with no
B. The median average hearing threshold of the general history of hearing problem, ear infection or recent colds
population
Audiometric Zero (ANSI)
C. The median average hearing threshold of young children  The median average hearing threshold of young adults with no
with no history of ear infection or recent colds history of hearing problem, ear infection or recent colds.
D. The median average hearing threshold of young adults with  Each frequency has its separate zero with zero calibrated values built
no history of hearing problem, ear infection or recent colds into the audiometer.

[ENT] T.07 - Audiology p.5


33. What is the maximum decibels HTL for a normal hearing ANSWER:
threshold? D. 25 dB
A. 10 dB
B. 15 dB
C. 20 dB
D. 25 dB

 It is acceptable if the patient would hear deviation by 25 dB hearing


threshold.
 25-40 dB is mild hearing loss
 40-60 dB is moderate hearing loss
 60-80 dB is severe hearing loss
 >80 dB is profound hearing loss
[ENT] T.07 - Audiology p.5

4 | 13 EARS, NOSE, AND THROAT Final Examination Editors | ENT TRANS TEAM
34. What is the criteria of sensorineural hearing loss on an ANSWER:
audiometry? A. When bone conduction thresholds are the same as air conduction
A. When bone conduction thresholds are the same as air & neither is normal
conduction & neither is normal
Air conduction-Bone conduction Relationship
B. When bone conduction thresholds are reduced but are still  When BC thresholds are better than air conduction by 10 db or more
better than air conduction by 10 db or more and are normal, the loss is conductive
C. When bone conduction thresholds are better than air  When BC thresholds are the same as air conduction and neither is
conduction by 10 db or more & are normal normal, the loss is sensorineural
D. None of the above  When BC thresholds are reduced but are still better than air conduction
by 10 db or more, the loss is mixed or combined

[ENT] T.07 - Audiology p.5


35. Uncinate process seen on the lateral nasal wall of nose is ANSWER:
part of: B. Ethmoid bone
A. Maxilla
Uncinate process of the ethmoid bone is a thin hook-like osseous structure
B. ethmoid bone
of the wall of the lateral nasal cavity.
C. palatine bone
D. lacrimal bone https://radiopaedia.org/articles/uncinate-process
36. Nasolacrimal duct opening is situated in the: ANSWER:
A. Superior meatus D. Inferior meatus
B. Middle meatus
The opening of the nasolacrimal duct is located in the corresponding
C. Ethmoid infundibulum
inferior meatus.
D. Inferior meatus
[ENT] T.09 – Anatomy and Physiology of the Nose and Paranasal Sinuses p. 7
37. All of the following are part of kiesselbach plexus except: ANSWER:
A. Ethmoidal artery C. Lesser palatine artery
B. Superior labial artery
Arteries important in the Kieselbach area: LEGS
C. Lesser palatine artery  L- superior Labial artery
D. Sphenopalatine artery  E- anterior and posterior Ethmoidal artery
 G- Greater palatine artery
 S- Sphenopalatine artery

[ENT] T.09 – Anatomy and Physiology of the Nose and Paranasal Sinuses p.11
38. The superior median projection of the ethmoid bone, ANSWER:
which may vary in size and pneumatization. B. Fovea ethmoidalis
A. crista galli  The crista galli projects upward at the median anterior portion of the
B. fovea ethmoidalis cribriform plate and serves as the site of attachment of the falx cerebri.
C. ethmoid labyrinth  The cribriform plate has a medial part and a lateral lamella.
D. cribriform plate  The fovea ethmoidalis extends laterally from the lateral lamella of the
cribriform plate to form the roof of the ethmoid sinuses laterally.
Hyperpneumatization of the frontal sinus or presence of a frontal cell
may be associated with a deep fovea ethmoidalis.
Cummings Otolaryngology 6th ed. 2015 p. 759, 763
39. Which of the following drains into the inferior meatus: ANSWER:
A. Sphenoid Sinus C. Nasolacrimal duct
B. Posterior ethmoids
Inferior Meatus: Opening of Nasolacrimal duct
C. Nasolacrimal duct
D. None of the above [ENT] T.09 – Anatomy and Physiology of the Nose and Paranasal Sinuses p.9.
40. The following are components of the nasal septum, ANSWER:
except: D. Cribriform plate of the ethmoid
A. Vomer
Bony components of the septum:
B. Maxillary crest
 Nasal crest of the palatine bone
C. Perpendicular plate of the ethmoid
 Nasal crest of the maxilla and premaxilla
D. Cribriform plate of the ethmoid  Vomer
 Perpendicular plate of the ethmoid
 Nasal crest of the frontal bone
 Spine of the paired nasal bones

[ENT] T.09 – Anatomy and Physiology of the Nose and Paranasal Sinuses p.10.
41. The superior limit of the nasopharynx is the ANSWER:
A. tip of the epiglottis B. Pharyngeal tonsils
B. pharyngeal tonsils
 Superiorly: Floor of the sphenoid sinus, pharyngeal roof (Pharyngeal
C. soft palate
tonsil)
D. superior turbinate
 Medial to the ET orifice: tubal cartilage forms a projecting lip called the
torus tubarius
 Posteriorly: curve of the first cervical vertebra with its prevertebral
fascia, prevertebral musculature
 Inferiorly: soft palate

[ENT] T.10 – Diseases of the Nose, Paranasal sinuses, and Nasopharynx p. 16

5 | 13 EARS, NOSE, AND THROAT Final Examination Editors | ENT TRANS TEAM
42. All of the following are part of the lateral border of the ANSWER:
nasal cavity except: D. None of the above
A. Turbinates
Lateral wall of the nasal cavity
B. Horizontal plate of palatine bone
 Nasal bone
C. Maxillary bone
 Maxillary
D. None of the above  Inferior concha/turbinate
 Palatine
 Ethmoid
 Sphenoid

[ENT] T.09 – Anatomy and Physiology of the Nose and Paranasal Sinuses p.6.
43. The osteomeatal unit is composed of the following except: ANSWER:
A. Ethmoidal bulla D. Middle turbinate
B. Uncinate process Composed of:
C. Hiatus semilunaris  Maxillary sinus ostia
 Anterior ethmoid cells and their ostia
D. Middle turbinate
 Ethmoid infundibulum
 Hiatus semilunaris
 Middle meatus
 Uncinate process

[ENT] T.09 – Anatomy and Physiology of the nose and Paranasal Sinuses p. 14
44. All of the following describes an angiofibroma except: ANSWER:
A. Occurs in male adolescents 44. B. Originates at the superior margin of the greater palatine
B. Originates at the superior margin of the greater palatine foramen
foramen
45. B. Surgical
C. Can extend intracranially in about 10-36%
D. Presents with unilateral nasal obstruction and epistaxis 46. D. all of the above
45. Mainstay treatment of angiofibroma:
A. Medical Juvenile angiofibroma
B. Surgical  Histologically benign, locally aggressive
C. Chemotherapy  Male adolescents
D. Radiotherapy  Origin: Superior margin of sphenopalatine foramen
46. Possible complications of angiofibroma surgery  Intracranial extension 10%-36%
A. Hemorrhage  Clinical Features:
o Occurs in males 10-25 y.o.; Unilateral nasal obstruction, Epistaxis,
B. cranial nerve injury Facial swelling, Proptosis, Diplopia
C. meningitis  Management: Surgery is the mainstay of treatment
D. all of the above  Complication
o Surgery
 Hemorrhage
 Cranial nerve injury
 Meningitis

[ENT] T.10 – Diseases of the Nose, Paranasal Sinuses, and Nasopharynx p.14-15
47. Type III nasopharyngeal carcinoma ANSWER:
A. keratinizing C. Undifferentiated
B. non keratinizing Pathology of Nasopharyngeal Carcinoma
C. undifferentiated  Type I – keratinizing squamous cell CA
D. none of the above  Type II – nonkeratinizing CA
 Type III – undifferentiated CA (lymphoepithelioma)

[ENT] T.10 – Diseases of the Nose, Paranasal Sinuses, and Nasopharynx p.17
48. The most common bacterial species isolated from the ANSWER:
maxillary sinuses in adults with acute bacterial D. Pseudomonas aeruginosa
rhinosinusitis except: Acute Bacterial Rhinosinusitis
A. Streptococcus pneumonia  Most common bacterial species:
B. Haemophilus influenza o Streptococcus pneumoniae
C. Moraxella catarrhalis o Haemophilus influenzae
D. Pseudomonas aeruginosa o Moraxella catarrhalis
 Other streptococcal spp, anaerobic bacteria
 Staphylococcus aureus

[ENT] T.10 – Diseases of the Nose, Paranasal Sinuses, and Nasopharynx p.7
49. Chronic rhinosinusitis (CRS) is defined as inflammation ANSWER:
of the nasal cavity and paranasal sinuses and/or the A. 12 weeks
underlying bone that has been present for at least:
Chronic rhinosinusitis
A. 12 weeks
 Inflammation of the nasal cavity and paranasal
B. 13 weeks
 Sinuses and/or the underlying bone that has been present for at least
C. 14 weeks 12 weeks
D. 15 weeks
[ENT] T.10 – Diseases of the Nose, Paranasal Sinuses, and Nasopharynx p. 13

6 | 13 EARS, NOSE, AND THROAT Final Examination Editors | ENT TRANS TEAM
50. Which of the following is/are true about the acute bacterial ANSWER:
rhinosinusitis (ABRS) may be made in adults with: D. All of the above
A. Symptoms of a viral upper respiratory infection (URI) that
Acute Bacterial Rhinosinusitis
have not improved after 10 days or worsen after 5 to 10
 Symptoms of a viral upper respiratory infection (URI) that have not
days improved after 10 days or worsen after 5 to 10 days.
B. Symptoms may include nasal drainage, nasal congestion,  Some or all of the following symptoms: nasal drainage, nasal
facial pressure/pain, postnasal drainage, hyposmia/anosmia, congestion, facial pressure/pain, postnasal drainage,
fever, cough, fatigue, maxillary dental pain, and ear hyposmia/anosmia, fever, cough, fatigue, maxillary dental pain, and ear
pressure/fullness pressure/fullness.
C. Duration of symptoms is less than 12 weeks  European Position Paper on Rhinosinusitis and Nasal Polyps (EPOS) --
D. All of the above all cases lasting for < 12 weeks with complete resolution of symptoms
 Most common bacterial species:
o Streptococcus pneumoniae
o Haemophilus influenza
o Moraxella catarrhalis
 Other streptococcal spp, anaerobic bacteria
 Staphylococcus aureus

[ENT] T.10 – Diseases of the Nose, Paranasal Sinuses, and Nasopharynx p.7
51. What do you call when there is sudden deterioration of the ANSWER:
patient’s condition with either worsening of baseline A. Acute Exacerbation of CRS
symptoms or development of additional symptoms in Symptoms lasting for less than 4
Acute rhinosinusitis (ARS)
cases of CRS? weeks with complete resolution
A. Acute exacerbation of CRS Subacute RS
duration between 4 and 12
B. Recurrent ABRS weeks
C. Both Symptoms lasting for more than
Chronic RS (CRS) (with or
D. CRS 12 weeks without complete
without nasal polyps)
resolution of symptoms
≥ 4 episodes per year, each
lasting ≥ 7-10 days with
Recurrent ARS
complete resolution in between
episodes
sudden worsening of baseline
Acute exacerbation of CRS: CRS with return to baseline
after treatment
ENT] T.10 – Diseases of the Nose, Paranasal Sinuses, and Nasopharynx p. 6
52. Cleft lip and palate deformities are the most common ANSWER:
congenital defect of the head. The most common etiology D. Multifactorial
is:
A. Genetic Most common etiology – multifactorial (prevalence varies among ethnic
B. Teratogens groups and within families)
C. Prenatal vitamin deficiency
D. Multifactorial [ENT] T.11 - Cleft Lip and Palate p.1
53. At this age of development, the nasal placodes invaginate ANSWER:
to form the nasal pits creating a ridge of tissue around the A. 5th week AOG
pit: Lateral nasal prominences laterally and Medial nasal 5 WEEKS AOG
prominences medially.  Nasal placodes invaginate to form the nasal pits o
 Creates a ridge of tissue around the pit , called the:
A. 5th week AOG
o Lateral nasal prominences – laterally
B. 6th week AOG o Medial nasal prominences – medially
C. 7th week AOG
D. 8th week AOG [ENT] T.11 - Cleft Lip and Palate p.1
54. This is formed when the paired maxillary prominences ANSWER:
grow medially toward the paired medial nasal B. Upper lip
prominences.
Paired maxillary prominences grow medially toward the paired medial
A. Nasal tip
nasal prominences — UPPER LIP
B. Upper lip
C. Columella
D. Nasal ala [ENT] T.11 - Cleft Lip and Palate p. 1
55. Medial nasal prominences fuse to form ANSWER:
A. Philtrum E. All of the above
B. Medial upper lip  Medial nasal prominences fuse to form:
o Philtrum
C. Nasal tip
o Medial Upper Lip
D. Columella o Nasal Tip
E. All of the above o Collumela
[ENT] T.11 - Cleft Lip and Palate p. 1
56. Maxillary prominences fuse to form ANSWER:
A. Medial aspect of upper lip B. Lateral aspect of upper lip
B. Lateral aspect of upper lip  Maxillary prominences fuse to form:
C. Bridge of nose o Upper cheeks
o Lateral portions of upper lip
D. Nasal ala o Secondary palate
E. All of the above [ENT] T.11 - Cleft Lip and Palate p. 1

7 | 13 EARS, NOSE, AND THROAT Final Examination Editors | ENT TRANS TEAM
57. Lateral nasal prominences fuse to form ANSWER:
A. Collumela D. Nasal ala
B. Nasal tip  Lateral nasal prominences fuse to form:
C. Bridge of nose o Alae of nose
D. Nasal ala [ENT] T.11 - Cleft Lip and Palate p. 1
58. Palatogenesis begins at the end of: ANSWER:
A. 4th Week 58. B. 5th Week
B. 5th Week
59. C. 12th week
C. 6th Week
D. 7th Week Palatogenesis
59. The age of development when there is complete fusion of  Palatogenesis begins at end of 5th week
the palate.  Complete fusion at 12 weeks.
A. 4th week
B. 8th week
C. 12th week
D. 16th week [ENT] T.11 - Cleft Lip and Palate p.1
60. A male newborn baby is referred to you due to lip ANSWER:
deformity. What muscle is affected in complete cleft lip? D. Orbicularis oris
A. Orbicularis oculi Orbicularis oris
B. Levator labii superioris  Main muscle affected in cleft lip
 Forms a complete sphincter around the oral cavity and provides the
C. Risorius
substrate for proper form and function of the lips and mouth.
D. Orbicularis oris
[ENT] T.11 - Cleft Lip and Palate p. 2
61. A 63 y/o male, smoker and alcohol beverage drinker came ANSWER:
into your clinic with an incision biopsy result of his middle 61. B. Do panendoscopic physical examination
jugulodigastric cervical lymph node, right. The result read:
62. E. AOTA
Squamous Cell Carcinoma. As an ENT, what would be your
next step?
A. Schedule for neck dissection Occult Regional Metastatic Squamous Cell Carcinoma
B. Do panendoscopic physical examination  Diagnosed histologically by FNA biopsy/excisional biopsy of a
C. Refer for radiotherapy cervical lymph node
D. Request abdominal ultrasound and chest X-ray  Most common sites that harbor occult primaries commonly missed
by non-ENTs (ie. Nasopharynx, base of tongue, tonsil, and piriform
62. Most common site/s to harbor occult malignant primaries sinus) must remain in the practitioner’s mind during physical
is/are: examination.
A. Nasopharynx
B. Base of the tongue
C. Piriform sinus
D. Tonsils
E. AOTA
F. NOTA [ENT] T.12 - Anatomy, Physiology, and Diseases of the Neck p. 9
63. A 40 y/o, female came in with a gradually enlarging ANSWER:
nonerythematous nonulcerating nontender anterior neck C. In palpating the neck for thyroid nodules, the examiner should
mass. No associated palpitations, easy fatiguability, and stay in front of the patient to inspect key structures such as
trachea and cricoid cartilage
weight loss. Upon physical assessment, you note an
anterior neck mass that moves with deglutition. The Thyroid Neoplasm:
following statements are TRUE. Except.  You have to be on the back of the patient when assessing the neck.
A. Request for a neck ultrasound and thyroid function test Palpate for the cricoid cartilage. Swipe fingers laterally.
B. Overwhelming majority of thyroid nodules are benign, and  An overwhelming majority of thyroid nodules are benign and only about
only about 10% harbor malignancy 10% harbor malignancy
C. In palpating the neck for thyroid nodules, the examiner  Papillary carcinoma is the most common form of thyroid malignancy and
should stay in front of the patient to inspect key accounts for 60-70% of all thyroid cancer
structures such as trachea and cricoid cartilage  Initial evaluation of thyroid nodule
D. Papillary carcinoma is the most common form of thyroid o TSH
o Ultrasound
malignancy and accounts for 60% to 70% of all thyroid
o Fine needle aspiration biopsy
cancer
[ENT] T.12 - Anatomy, Physiology, and Diseases of the Neck p.8
64. What divides the submandibular space into a sublingual ANSWER:
(infections anterior to second molar) and submaxillary A. Mylohyoid line
(infections of second and third molars) compartments? Involvement of the submandibular and sublingual spaces are usually
A. Mylohyoid line secondary to infection in dentition. If the infection exceeds the mylohyoid
B. Geniohyoid line line which divides the sublingual and submandibular spaces, a
C. Digastric line complication called Ludwig’s angina can occur which is definitely an
emergency.
D. Level I line [ENT] T.12 - Anatomy, Physiology, and Diseases of the Neck p. 3
65. These synovial joints serve as the most functionally ANSWER:
important joints in the larynx. A. Cricoarytenoid joints
A. Cricoarytenoid joints
Cricoarytenoid Joints
B. Cricothyroid joint
 The most functionally important joint
C. Epiglottoarytenoid joint
D. Arytenocorniculate joints [ENT] T.13 - Anatomy and Physiology of the Larynx and Trachea p.1

8 | 13 EARS, NOSE, AND THROAT Final Examination Editors | ENT TRANS TEAM
66. Which of the following does NOT define the Carotid ANSWER:
triangle? A. Superior: Hyoid bone
A. Superior: Hyoid bone Carotid (Superior Carotid/Vascular) Triangle
B. Anterior: Superior belly of omohyoid  Superior: Posterior belly of the digastric
C. Posterior: Sternocleidomastoid  Anterior: Superior belly of omohyoid
D. Superior: Posterior belly of the digastric o The omohyoid runs from the hyoid bone to the clavicle
E. NOTA  Posterior: Sternocleidomastoid
 Floor: Parts of the thyrohyoid, hyoglossus, medial and inferior
pharyngeal constrictor muscles
[ENT] T.12 - Anatomy, Physiology, and Diseases of the Neck p. 1
67. A 10 y/o male with an anterior neck mass located inferior ANSWER:
to the hyoid bone. His parents noticed the mass 5 years C. Thyroglossal Duct Cyst
ago but no consult was done until the child has URTI and
Thyroglossal Duct Cyst
the mass has been noticeable to be enlarging and tender.  Most common congenital neck lesion in children
Antibiotics were given at a local RHU. The tenderness  Represents 70% of congenital pediatric neck masses
subsided however the mass still persisted hence sought  Located inferior to the hyoid bone (most common) but can also be
consult at your clinic. Upon examination, the mass is soft found just superior to the hyoid bone
and cystic and elevates upon tongue protrusion. Your  On examination, TGDCs are usually soft and cystic to palpation and
working diagnosis would be? will elevate in the neck with tongue protrusion
A. Branchial cleft cyst  Usually patients come for consultation, when there is already infection.
B. Ranula
[ENT] T.12 - Anatomy, Physiology, and Diseases of the Neck p. 7
C. Thyroglossal duct cyst
D. Lymphangioma
68. TRUE of carotid paraganglioma, Except ANSWER:
A. Most common class of benign vascular neoplasms of the C. Fontaine sign characterized as a bowing and displacing of the
neck and arise from extraadrenal paraganglionic cells internal and external carotid arteries.
derived from the neural crest.
Carotid Paraganglioma
B. Usually presents as a lateral pulsatile soft and elastic to firm  Most common class of benign vascular neoplasms of the neck and arise
non-tender cervical mass, which is mobile laterally but less from extraadrenal paraganglionic cells derived from the neural crest
mobile in the craniocaudal direction because of its  Pathognomonic sign:
adherence to the carotid arteries. o Usually presents as a lateral pulsatile soft and elastic to firm non-
C. Fontaine sign characterized as a bowing and displacing of tender cervical mass, which is mobile laterally but less mobile in the
the internal and external carotid arteries. craniocaudal direction because of its adherence to the carotid
D. Expectant treatment in older, debilitated patients in absence arteries. (Fontaine sign)
of symptoms due to risk of bleeding during operations; may  Pathognomonic in imaging (CT angiogram):
o Lyre sign characterized as a bowing and displacing of the
include radiation
internal and external carotid arteries
 Expectant treatment in older, debilitated patients in absence of
symptoms due to risk of bleeding during operations; may include
radiation.

[ENT] T.12 - Anatomy, Physiology, and Diseases of the Neck p. 8


69. Which among these laryngeal skeletons are paired? ANSWER:
A. Arytenoid cartilage A. Arytenoid cartilage
B. Cricoid cartilage
Laryngeal Skeleton
C. Thyroid cartilage
 Unpaired or single cartilages:  Paired cartilages:
D. Epiglottis o Epiglottis o Arytenoids
o Thyroid Cartilage o Cuneiform Cartilage
o Cricoid Cartilage o Corniculate Cartilage
o Hyoid
[ENT] T.13 - Anatomy and Physiology of the Larynx and Trachea p.1
70. STRAP muscles of the neck are located: ANSWER:
A. Anterior triangle C. Muscular triangle
B. Investing fascia
Muscular triangle
C. Muscular triangle
 Called as the muscular triangle because it is mostly composed of the
D. Superficial layer of the deep cervical fascia STRAP muscles (infrahyoid muscles) which are commonly connected
E. A, B, C to the hyoid and the sternum.
F. AOTA o Mnemonics: TOSS (Thyrohyoid, Omohyoid, Sternohyoid,
Sternothyroid)

[ENT] T.12 - Anatomy, Physiology, and Diseases of the Neck p.1


71. Which of the following does NOT define Occipital triangle? ANSWER:
A. Anterior: Sternocleidomastoid D. Inferior: Clavicle
B. Posterior: trapezius
Occipital Triangle
C. Inferior: Omohyoid
 Anterior: SCM
D. Inferior: Clavicle
 Posterior: Trapezius
 Inferior: Inferior belly of the omohyoid
 Floor: Splenius capitis, Levator scapulae, and the scalene medius and
posterior muscles

[ENT] T.12 - Anatomy, Physiology, and Diseases of the Neck p.2

9 | 13 EARS, NOSE, AND THROAT Final Examination Editors | ENT TRANS TEAM
72. This muscle in the ONLY ABDUCTOR of the vocal folds ANSWER:
A. Interarytenoid muscle C. Posterior cricoarytenoid muscle
B. Lateral cricoarytenoid muscle  Posterior Cricoarytenoid (PCA): Only vocal fold abductor
C. Posterior cricoarytenoid muscle  All intrinsic muscles of the larynx adducts the vocal chords except for
D. Cricothyroid muscle the Posterior Cricoarytenoid
[ENT] T.13 - Anatomy and Physiology of the Larynx and Trachea p.2
73. All intrinsic muscles of the larynx are innervated by this ANSWER:
nerve except for the cricothyroid muscle C. Recurrent laryngeal nerve
A. External branch of Superior laryngeal nerve
All Intrinsic muscles of the larynx are innervated by the Recurrent
B. Internal branch of Superior laryngeal nerve
Laryngeal Nerve except for the Cricothyroid
C. Recurrent laryngeal nerve
D. None of the above [ENT] T.13 - Anatomy and Physiology of the Larynx and Trachea p.2
74. The Cricothyroid muscle is innervated by what nerve ANSWER:
A. External branch of Superior Laryngeal Nerve A. External branch of Superior Laryngeal Nerve
B. Internal branch of Superior Laryngeal Nerve  Superior Laryngeal Nerve (primary sensory) It supplies motor function
C. Recurrent Laryngeal Nerve to only one muscle (Cricothyroid)
D. None of the above o Internal branch: sensory to supraglottic endolarynx
o External branch: motor to cricothyroid muscle

[ENT] T.13 - Anatomy and Physiology of the Larynx and Trachea p.3
75. The size and physical properties of larynx determine pitch ANSWER:
range. Which muscle contraction is true for pitch control? D. All of the above
A. Thyroarytenoid muscle (TA) only, minimal Cricothyroid Pitch Control
muscle (CT): lower range  Contractions of
o Thyroarythenoid (TA) only, Minimal Cricothyroid (CT): Lower range
B. CT with TA: rising pitch o CT with TA: rising pitch
C. CT only: falsetto o CT only: falsetto
D. All of the above
[ENT] T.13 - Anatomy and Physiology of the Larynx and Trachea p.4
76. This procedure is used to view the larynx in speech and ANSWER:
singing C. Flexible endoscopy
A. Mirror laryngoscopy Flexible Laryngoscopy
B. Rigid endoscopy  Advantage: View larynx in speech and singing because this is usually
C. Flexible endoscopy inserted in the nose, Gottic gap more accurate
D. Direct laryngoscopy  Assess nasal cavity and velopharynx; Better for movement problem
than structure or mucosal health
 Disadvantage: Inferior light transport and magnification; distortion of
periphery of image

[ENT] T.13 - Anatomy and Physiology of the Larynx and Trachea p.5
77. True of the trachea ANSWER:
A. From the cricoid down to the sternal angle or below the 77. D. All of the above
carina where it divides into left and right main bronchus. 78. B. Trachealis muscle
B. Mobile cartilagenous and membranous tube
Trachea
C. Approximately 11.25cm long (adults) and 2.5cm in  From the cricoid (C6) down to the sternal angle / below the carina where
diameter it divides into the left and right main bronchus
D. All of the above  Mobile artilaginous and membranous tube
78. Posteriorly, the trachea is attached to the esophagus via  Approximately 11.25 cm long (adult) and 2.5 cm in diameter
the:  The fibroelastic cartilage is kept patent by a U – shaped rings of hyaline
A. Hyaline cartilage cartilage
B. Trachealis muscle o 14 to 20 hyaline cartilage (tracheal rings) – incomplete posteriorly
C. Circular esophageal muscle  Posteriorly, it is attached to the esophagus via the Trachealis muscle
D. All of the above  Lined by pseudostratified columnar ciliated epithelium resting on an
elastic lamina propria

[ENT] T.13 - Anatomy and Physiology of the Larynx and Trachea p.6
79. Majority of the oropharynx and hypopharynx is lined with ANSWER:
what type of epithelium? B. Nonkeratinized stratified squamous epithelium
A. Ciliated pseudostratified columnar epithelium
Mucosal lining of the oropharynx and hypopharynx:
B. Nonkeratinized stratified squamous epithelium
Stratified, nonkeratinized squamous epithelium
C. Keratinized stratified squamous epithelium
D. Simple columnar epithelium [ENT] T.16 – Anatomy, Physiology, and Diseases of the Oropharynx and Hypopharynx p.1
80. What is the superior border of the hypopharynx? ANSWER:
A. Hyoid bone A. Hyoid bone
B. Superior border of the thyroid cartilage
Borders of the hypopharynx
C. Superior border of the epiglottis
 Superior border: hyoid bone, upper esophageal sphincter (UES)
D. Base of the tongue
 Inferior border: cricopharyngeus
 Anterior border: epiglottis, paired aryepiglottic folds and arytenoid
cartilage

Throat Anatomy. By Tjoson Tjoa, Department of Otolaryngology-Head and Neck Surgery, University
of California, Irvine, School of Medicine. July 10, 2013

10 | 13 EARS, NOSE, AND THROAT Final Examination Editors | ENT TRANS TEAM
81. Which portion would have fibers of the pharyngeal ANSWER:
muscles would have fibers that run a transverse direction? C. Constrictor inferioris
A. Constrictor superioris
Killian–Jamieson region: between the oblique and transverse fibers of
B. Constrictor medius
the constrictor pharyngis.
C. Constrictor inferioris
D. None of the above

[ENT] T.16 – Anatomy, Physiology, and Diseases of the Oropharynx and Hypopharynx p.2
82. Which of the following is not true about the pharyngeal ANSWER:
musculature? D. They are all overlapping
A. There is a gap between each muscle
Only the constrictor pharyngis medius and inferior muscles overlap
B. The inferior most fibers joins with the esophagus
C. Almost all fibers run an oblique direction
D. They are all overlapping [ENT] T.16 – Anatomy, Physiology, and Diseases of the Oropharynx and Hypopharynx p.2
83. This weak point of the pharyngeal musculature occurs ANSWER:
between the constrictor pharyngis inferior and the A. Killian triangle
uppermost fibers of the cricopharyngeus muscle.
Killian triangle is between the constrictor pharyngis inferior and the
A. Killian triangle
uppermost fibers of the cricopharyngeus muscle.
B. Killian-Jamieson triangle
C. Lamier’s triangle
D. Killian-Jamieson region [ENT] T.16 – Anatomy, Physiology, and Diseases of the Oropharynx and Hypopharynx p.2
84. This weak point of the pharyngeal musculature occurs ANSWER:
between the oblique and transverse fibers of the D. Killian-Jamieson region
constrictor pharynges.
Killian–Jamieson region: between the oblique and transverse fibers of
A. Killian triangle
the constrictor pharynges.
B. Killian-Jamieson triangle
C. Lamier’s triangle
D. Killian-Jamieson region [ENT] T.16 – Anatomy, Physiology, and Diseases of the Oropharynx and Hypopharynx p.2
85. This weak point of the pharyngeal musculature is bounded ANSWER:
above by the cricopharyngeus and below by the C. Lamier’s triangle
uppermost fibers of the esophageal musculature.
Laimer triangle, which is bounded above by the cricopharyngeus and
A. Killian triangle
below by the uppermost fibers of the esophageal musculature.
B. Killian-Jamieson triangle
C. Lamier’s triangle
D. Killian-Jamieson region [ENT] T.16 – Anatomy, Physiology, and Diseases of the Oropharynx and Hypopharynx p.2
86. This weak point of the pharyngeal musculature is the most ANSWER:
common area of occurrence of hypopharyngeal diverticula A. Killian triangle
A. Killian triangle
Killian triangle: between the constrictor pharyngis inferior and the
B. Killian-Jamieson triangle
uppermost fibers of the cricopharyngeus muscle - common site for the
C. Lamier’s triangle formation of hypopharyngeal diverticula.
D. Killian-Jamieson region
[ENT] T.16 – Anatomy, Physiology, and Diseases of the Oropharynx and Hypopharynx p.2
87. Which of the tonsils is covered with stratified squamous ANSWER:
epithelium? A or D
A. Palatine tonsils
Palatine and lingual tonsils are covered by stratified, nonkeratinized
B. Tubulopharyngeal tonsils
squamous epithelium
C. Pharyngeal tonsils
D. Lingual tonsils [ENT] T.16 – Anatomy, Physiology, and Diseases of the Oropharynx and Hypopharynx p.2
88. Up to what age are your palatine tonsil have pronounced ANSWER:
activity? B. 8-10 years old
A. 6-8 years old
Activity pronounced during childhood until 8-10 years of age, when
B. 8-10 years old
immunologic challenges from the environment induce hyperplasia of the
C. 10-12 years old palatine tonsils.
D. 12-14 years old
[ENT] T.16 – Anatomy, Physiology, and Diseases of the Oropharynx and Hypopharynx p.3

11 | 13 EARS, NOSE, AND THROAT Final Examination Editors | ENT TRANS TEAM
89. In doing a chest radiogram in a patient who ingested ANSWER:
caustic fluids, what are we actually investigating? B. Esophageal perforation
A. Aspiration pneumonia
Chest radiograph must be done to check for mediastinal widening due to
B. Esophageal perforation
esophageal perforation.
C. Esophageal stenosis
D. Extent of mucosal erosion [ENT] T.16 – Anatomy, Physiology, and Diseases of the Oropharynx and Hypopharynx p.4
90. What is the initial appearance of mucosa injured by a ANSWER:
scalding or caustic fluids? C. Erythema
A. Blister formation
Diagnosis – Acute evaluation
B. Ulceration
Begins with a mirror examination of the oral cavity, oropharynx,
C. Erythema hypopharynx, and larynx – mucosa initially appears erythematous and
D. Perforation edematous and later may show epithelial defects and a whitish fibrin
coating.

[ENT] T.16 – Anatomy, Physiology, and Diseases of the Oropharynx and Hypopharynx p.9
91. In accidental foreign body ingestion, where are they ANSWER:
usually embedded/found? C. Palatine tonsil
A. Pyriform sinus
Foreign bodies are most commonly located in the tonsils and at the
B. Vallecula
tongue base. Foreign bodies typically become lodged in the hypopharynx
C. Palatine tonsil or in the upper constriction of the esophagus.
D. Vestibular folds
[ENT] T.16 – Anatomy, Physiology, Diseases of the Oropharynx and Hypopharynx p.4, 10
92. Which of the following is true about rapid streptococcal ANSWER:
tests? C. Asymptomatic patients with positive test should
A. Results can be done in one hour not be treated with antibiotics
B. Has a sensitivity of 75%
Rapid Streptococcal test
C. Asymptomatic patients with positive test should not be  Makes use of colloid-labeled specific antibodies which are placed onto
treated with antibiotics reaction strips along with the pharyngeal smear
D. Streptococcal angina is ruled out if the test is negative in  Sensitivity of 80%to 90% making them useful tools in deciding whether
symptomatic patients to administer antibiotics
 Should be correlated to clinical findings
 Asymptomatic patients with a positive rapid test should not be
placed on antibiotics
 Conversely, a culture should be taken in cases where there is clinical
suspicion of streptococcal tonsillitis but the rapid test is negative

[ENT] T.16 - Anatomy, Physiology, and Diseases of the Oropharynx and Hypopharynx p. 4
93. This acute inflammatory condition of the pharynx is ANSWER:
characterized by rash formation over the entire body with D. Scarlet Fever
perioral pallor
Scarlet fever – Clinical presentation
A. Streptococcal angina
 Rash that begins on the trunk with sparing of the area around the mouth
B. Plaut-Vincent Angina is spared (“perioral pallor”)
C. Diphtheria  Pathognomonic feature is a bright red tongue with a glistening surface
D. Scarlet fever and hyperplastic papillae (“raspberry tongue,”)
 The tonsils are greatly swollen with a deep red color
 Occasionally there is an enanthema of the soft palate with
 hemorrhagic areas

[ENT] T.16 - Anatomy, Physiology, and Diseases of the Oropharynx and Hypopharynx p. 5
94. This acute inflammation of the pharynx is caused by a ANSWER:
bacteria that produces a toxin that causes cell necrosis C. Diphtheria
and ulcerations
Diphtheria
A. Streptococcal angina
 Caused by Corynebacterium diphtheriae, transmitted by droplet
B. Plaut-Vincent Angina inhalation or skin-to-skin contact with an incubation period of 1–5 days
C. Diphtheria  Bacterium produces a special endotoxin that causes epithelial cell
D. Scarlet fever necrosis and ulcerations

[ENT] T.16 - Anatomy, Physiology, and Diseases of the Oropharynx and Hypopharynx p. 5
95. Which of these following is not a measure to treat ANSWER:
peripheral obstructive sleep apnea syndrome? D. Use of muscle stimulants to increase muscle tone during sleep
A. Continuous positive air pressure masks
Peripheral Obstructive Sleep Apnea Syndrome – Treatment
B. Uvulopharyngoplastry
 General measures: weight reduction, abstinence from
C. Weight reduction
 alcohol and nicotine, and avoiding big meals, and avoid the use of
D. Use of muscle stimulants to increase muscle tone during sedatives
sleep  Esmarch splint (a mandibular advancement device), an occlusive splint
that advances the lower jaw
 Continuous positive airway pressure (CPAP) mask
 Uvulopalatopharyngoplasty (UPPP) with tonsillectomy

[ENT] T.16 - Anatomy, Physiology, and Diseases of the Oropharynx and Hypopharynx p. 8

12 | 13 EARS, NOSE, AND THROAT Final Examination Editors | ENT TRANS TEAM
96. Diverticula in the hypopharynx is usually of what type ANSWER:
A. Traction diverticulum C. Both can occur in the hypopharynx
B. Pulsion diverticulum
Diverticula – Two distinct types:
C. Both can occur in the hypopharynx
D. None of the above • Pulsion Diverticula - Mucosa herniates through a weak point in the
muscular coat due to a rise of intraluminal pressure
• Traction diverticula - Which usually form at parabronchial sites due to
scar traction following hilar lymphadenitis and involve all layers of the
esophageal wall

[ENT] T.16 - Anatomy, Physiology, and Diseases of the Oropharynx and Hypopharynx p10
97. Which of the following is/are TRUE regarding the Parotid ANSWER:
Gland? G. All of the above
A. Major salivary gland  One of the major salivary glands
B. Exclusive: Whartin’s Tumor o Parotid Glands
C. 20-30% malignant o Submandibular Glands
D. Most susceptible: Acute suppurative sialadenitis o Sublingual Glands
E. Exclusive: Adenolymphoma  Warthin Tumor/Adenolymphoma: EXCLUSIVELY in the parotid gland
F. A,B,C,D,  Acute suppurative sialadenitis: Parotid gland - most susceptible,
G. All of the above mainly serous saliva
 Neoplasms in the parotid gland: 20% to 30% of lesions will be
malignant

[ENT] T.17 – Anatomy, Physiology, and Diseases of the Salivary Glands p.1, 4, 6, 8
98. Which of the following is NOT a branch of the facial nerve? ANSWER:
A. Temporal C. Maxillary
B. Zygomatic
5 terminal branches of the Facial nerve
C. Maxillary
 Temporal
D. Mandibular
 Zygomatic
E. Cervical  Buccal
F. NOTA  Mandibular
 Cervical

[ENT] T.17 – Anatomy, Physiology, and Diseases of the Salivary Glands p.1, 8
99. Which of the following is/are TRUE of the Submandibular ANSWER:
gland? F. AOTA
A. Major salivary gland
B. Primary producer of saliva in the unstimulated state  One of the major salivary glands
o Parotid Glands
C. Exclusive: Kuttner Tumor o Submandibular Glands
D. Most susceptible: Sialolithiasis o Sublingual Glands
E. Wharton duct is longer, wider, more tortuous, and  Unstimulated state produced primarily: Submandibular glands
angulated against gravity  Kuttner Tumor: EXCLUSIVE in the submandibular gland
F. AOTA  Chronic Sialodenitis: Submandibular gland - Most commonly affected
 Sialolithiasis
o Submandibular gland
 Wharton duct is longer, wider, more tortuous, and angulated
against gravity → Slower salivary flow rate → Poor salivary
egress in the oral cavity → Stasis

[ENT] T.17 – Anatomy, Physiology, and Diseases of the Salivary Glands p.1,3,4,7,8
100. A 56 y/o/M, s/p superficial parotidectomy left came to ANSWER:
your clinic for follow-up. He has been experiencing E. Advise application of antiperspirant
flushing and sweating of the ipsilateral fascial skin during
Diagnosis: Gustatory sweating or Frey syndrome
mastication. After thorough assessment, your first line of
 Flushing and sweating of the ipsilateral fascial skin during mastication
management would be:
 Diagnostics: Minor starch/iodine test: Dark blue spots
A. Observation, as this complication often is transient and  Apply antiperspirant: Glycopyrrolate 1% roll-on lotion
spontaneously resolves  Tympanic neurectomy – for severe or if bothersome
B. Observation, as this complication often is not bothersome
C. Injection of botulinum toxin
D. Surgical intervention via tympanic neurectomy
E. Advise application of antiperspirant [ENT] T.17 – Anatomy, Physiology, and Diseases of the Salivary Glands p.8

13 | 13 EARS, NOSE, AND THROAT Final Examination Editors | ENT TRANS TEAM

You might also like